Mathematical Olympiad in China : Problems and Solutions

Transcription

Mathematical Olympiad in China : Problems and Solutions
Mathematical
Blympiad
in China
Problems and Solutions
This page intentionally left blank
Mathematical
Olympiad
in China
Problems and Solutions
Editors
Xiong Bin
East China Normal University, China
Lee Peng Yee
Nanyang Technological University, Singapore
East China Normal
University Press
World Scientific
Editors
XIONG Bin East China ~ o r m a lUniversity, China
LEE Peng Yee Nanpng Technological University, Singapore
Original Authors
MO Chinese National Coaches Team of
2003 - 2006
English Translators
XIONG Bin East China N O ~ T T UUniversity,
Z~
China
FENG Zhigang shanghai High School, China
MA Guoxuan h s t China Normal University, China
LIN Lei East China ~ormalUniversity, China
WANG Shanping East China Normal university, China
Z m N G Zhongyi High School Affiliated to Fudan University, China
HA0 Lili Shanghai @baa Senior High School, China
WEE Khangping Nanpng Technological University, singupore
Copy Editors
NI Ming
m t China N
O
~
University
Z
press, China
Z M G Ji World Scientific Publishing GI., Singapore
xu Jin h s t China Normal Universitypress, China
V
This page intentionally left blank
Preface
The first time China sent a team to IMO was in 1985. At that time, two
students were sent to take part in the 26th IMO. Since 1986, China has
always sent a team of 6 students to IMO except in 1998 when it was held in
%wan. So far (up to 2006) , China has achieved the number one ranking in
team effort for 13 times. A great majority of students have received gold
medals. The fact that China achieved such encouraging result is due to, on
one hand, Chinese students’ hard working and perseverance, and on the other
hand, the effort of teachers in schools and the training offered by national
coaches. As we believe, it is also a result of the educational system in China,
in particular, the emphasis on training of basic skills in science education.
The materials of this book come from a series of four books (in
Chinese) on Forurzrd to IMO: a collection of mathematical Olympiad
problems (2003 - 2006). It is a collection of problems and solutions of the
major mathematical competitions in China, which provides a glimpse on
how the China national team is selected and formed. First, it is the China
Mathematical Competition, a national event, which is held on the second
Sunday of October every year. Through the competition, about 120
students are selected to join the China Mathematical Olympiad (commonly
known as the Winter Camp) , or in short CMO, in January of the second
year. CMO lasts for five days. Both the type and the difficulty of the
problems match those of IMO. Similarly, they solve three problems every
day in four and half hours. From CMO, about 20 to 30 students are
selected to form a national training team. The training lasts for two weeks
in March every year. After six to eight tests, plus two qualifying
vii
viii
Mathematical Olympiad in China
examinations, six students are finally selected to form the national team, to
take part in IMO in July that year.
Because of the differences in education, culture and economy of
West China in comparison with East China, mathematical
competitions in the west did not develop as fast as in the east. In
order to promote the activity of mathematical competition there,
China Mathematical Olympiad Committee conducted the China
Western Mathematical Olympiad from 2001. The top two winners will
be admitted to the national training team. Through the China
Western Mathematical Olympiad, there have been two students who
entered the national team and received Gold Medals at IMO.
Since 1986, the china team has never had a female student. In
order to encourage more female students to participate in the
mathematical competition, starting from 2002, China Mathematical
Olympiad Committee conducted the China Girls’ mathematical
Olympiad. Again, the top two winners will be admitted directly into
the national training team.
The authors of this book are coaches of the China national team.
They are Xiong Bin, Li Shenghong, Chen Yonggao , Leng Gangsong,
Wang Jianwei, Li Weigu, Zhu Huawei, Feng Zhigang, Wang
Haiming, Xu Wenbin, Tao Pingshen, and Zheng Chongyi. Those
who took part in the translation work are Xiong Bin, Feng Zhigang,
Ma Guoxuan, Lin Lei, Wang Shanping, Zheng Chongyi, and Hao
Lili. We are grateful to Qiu Zhonghu, Wang Jie, Wu Jianping, and
Pan Chengbiao for their guidance and assistance to authors. We are
grateful to Ni Ming and Xu Jin of East China Normal University
Press. Their effort has helped make our job easier. We are also
grateful to Zhang Ji of World Scientific Publishing for her hard work
leading to the final publication of the book.
Authors
March 2007
Introduction
Early days
The International Mathematical Olympiad (IMO) , founded in 1959,
is one of the most competitive and highly intellectual activities in the
world for high school students.
Even before IMO, there were already many countries which had
mathematics competition. They were mainly the countries in Eastern
Europe and in Asia. In addition to the popularization of mathematics
and the convergence in educational systems among different
countries, the success of mathematical competitions at the national
level provided a foundation for the setting-up of IMO. The countries
that asserted great influence are Hungary, the former Soviet Union
and the United States. Here is a brief review of the IMO and
mathematical competition in China.
In 1894, the Department of Education in Hungary passed a
motion and decided to conduct a mathematical competition for the
secondary schools. The well-known scientist, 1. volt Etovos , was the
Minister of Education at that time. His support in the event had made
it a success and thus it was well publicized. In addition, the success of
his son, R . volt Etovos , who was also a physicist , in proving the
principle of equivalence of the general theory of relativity by A .
Einstein through experiment, had brought Hungary to the world stage
in science. Thereafter, the prize for mathematics competition in
Hungary was named “Etovos prize”. This was the first formally
organized mathematical competition in the world. In what follows,
X
Mathematical Olympiad in China
Hungary had indeed produced a lot of well-known scientists including
L. Fejer, G. Szego, T . Rado, A . Haar and M . Riesz (in real
analysis), D. Konig ( in combinatorics) , T. von Kdrmdn ( in
aerodynamics) , and 1. C. Harsanyi (in game theory, who had also
won the Nobel Prize for Economics in 1994). They all were the
winners of Hungary mathematical competition. The top scientific
genius of Hungary, 1. von Neumann, was one of the leading
mathematicians in the 20th century. Neumann was overseas while the
competition took place. Later he did it himself and it took him half
an hour to complete. Another mathematician worth mentioning is the
highly productive number theorist P. Erdos. He was a pupil of Fejer
and also a winner of the Wolf Prize. Erdos was very passionate about
mathematical competition and setting competition questions. His
contribution to discrete mathematics was unique and greatly
significant. The rapid progress and development of discrete
mathematics over the subsequent decades had indirectly influenced the
types of questions set in IMO. An internationally recognized prize
named after Erdos was to honour those who had contributed to the
education of mathematical competition. Professor Qiu Zonghu from
China had won the prize in 1993.
In 1934, B. Delone, a famous mathematician, conducted a
mathematical competition for high school students in Leningrad (now
St. Petersburg). In 1935, Moscow also started organizing such event.
Other than being interrupted during the World War II , these events
had been carried on until today. As for the Russian Mathematical
Competition ( later renamed as the Soviet Mathematical
Competition) , it was not started until 1961. Thus, the former Soviet
Union and Russia became the leading powers of Mathematical
Olympiad. A lot of grandmasters in mathematics including A . N.
Kolmogorov were all very enthusiastic about the mathematical
competition. They would personally involve in setting the questions
for the competition. The former Soviet Union even called it the
Mathematical Olympiad, believing that mathematics is the
Introduction
xi
“gymnastics of thinking”. These points of view gave a great impact on
the educational community. The winner of the Fields Medal in 1998,
M. Kontsevich, was once the first runner-up of the Russian
Mathematical Competition. G . Kasparov , the international chess
grandmaster, was once the second runner-up. Grigori Perelman , the
winner of the Fields Medal in 2006, who solved the Poincare’s
Conjecture, was a gold medalist of IMO in 1982.
In the United States of America, due to the active promotion by
the renowned mathematician Birkhoff and his son, together with G .
Polya , the Putnam mathematics competition was organized in 1938
for junior undergraduates. Many of the questions were within the
scope of high school students. The top five contestants of the Putnam
mathematical competition would be entitled to the membership of
Putnam. Many of these were eventually outstanding mathematicians.
There were R . Feynman (winner of the Nobel Prize for Physics,
1965), K . Wilson (winner of the Nobel Prize for Physics, 1982), 1.
Milnor (winner of the Fields Medal, 1962), D. Mumford (winner of
the Fields Medal, 1974), D. Quillen (winner of the Fields Medal,
1978), et al.
Since 1972, in order to prepare for the IMO, the United States of
American Mathematical Olympiad ( USAMO) was organized. The
standard of questions posed was very high, parallel to that of the
Winter Camp in China. Prior to this, the United States had organized
American High School Mathematics Examination (AHSME) for the
high school students since 1950. This was at the junior level yet the
most popular mathematics competition in America. Originally, it was
planned to select about 100 contestants from AHSME to participate in
USAMO. However, due to the discrepancy in the level of difficulty
between the two competitions and other restrictions, from 1983
onwards, an intermediate level of competition, namely, American
Invitational Mathematics Examination ( AIME ) , was introduced.
Henceforth both AHSME and AIME became internationally wellknown. A few cities in China had participated in the competition and
xii
Mathematical Olympiad in China
the results were encouraging.
The members of the national team who were selected from
USAMO would undergo training at the West Point Military Academy,
and would meet the President at the White House together with their
parents. Similarly as in the former Soviet Union, the Mathematical
Olympiad education was widely recognized in America. The book
“HOWto Solve it” written by George Polya along with many other
titles had been translated into many different languages. George Polya
provided a whole series of general heuristics for solving problems of
all kinds. His influence in the educational community in China should
not be underestimated.
International Mathematical Olympiad
In 1956, the East European countries and the Soviet Union took the
initiative to organize the IMO formally. The first International
Mathematical Olympiad (IMO) was held in Brasov, Romania, in
1959. At the time, there were only seven participating countries,
namely , Romania , Bulgaria, Poland , Hungary , Czechoslovakia, East
Germany and the Soviet Union. Subsequently, the United States of
America, United Kingdom, France, Germany and also other
countries including those from Asia joined. Today, the IMO had
managed to reach almost all the developed and developing countries.
Except in the year 1980 due to financial difficulties faced by the host
country, Mongolia, there were already 47 Olympiads held and 90
countries participating.
The mathematical topics in the IMO include number theory,
polynomials, functional equations, inequalities, graph theory,
complex numbers, combinatorics, geometry and game theory. These
areas had provided guidance for setting questions for the
competitions. Other than the first few Olympiads, each IMO is
normally held in mid-July every year and the test paper consists of 6
questions in all. The actual competition lasts for 2 days for a total of 9
hours where participants are required to complete 3 questions each
Introduction
xi
day. Each question is 7 marks which total up to 42 marks. The full
score for a team is 252 marks. About half of the participants will be
awarded a medal, where 1/12 will be awarded a gold medal. The
numbers of gold, silver and bronze medals awarded are in the ratio of
1:2:3 approximately. In the case when a participant provides a better
solution than the official answer, a special award is given.
Each participating country will take turn to host the IMO. The
cost is borne by the host country. China had successfully hosted the
31st IMO in Beijing in 1990. The event had made a great impact on
the mathematical community in China. According to the rules and
regulations of the IMO, all participating countries are required to
send a delegation consisting of a leader, a deputy leader and 6
contestants. The problems are contributed by the participating
countries and are later selected carefully by the host country for
submission to the international jury set up by the host country.
Eventually, only 6 problems will be accepted for use in the
competition. The host country does not provide any question. The
short-listed problems are subsequently translated, if necessary , in
English, French, German, Russian and other working languages.
After that , the team leaders will translate the problems into their own
languages.
The answer scripts of each participating team will be marked by
the team leader and the deputy leader. The team leader will later
present the scripts of their contestants to the coordinators for
assessment. If there is any dispute, the matter will be settled by the
jury. The jury is formed by the various team leaders and an appointed
chairman by the host country. The jury is responsible for deciding the
final 6 problems for the competition. Their duties also include
finalizing the marking standard, ensuring the accuracy of the
translation of the problems, standardizing replies to written queries
raised by participants during the competition, synchronizing
differences in marking between the leaders and the coordinators and
also deciding on the cut-off points for the medals depending on the
xiv
Mathematical Olympiad in China
contestants’ results as the difficulties of problems each year are
different.
China had participated informally in the 26th IMO in 1985. Only
two students were sent. Starting from 1986, except in 1998 when the
IMO was held in Taiwan, China had always sent 6 official contestants
to the IMO. Today, the Chinese contestants not only performed
outstandingly in the IMO, but also in the International Physics,
Chemistry, Informatics, and Biology Olympiads. So far, no other
countries have overtaken China in the number of gold and silver
medals received. This can be regarded as an indication that China
pays great attention to the training of basic skills in mathematics and
science education.
Winners of the IMO
Among all the IMO medalists, there were many of them who
eventually became great mathematicians. Some of them were also
awarded the Fields Medal, Wolf Prize or Nevanlinna Prize ( a
prominent mathematics prize for computing and informatics). In
what follows, we name some of the winners.
G . Margulis , a silver medalist of IMO in 1959, was awarded the
Fields Medal in 1978. L. Lovasz, who won the Wolf Prize in 1999,
was awarded the Special Award in IMO consecutively in 1965 and
1966. V. Drinfeld , a gold medalist of IMO in 1969, was awarded the
Fields Medal in 1990. 1. -C. Yoccoz and T . Gowers, who were both
awarded the Fields Medal in 1998, were gold medalists in IMO in 1974
and 1981 respectively. A silver medalist of IMO in 1985, L.
Lafforgue , won the Fields Medal in 2002. A gold medalist of IMO in
1982, Grigori Perelman from Russia, was awarded the Fields Medal in
2006 for solving the final step of the Poincar6 conjecture. In 1986,
1987, and 1988, Terence Tao won a bronze, silver, and gold medal
respectively. He was the youngest participant to date in the IMO,
first competing at the age of ten. He was also awarded the Fields
Medal in 2006.
Introduction
xv
A silver medalist of IMO in 1977, P. Shor, was awarded the
Nevanlinna Prize. A gold medalist of IMO in 1979, A . Razborov ,
was awarded the Nevanlinna Prize. Another gold medalist of IMO in
1986, S. Smirnov, was awarded the Clay Research Award. V.
Lafforgue, a gold medalist of IMO in 1990, was awarded the
European Mathematical Society prize. He is L. Laforgue’s younger
brother.
Also, a famous mathematician in number theory, N. Elkis, who
is also a foundation professor at Havard University, was awarded a
gold medal of IMO in 1981. Other winners include P. Kronheimer
awarded a silver medal in 1981 and R . Taylor a contestant of IMO in
1980.
MathematicaI competitions in China
Due to various reasons , mathematical competitions in China started
relatively late but is progressing vigorously.
“We are going to have our own mathematical competition too!”
said Hua Luogeng. Hua is a house-hold name in China. The first
mathematical competition was held concurrently in Beijing , Tianjing,
Shanghai and Wuhan in 1956. Due to the political situation at the
time, this event was interrupted a few times. Until 1962, when the
political environment started to improve, Beijing and other cities
started organizing the competition though not regularly. In the era of
cultural revolution, the whole educational system in China was in
chaos. The mathematical competition came to a complete halt. In
contrast, the mathematical competition in the former Soviet Union
was still on-going during the war and at a time under the difficult
political situation. The competitions in Moscow were interrupted only
3 times between 1942 and 1944. It was indeed commendable.
In 1978, it was the spring of science. Hua Luogeng conducted the
Middle School Mathematical Competition for 8 provinces in China.
The mathematical competition in China was then making a fresh start
and embarked on a road of rapid development. Hua passed away in
xvi
Mathematical Olympiad in China
1985. In commemorating him, a competition named Hua Luogeng
Gold Cup was set up in 1986 for the junior middle school students and
it had a great impact.
The mathematical competitions in China before 1980 can be
considered as the initial period. The problems set were within the
scope of middle school textbooks. After 1980, the competitions were
gradually moving towards the senior middle school level. In 1981, the
Chinese Mathematical Society decided to conduct the China
Mathematical Competition, a national event for high schools.
In 1981, the United States of America, the host country of IMO,
issued an invitation to China to participate in the event. Only in 1985,
China sent two contestants to participate informally in the IMO. The
results were not encouraging. In view of this, another activity called
the Winter Camp was conducted after the China Mathematical
Competition. The Winter Camp was later renamed as the China
Mathematical Olympiad or CMO. The winning team would be
awarded the Chern Shiing-Shen Cup. Based on the outcome at the
Winter Camp, a selection would be made to form the 6-member
national team for IMO. From 1986 onwards, other than the year
when IMO was organized in Taiwan, China had been sending a 6member team to IMO every year. China is normally awarded the
champion or first runner-up except on three occasions when the results
were lacking. Up to 2006, China had been awarded the overall team
champion for 13 times.
In 1990, China had successfully hosted the 31st IMO. It showed
that the standard of mathematical competition in China has leveled
that of other leading countries. First, the fact that China achieves the
highest marks at the 31st IMO for the team is an evidence of the
effectiveness of the pyramid approach in selecting the contestants in
China. Secondly, the Chinese mathematicians had simplified and
modified over 100 problems and submitted them to the team leaders of
the 35 countries for their perusal. Eventually, 28 problems were
recommended. At the end, 5 problems were chosen O M 0 requires 6
Introduction
xvii
problems). This is another evidence to show that China has achieved
the highest quality in setting problems. Thirdly, the answer scripts of
the participants were marked by the various team leaders and assessed
by the coordinators who were nominated by the host countries. China
had formed a group 50 mathematicians to serve as coordinators who
would ensure the high accuracy and fairness in marking. The marking
process was completed half a day earlier than it was scheduled.
Fourthly, that was the first ever IMO organized in Asia. The
outstanding performance by China had encouraged the other
developing countries, especially those in Asia. The organizing and
coordinating work of the IMO by the host country was also reasonably
good.
In China, the outstanding performance in mathematical
competition is a result of many contributions from all the quarters of
mathematical community. There are the older generation of
mathematicians, middle-aged mathematicians and also the middle and
elementary school teachers. There is one person who deserves a
special mention and he is Hua Luogeng. He initiated and promoted
the mathematical competition. He is also the author of the following
books: Beyond Yang hui’s Triangle, Beyond the pi of Zu Chongzhi ,
Beyond the Magic Computation of Sun-zi , Mathematical Induction,
and Mathematical Problems of Bee Hive. These books were derived
from mathematics competitions. When China resumed mathematical
competition in 1978, he participated in setting problems and giving
critique to solutions of the problems. Other outstanding books derived
from the Chinese mathematics competitions are: Symmetry by Duan
Xuefu, Lattice and Area by He Sihe, One Stroke Drawing and
Postman Problem by Jiang Boju .
After 1980, the younger mathematicians in China had taken over
from the older generation of mathematicians in running the
mathematical competition. They worked and strived hard to bring the
level of mathematical competition in China to a new height. Qiu
Zonghu is one such outstanding representative. From the training of
xviii
Mathematical Olympiad in China
contestants and leading the team 3 times to IMO to the organizing of
the 31th IMO in China, he had contributed prominently and was
awarded the P. Erdos prize.
Preparation for IMO
Currently, the selection process of participants for IMO in China is as
follows.
First, the China Mathematical Competition, a national
competition for high Schools, is organized on the second Sunday in
October every year. The objectives are: to increase the interest of
students in learning mathematics, to promote the development of cocurricular activities in mathematics, to help improve the teaching of
mathematics in high schools, to discover and cultivate the talents and
also to prepare for the IMO. This happens since 1981. Currently there
are about 200 000 participants taking part.
Through the China Mathematical Competition, around 120 of
students are selected to take part in the China Mathematical Olympiad
or CMO, that is, the Winter Camp. The CMO lasts for 5 days and is
held in January every year. The types and difficulties of the problems
in CMO are very much similar to the IMO. There are also 3 problems
to be completed within four and half hours each day. However, the
score for each problem is 21 marks which add up to 126 marks in
total. Starting from 1990, the Winter Camp instituted the Chern
Shiing-Shen Cup for team championship. In 1991, the Winter Camp
was officially renamed as the China Mathematical Olympiad (CMO) .
It is similar to the highest national mathematical competition in the
former Soviet Union and the United States.
The CMO awards the first, second and third prizes. Among the
participants of CMO, about 20 to 30 students are selected to
participate in the training for IMO. The training takes place in March
every year. After 6 to 8 tests and another 2 rounds of qualifying
examinations, only 6 contestants are short-listed to form the China
IMO national team to take part in the IMO in July.
Introduction
XiX
Besides the China Mathematical Competition (for high schools) ,
the Junior Middle School Mathematical Competition is also developing
well. Starting from 1984, the competition is organized in April every
year by the Popularization Committee of the Chinese Mathematical
Society. The various provinces, cities and autonomous regions would
rotate to host the event. Another mathematical competition for the
junior middle schools is also conducted in April every year by the
Middle School Mathematics Education Society of the Chinese
Educational Society since 1998 till now.
The Hua Luogeng Gold Cup, a competition by invitation, had
also been successfully conducted since 1986. The participating students
comprise elementary six and junior middle one students. The format
of the competition consists of a preliminary round, semifinals in
various provinces, cities and autonomous regions, then the finals.
Mathematical competition in China provides a platform for
students to showcase their talents in mathematics. It encourages
learning of mathematics among students. It helps identify talented
students and to provide them with differentiated learning
opportunity. It develops co-curricular activities in mathematics.
Finally, it brings about changes in the teaching of mathematics.
This page intentionally left blank
Contents
Preface
lntroduction
vii
ix
China Mathematical Competition
1
2
13
24
38
2002
2003
2004
2005
(Jilin)
(Shaanxi)
(Hainan)
(Jiangxi)
China Mathematical Competition (Extra Test)
2002
2003
2004
2005
(Jilin)
(Shaanxi)
(Hainan)
(Jiangxi)
China Mathematical Olympiad
2003
2004
2005
2006
(Changsha, Hunan)
(Macao)
(Zhengzhou, Henan)
(Fuzhou, Fujian)
China Girls’ Mathematical Olympiad
2002
2003
2004
2005
(Zhuhai, Guangdong)
(Wuhan, Hubei)
(Nanchang , Jiangxi)
(Changchun, Jilin)
51
51
55
60
67
74
74
90
99
113
126
126
134
142
156
Mathematical Olympiad in China
xxii
China Western Mathematical Olympiad
166
2002
2003
2004
2005
(Lanzhou, Gansu)
(Urumqi, Xinjiang)
(Yinchuan, Ningxia)
(Chengdu, Sichuan)
International Mathematical Olympiad
166
177
185
195
2003
2004
2005
2006
203
204
213
232
243
(Tokyo, Japan)
(Athens, Greece)
(Mkrida, Mexico)
(Ljubljana, Slovenia)
China Mathematical
Competition
T h e China Mathematical Competition is organiEd in October
every year. The Popularization Committee of the Chinese
Mathematical Society and the local Mathematical Society are
responsible for the assignments of the competition problems.
The test paper consists of 6 choices, 6 blanks and 3
questions to be solved with complete process. The full score is
150 marks. Besides, 3 questions are used in the Extra Test, with
50 marks each. The participants with high total marks in the China
Mathematical Competition plus the Extra Test are awarded the first
prize (1 000 participants around China), and they will be admitted
into the university directly.
The participants with excellent marks are selected to take part
in the China Mathematical Olympiad the next year. Thus, for
Chinese high school students, the China Mathematical Olympiad
is the first step to IMO.
Mathematical Olympiad in China
2
2002
(Jilin)
Popularization Committee of CMS and Jilin Mathematical Society
were responsible for the assignment of the competition problems in
the first and second rounds of the contests.
Part I Multiple-choice Questions (Questions1 to 6 carry 6 marks each.)
@@ The interval on which the function f ( x ) = log+ (2- 2x - 3 ) is
monotone increasing is (
).
(A) (Em, -1)
(€3) (Em, 1)
(C)( l , + m )
(D) (3, +m>
Solution First, we will find the domain of f(x). From 2 - 2x 3 > 0, we obtain x <- 1 or x > 3. So the domain of definition for
f(x)is(--,
-1)U (3,+m). B u t u = 2 - 2 x - 3 =
(~-1)~4 is monotone decreasing on (- 00 , - 1) , and monotone increasing on
(3 , 00). So fcx) = log3 (2- 2x - 3) is monotone increasing on
(- 00 , - 1) , and monotone decreasing on (3 , 00). Answer: A.
+
+
If r e a l n u m b e r s x a n d y s a t i s f y ( ~ + 5 ) ~ + ( y - 1 2 ) ~ = 1 4 ~then
,
the minimum value of 2 3 is (
).
+
(A) 2
(B) 1
(C)43
(D) ./z
Solution Let x + 5 = 14cosOand y - 12 = 14sin0, for 0 E [0, 2x).
Hence
=142
+ 52 + 122
-
=365 +28(12sin8-
1 4 0 ~ 08+
s 336sin 0
5~0~8)
China Mathematical Competition 2002
3
+28 x 13sidO- 9)
=365 + 364sidO- 9) ,
=365
5
where t a n 9 = 12'
So 2
x
5
13
+$ has the minimum value 1, when O=
= - andy
=--.12
13
3x
2
5
i. e.
12
-+arctan-,
Answer: B.
+
Remark A geometric significance of this problem is: ( x + 5 I 2
( y - 1212 = 142is a circle with C(- 5, 12) as center and 14 as radius.
We can find a point P on the circumference of this circle such that
I PO I is minimal, where 0 is the origin of the coordinate system.
We join CO and extend it to intersect the circumference of the circle
at P. Then it follows that I PO I is the minimum value of 2 +y2.
X
X
is (
>.
1-2x
2
(A) an even but not odd function
(B) an odd but not even function
(C)a both even and odd function
(D) a neither even nor odd function
Solution It is easy to see that the domain of f (x) is ((0, +-I.
Whenx E (--, 0) U (0, +-I, we have
@& The function f (x) =
~
-
--
X
1-2x
X
2
-
-,
0)
U
fh>.
Therefore, f ( x ) is an even function, and obviously not an odd
Mathematical Olympiad in China
4
function. Answer: A.
c
% The straight line 4 + 3
.-
I
=
2 + 2= 1 at
1 intersects the ellipse 16
9
two points A and B. There is a point P on this ellipse such that
) such point/
the area of APAB is equal to 3. There is/are (
points P.
(A) 1
(B) 2
(C)3
(D) 4
Solution Suppose that there is a point P(4cosa, 3sina) on the
ellipse. When P and the origin 0 are not on the same side of AB , the
distance from P to AB is
3 ( 4 ~ 0a)+
s 4(3sina)- 12
5
12
5
= -(cosa+sina-1)
< 76 .
But AB
= 5,
1 X5X 6 = 3.
SOAPAB< -
5
2
Therefore, when the area of APAB is equal to 3 , points P and 0
are on the same side of AB . There are two such points P. Answer: B.
&@
It is given that there are two sets of real numbers A =
{ a l , a2,
q o o ) and B = { b l , b 2 , .-, b50). If there is a
mapping f from A to B such that every element in B has an
inverse image and
..a,
f(a1)< f ( a 2 I<
...< f(a1oo
1 9
then the number of such mappings is (
).
(A) C;{o
(B) C%
(C)c%l
(D) C%
Solution We might as well suppose bl < b2 < < b50 , and divide
China Mathematical Competition 2002
5
-
elementsal , a2 , , a100inA into 50 nonempty groups according to
their order. Define a mapping f: A
B, so that the images of all
the elements in the i-th group are bi ( i = 1, 2 , , 5 0 ) under the
mapping. Obviously, f satisfies the requirements given in the
problem. Furthermore, there is a one-to-one correspondence
between all groups so divided and the mappings satisfying the
condition. So the number of mappings f satisfying the requirements
is equal to the number of ways dividingA into 50 groups according to
the order of the subscripts. The number of ways dividing A is C$8.
Then there are, in all, C$8 such mappings. Answer: D.
Remark Since C$g = C$8, Answer B is also true in this problem.
This may be an oversight when the problem was set.
A region is enclosed by the curves 2 = 4y, 2 =- 4y, x = 4 and
x =- 4. V1 is the volume of the solid obtained by rotating the
above region round the y-axis. Another region consists of points
(x,y satisfying 2 y2 16, 2 (y - 2>2 4 and 2 (y
2>2 2 4 . V2 is the volume of the solid obtained by rotating this
region round the y-axis. Then (
).
1
2
(A) VI = yV2
(€3) V1 = 3V2
+ <
(C)VI
=v
2
5'
+
(D) V1
>
+ +
= 2V2
t5'
Solution As shown in the diagram, two solids of rotation obtained
by rotating respectively two regions round the y-axis lie between two
parallel planes, which are 8 units apart. We cut two solids of rotation
by any plane which is perpendicular to the y-axis. Suppose the
Mathematical Olympiad in China
6
distance from the plane to the origin is I y I
sectional areas are
and
S2 = ~
(2-
( - y42 ) - ~
x[4-
so
s
1=
(<4).
I yI 12]= ~
Then the two
( -41
4 yI
~
1.
s,.
From the Zugen Principle (or Cavalieri Principle) , we know that
the volumes of the two geometric solids are equal, and that is, V1 =
V2. Answer: C.
Part I1 Short-answer Questions (Questions7 to 12 carry 6 marks each. )
@& It is given that complex numbers z1 and z2 satisfy I z1 I = 2 and
I z2 I = 3. If the included angle of their corresponding vectors is
60", then
1
z1 +z2
~
z1 -z2
1
=
Solution By the cosine rule, we obtain
I z1 + z2 I
and
=4/
I z1 I 2 + I z2 I 2
-
2 I z1 I
I z1 - z 2 I =2/1 z1 l 2 + I z2 l 2 -21
I z2 I cos 120" =2/19,
z1 I I z2 I cos60"
=fl.
Therefore ,
We arrange the expansion of
1 "
( +29;-)
in decreasing powers of
x . If the coefficients of the first three terms form an arithmetic
progression, then, in the expansion, there are
x with integer power.
terms of
1
Solution The coefficients of the first three terms are 1, TC; and
1
-Ci.
22
By the assumption, we have
China Mathematical Competition 2002
1
-cz,
2
2
That is
=
7
1
+-c$
22
1
1
n = 1+-n(n-1).
8
Solving for n, we obtain n = 8 and n = 1 (not admissible
W h e n n = 8, Tr+l
1
a(2)
x 7 ,r =
16-37
=
0, 1, 2 ,
..a,
8. But
I
must satisfy 4 I 16-3r, so rcan only take 0, 4 and 8. Therefore, there
are 3 terms in the expansion of x with integer power.
&
>
Q
As shown in the diagram, points PI , P2 ,
..a,
PIOare either the
vertices or the midpoints of the edges of a tetrahedron
respectively. Then there are
groups of four points (PI ,
Pi, P j , P k ) (1 < i < j < K 10) on the same plane.
Solution
On each lateral face of the
tetrahedron other than point PI there are five
points. Take any three out of the five points and
add point PI. These four points lie in the same
<
- - - - __ _ _ _ P J
h
face. Hence there are 3@ groups in all for three
P,
1’8
lateral faces.
Furthermore, there are three points on each edge containing
point PI. We add a midpoint taking from the edge on the base, which
is not on the same plane with the edge above. Now, we obtain
another group consisting of four points which also are on the same
plane. There are 3 groups like this.
Consequently, there are 3@ +3 = 33 groups of four points on the
same plane.
@@ It is given that f ( x ) is a function defined on R, satisfying f(1) =
1, and for any x E R,
f(x
+ 5 1 2 fh)+5
9
Mathematical Olympiad in China
8
and
f(x
+ 1I< f(x)+
1.
If g(x) = f(x)+ 1- x, then g ( 2 002) =
Solution We determine f ( 2 002) first. From the conditions given,
we have
+ < f (x+ 5 I< f ( x +4)+ 1
f<x> 5
<f ( x + 2 ) + 3
<f(x + 1I+ 4 < f(x)+ 5.
Thus the equality holds for all. So we have f(x + 1) f(x)+
<f(x+3)+2
1.
Hence, from f(1) = 1, we get f ( 2 ) = 2, f ( 3 ) = 3 , .-,
f ( 2 002) = 2 002. Therefore, g ( 2 002) = f ( 2 002)+ 1- 2 002 = 1.
=
+ + log4(x
@@ If log4(x 2y)
-
2y)
=
1, then the minimum value of
1x1- I Y I is
Solution First, from
x + 2 y > 0,
x -2 y >O ,
(x++y)(x-2y)
=
4,
we obtain
.z
> 21 yI 2 0 ,
2-43?
=
4.
By the symmetry, there is no loss of generality in considering
only the case when y
0. In view of x > 0, we need to find the
minimum value of x - y only.
Setting u = x- y , and substituting it into 2 -4y2 = 4 , we obtain
>
3y2 - 2 ~ y
+ (4-2~’)
= 0.
Equation ( * ) with respect to y has real solutions. So we have
A=4~~-12(4-~~)>0.
Thereby
U
>&.
(*>
China Mathematical Competition 2002
4 43 and y
In addition, when x = 3
9
43 , we have u = 43.
=-
3
Therefore , the minimum value of I x I - I y I is
a.
s@@$If the inequality
sin2x+acosx+a2
> 1+ c o s x
holds for any x E R, the range of values for negative a is
Solution a+a2 2 2 whenx= 0. Sou<-2
a
2, we have
<-
(becausea<O). When
a~+acosx>a~+a>2>cos~x+cosx
=I
that is,
+ cosx-
sin2x+acosx+a2
sin2x,
> 1+cosx,
Hence, the range of values for negative a is a
Solution Suppose that (y:
-
<- 2.
4, y1 ) is
c
Mathematical Olympiad in China
10
>
<
>
From A 0, we obtain y 0 or y 4 .
When y = 0 , the coordinates of B are ( - 3 , - 1) and when y = 4,
they are ( 5 , - 3 ) . They both satisfy the conditions given by the
problem. So, the range of values for the y-coordinate of point C is
y
0 or y 4.
<
>
@& As shown in the diagram, there is a sequence of the curves P o ,
PI , P2 ,
It is known that the region enclosed by Po has area 1
and PO is an equilateral triangle. We obtain P&l from Pk by
operating as follows: Trisecting every side of pk, then we
construct an equilateral triangle outwardly on every side of pk
sitting on the middle segment of the side and finally remove this
middle segment (K = 0, 1 , 2,
Write S, as the area of the
region enclosed by P, .
(1) Find a formula for the general term of the sequence of
numbers { S, } ;
(2) Find limS,.
..a.
..a).
?I--
A
Po
p,
p2
Solution (1) We perform the operation on PO. It is easy to see that
each side of Po becomes 4 sides of PI. So the number of sides of PI is
3 4. In the same way, we operate on PI. Each side of PI becomes 4
sides of P2. So the number of sides of P2 is 3 42. Consequently, it is
not difficult to get that the number of sides of P, is3 4".
It is known that the area of Po is So = 1. Comparing PI with Po ,
it is easy to see that we add to PI a smaller equilateral triangle with
1
1
area - on each side of PO. Since POhas 3 sides, so S1 = SO 3 - =
32
32
+
China Mathematical Competition 2002
+-.31
1
11
Again, comparing P2 with PI , we see that P2 has an additional
1 1
- on each side of PI , and
smaller equilateral triangle with area 32 32
PI has 3 4 sides. So that
Similarly, we have
Hence, we have
4k-I
k=l
(9)
4 "]
=1++3
-
k=l
---.
8
3 ($)".
5
5
We will prove ( * ) by mathematical induction as follows:
When n = 1, it is known that ( * ) holds from above.
Suppose, when n = k, we have Sk
8
3 ($)k
5
5
When n = k
1, it is easy to see that, after k
1 times of
operations, by comparing Pkfl with Pk, we have added to Pkfl a
1
on each side of Pk and
smaller equilateral triangle with area
=
---
+
+
32<ktl>
Mathematical Olympiad in China
12
Pk has 3 4' sides. SO we get
By mathematical induction, ( * ) is proved.
(2) From ( l ) , we have S,
= ---
5
5
+ + c ( a , b , c E R,
@&BSuppose a quadratic function f(x) = m2 bx
and a # 0) satisfies the following conditions:
(1) Whenx E R, f ( x - 4 ) = f ( 2 - x ) andf(x) >x.
x+l
(2) Whenx E (0, 21, fh><
(
7
. )
(3) The minimum value of f(x> on R is 0.
Find the maximal wz (m > 1) such that there exists t E R,
f ( x + t ) < x h o l d s so long a s x E [l, ml.
Analysis We will determine the analytic expression for f(x) by the
known conditions first. Then discuss about m and t , and finally
determine the maximal value for m.
Solution Since f(x- 4) = f(2 - x) for x E R, it is known that the
quadratic function f(x) has x =- 1 as its axis of symmetry. By
condition ( 3 ) , we know that f ( x ) opens upward, that is, a > 0.
Hence
f(x) = a ( x + 112( a > 0).
By condition ( l ) , we get f ( l ) > 1 and by ( 2 ) , f ( l > <
( y )=l.2I t f o l l o w s t h a t f ( l ) = l , i . e .
Thereby ,
f(x>= , 1( x + 1 > 2 .
1
~ ( 1 + 1 ) ~ = S1o.u = - .
4
China Mathematical Competition 2003
13
Since the graph of the parabola f ( x > = - 1( ~ + + 1 ) ~
4
+
opens
upward, and a graph of y = fcx t>can be obtained by translating
that of f ( x > by t units. If we want the graph of y = f ( x + t > to lie
under the graph of y = x when x E [1 , m] , and m to be maximal,
then 1 and m should be two roots of an equation with respect to x
-1( x + t + 1 > 2
4
= x.
0
Substituting x = 1 into 0,
we get t = 0 or t =- 4.
Whent = 0, substituting it into 0,
we get x1 = x2 = 1 (in
contradiction with m > 1).
When t =- 4 , substituting it into 0,
we get x1 = 1 , and x2 = 9;
and so m = 9.
Moreover, when t =- 4 , for any x E [1 , 91, we have always
( ~ - 1 ) ( ~ - 9 )< O
1
H - ( ~ - 4 + 1 ) ~ <x,
4
that is
f(X-
4 > < x.
Therefore, the maximum value of m is 9 .
2003
(Shaanxi)
Popularization Committee of CMS and Shaanxj Mathematical Society
were responsible for the assignment of the competition problems in
the first and the second rounds of the contests.
Part I Multiple-choiceQuestions (Questions1 to 6 carry 6 marks each. )
@@$$ A new sequence is obtained from the sequence of the positive
Mathematical Olympiad in China
14
integers { 1, 2 , 3 , . } by deleting all the perfect squares. Then
the 2 003rd term of the new sequence is (
).
(A) 2046
(B) 2047
(C)2048
(D) 2049
[.m]
Solution Since [dTOG]=
= [.SF@]
= [dSFB] = 45,
and 2 003 + 45 = 2 048, Answer: C.
Remark For any positive whole numbers n and m ,satisfying m2 < n
< (m 1>2, we always have n = an--m.
+
@&a
Suppose a , b E R, where ab # 0. Then the graph of the straight
line m - y + b
= 0 and
(A)
the conic section bx2 +ay2
(C)
(B)
).
= ab is (
(D)
Solution In each case, considerbanda, the y-axis intercept and the
slope , of the straight line m - y b = 0. In (A) , we have b > 0 and
a < 0, then the conic section must be a hyperbola, and it is
impossible. Similarly, (C)is also impossible. We have a >0 and b < 0
+
9
in both (B) and (D) , the conic section is a hyperbola. We have - U
2
-b
=
I. Answer: B.
@a Let a line with the inclination angle of 60" be drawn through the
+
focus F of the parabola y2 = 8(x 2). If the two intersection
points of the line and the parabola are A and B, and the
perpendicular bisector of the chord AB intersects the x-axis at the
point P, then the length of the segment PF is (
).
Solution It follows from the property of the focus of a parabola
China Mathematical Competition 2003
15
that F = (0, 0). Then the equation of the straight line through points
A and B will be y = fix. Substitute it into the parabola equation, and
then obtain
3 2 -8~-16
= 0.
Let E be the midpoint of the chord AB , then the x-coordinate of
4
8 IPFI=21FEI=-. 16
Eis-.4 Thenwehave IFEI=X-=-,
3
cos6Oo 3
3
3
Answer: A.
e
% LetXe [
;, 31.
--
y
is (
=
-
tan( x +
Then the maximum value of
$)-tan( x + :)+
cos(x
+):
).
Solution Let z =-X--.
x
6 Then z E
[“
“1
3’Z
6 ’ “1,
4 and2.z E [“
*
We have
Then
y
=
cotz+tanz+cosz
= -+cosz.
sin 22
Since both -and cos z are monotonic decreasing in this case,
sin 22
4
-
6
+-432
=
11
-43.
Answer: C.
6
Mathematical Olympiad in China
16
Suppose x, y E (- 2, 2) and xy =- 1. Then the minimum value
4
9
ofu=>.
+ y i s (
4-2
9-y
8
24
12
12
(A) 7
(€3) fi
(C)7
(D) 7
Solution
I
We have
u=-
4
4-2
when x
(-
2
-
=
1+
352
-9x4 + 3 7 2 -4
35
=1+
Since x E
+-9 29 -21
2 2 +12)
37- ((3x--) X
12
y1 ) U (y1 , 2) , so u reaches the minimum value 5
=&g.
Answer: D.
Solution I[ It is known from the conditions that 4 - 2 > O and9y2 > 0. Then
4
4-2
12
-
1/37-92-4Y2
12
Since u is - when x
5
Answer: D.
=
. :.
9
9-y2
12
1/36 - 9 2 - 4y2
>2/37-21/=12
+( x Y ) ~
12
5'
- -
, so u reaches the minimum.
Suppose in the tetrahedron AB CD, AB
=
1, CD
=
a,the
distance and angle between the lines AB and CD are 2 and
respectively. Then the volume of the tetrahedron equals (
3
).
Solution As in the diagram, from point C draw a line CE such that
China Mathematical Competition 2003
17
it is equal and parallel to AB. Construct a
prism ABF - ECD with ACDE as base and BC
as a lateral edge. Denote V1 as the volume of M
@
the tetrahedron and V2 the volume of the B
----------1
prism, thenV1 = -V2.
C
3
1
Since SAQIE = --cE
OCDsin L E D , and the common
2
perpendicular line IWV of AB and C D is the height of the prism, then
1 IWV C E OCD sin L E D = -.3 So Vl = -V2
1
1
V2 = = -.
2
2
3
2
Answer: B.
Remark If one is familiar with vector calculus, he will find that the
and 3is equal to I
I
volume of the parallelogram formed by
3
1
3
1
3
1
.sin60" = -.2 ThenSAaD = X - = 3 It is easier to find
2
2
4'
the solution using vectors.
'\
s
Part I1 &ort-answer Questions (Questions 7 to 12 carry 6 marks each. )
e
L
Z
s
i The solution set of the inequality I x I
-2
2
-
4 I xI
+ 3 < 0 is
XI
Solution Notice that 1 . T I = 3 is a root of the equation 1
-22 4 I x I 3 = 0. Then the original inequality can be rewritten as ( I x I 3)(1x12+ lxl-l)<O,
that is
+
I
( XI -3)
(I
XI -
2
Since I x I - -'-&>O
2,
(
3,
Suppose points FI
F2
So the solution set is
&@&
then -l+&<
-
~)
&-1
-
2
U
1x2:1<3.
(Jy,
).
3
2
are the foci of the ellipse g
-+L = 3
P is
9
4
Mathematical Olympiad in China
18
a point on the ellipse, and I PF1 I : I PF2 I = 2 : 1. Then the area
of APFlF2 is equal to
Solution I PF1 I I PF2 I = 2a = 6 by definition of an ellipse. Since
IPF11: IPF2I=2:l,thenIPFlI=4andIPF2I=2.Noticethat
+
I FlF2 I = 2c = 2&, and
I PF1 I + I PF2 I
= 42 +22 = 20 =
I FlF2 I '.
1
Then APFl F2 is a right triangle. So S ~ ~ F , =
F, I PF1 I
2
I PF2 I = 4.
@&@ L e t A = {x I 2 - 4 x + 3 < 0 ,
x € R}, B = {x I 21"+a<0,
2 - 2 ( ~ + 7 ) ~ + 5 < 0 , x E R } . I f A C B , thentherangeofreal
number a is
Solution It is easy to see thatA = (1, 3). Now, let
f<x>= 2l"
+ a , g(x)
=
2 -2(a+7)x+5.
Then, when 1 < x < 3 , the images of f(x) and g(x) are both below
the x-axis since A C B. Using the fact that f(x) is monotone
decreasing and g(x) is a quadratic function, we get A C B if and only
i f f ( l > < O , g(l)<O, andg(3)<0. So thesolutionis-4<u<-l1.
3 log,d
-........
2 3 Let a , b , c , d be positive integers and logab = -,
2
a-c=9,
-.
= 5
4
If
thenb-d=
Solution We have b
a $ , d = c? from the assumption. We may
assume that a = 2 , c = y4 with x and y being positive integers, since
a , b, c, dareallpositiveintegers. T h e n a - c = x ' - y 4
= (x-y'),
(x+$) = 9. It follows that (x-y' , x + y 2 > = (1, 9). So we obtain
the solutionx = 5, y = 2. That is, b-d = 2 -y5 = 125-32 = 93.
=
623 8 balls of radius 1 are placed in a cylinder in two layers, with
each layer containing 4 balls. Each ball is in contact with 2 balls
in the same layer, 2 balls in the other layer, one base and the
lateral surface of the cylinder. Then the height of the cylinder is
China Mathematical Competition 2003
19
Solution As in the diagram, letA, B, C, D
be the centers of the 4 balls in the bottom
layer, and A’, B’ , C’ , D’ the centers of the 4 balls
in the upper layer. Then A, B, C, D and A’, B‘,
C’, D’are the 4 vertices of squares of length 2,
D
respectively. Now , the circumscribed circles
with centers 0 and d of the squares constitute the bases of another
cylinder, and the projecting point of A’ on the bottom base is the
middle point M of arc AB.
In AA’AB , we have A’A
= A’B = AB = 2,
then A’N
N is the middle point of AB. Meanwhile, CYM = OA
=&,where
=a,ON
, so
=1
+
Then the height of the original cylinder is& 2.
Remark In order to solve the problem, you must first be clear about
the way the balls are placed (each ball in contact with 4 other balls) ,
then determine the positions of the centers of the balls.
<
S@& Let M, = (0. a l a 2 ~ - a , I ai = 0 or 1, 1 i < n- 1, a, = 1) be a
set of decimal fractions, T, and S, be the number and the sum of
the elements in M, respectively. Then
lim, S n
=
Solution Since a1 , a2 ,
anPl all have exactly two possible
values, so T, = 2”-’. Meanwhile, the frequency of ai = 1 is the same
as that of ai = 0 for 1 i n - 1, and a, = 1. Then
..a,
<<
s, = 21 x 2”-1
-
(h+
-+
1;2
1
...+-)+
1on-I
2”-I X - 1
10”
1
18
Mathematical Olympiad in China
20
so
Part I11 Word Problems (Questions 13 to 15 carry 20 marks each. )
3 X < 5. Prove that 2d-+
d m + d=<
@B Suppose
<
22/19.
Solution By Cauchy's inequality, we have
2diq7+dzT3+dGFG
=d-+d-+dzF3+dGFG
<J[(x+ 1)
+ (x+ 1) + ( 2 ~ -3) + (15
-
=2dGqT< 2 m ,
and the equality holds if and only/x
= 5.
=
But this is impossible. So 24-
--
3d](12
+ l2+ l2+ 12)
=
+-
+-
and
<
22/19.
Remark Some student contestants used the estimate that
2 d G + d m + d l F G
<./[(x+ 1)
+ (2~-33) + (15 - 3 ~ ) ] ( 2 ~+ l2+ 12>
but this does not give the value as required.
@E%
Suppose A , B , C are three non-collinear points corresponding to
complexnumberszo = a i ,
z1
1
=-+bi,
2
z2
= l + c i ( a , bandc
being real numbers) , respectively. Prove that the curve
z=
z o ~ o s 4 t + 2 z 1 ~ o s 2sin2t+z2sin4t
t
( t E R)
shares a single common point with the line bisecting AB and
parallel to AC in M C , and find this point.
China Mathematical Competition 2003
x+yi
= acos4t
i+2(-++i)cos2t*
1
2
sin2t+(l+ci)sin4t.
,-~
Separating real and imaginary parts, we get
x
y
=
cos2t sin2t
+ sin4t
=
sin2t,
= a(l-x)2+2b(l-x)x+m2
A
< x<
0
(0
That is, y
=
1).
(a+c-2b)2?
a
21
+2(b-a)x+
(1)
(O<x<l)
Since A, B, Care non-collinear , a+ c- 2b # 0. So Equation (1) is the
segment of a parabola (see the diagram). Furthermore, the midpoints
of AB and B C are D(
a, F)
and E(
the equation of line DE is y
= ( c -a ) x
9, F),
respectively. So
+ -41 (3a +2b
(2)
- c).
+c
Solving Equations (1) and (2) simultaneously, we get ( a
l 2
1
2b)(x-y)
= 0. Thenx=-,sincea+c--2b#O.
2
line AT have one and only one common point P(
-
Sotheparabolaand
, n+c+2b).
Notice
1
3
- , so point P is on the segment DE and satisfies
2
4
Equation (1) , as required.
1
4
that -
<
-
<
Solution I[ We can solve the problem using the method of complex
numbers directly. Let D, E be the midpoints of AB , CB , respectively.
1
2
Then the complex numbers corresponding to D, E are -( z o
1 a+b.
7
+-1, 2
1
2
-(ZI
+
z2) =
3 b+c.
7
+ -1, 2
+ z1)
respectively. So, complex
number z corresponding to a point on the segment DE satisfies
z=
=
A (:- + " f b 2i ) + ( l - A ) ( + + ~ c i ) , 2
O<A<I.
Mathematical Olympiad in China
22
Substitute the above expression into the equation of the curve
z = zo cos4t
+221 cos2t
sin2t
+ z2sin4t ,
and separate the real and imaginary parts from both sides to give the
following two equations,
/.-I
A
=
sin2tcos2t+sin4t,
Eliminating A from the equations, we get
3
-(a-c>
4
b+ = acos4t + (2b +a
+2
=a(l
=a
-
+ (2b
+ (2b +a
-
7
1
2
collinear , we know that z1 # -(zo
3 A
1
have --=4
2
4
+ (-)2l
2
=
+asin4
- c>
sin9 cos2t
a - c)sin9 cos2t.
(
=
c>sin2tcos2t
2sin2 t cos2t>
Then (2b- a - c> sin9cos2t- 4
9cos2t = sin2t(l-sin2t)
-
=
0. Since A, B , C are non-
+ >.So 2bz2
l,
4 that is,
(sin2t--)
a - c # 0. Then sin
l 2
= 0. Then we
2
1
1,
so A = E [0, 11. That means that
2
2
the curve and the line DE have one and only one common point, and
the complex number corresponding to this common point is
&(l2
4
4
@&3A circle with center 0 and radius R is drawn on a paper, and A is a
given point in the circle with Q4 = a. Fold the paper to make a point
A’ on the circderence coincident with point A, then a crease line is
left on the paper. Find out the set of all points on such crease
lines, when A’ goes through every point on the circumference.
China Mathematical Competition 2003
23
Solution Establish an xy-coodinate system
as in the diagram withA(a, 0) given. Then the
crease line IWV is the perpendicular bisector of
segment AA’ when A’ (Rcos a , Rsin a ) is made
coincident with A by folding the paper. Let
P(x, y) be any point on IWV, then I PA’ I =
I PAI. That is
(x-Rcosa)2 +(y-Rsina)2
xcosx
Then
+ysina
@T7
sin0 = d
where
2
R2 - a2
(x-a)2 +y2.
+2 m
2 R d m
m ’
X
R2 - a2
so
-
=
cos
=
+2 m
R
‘
Y
w‘
w
+-
43?
>
l.
Squaring both sides, we get (2x - a )
R2 -a2
’
R2
So the set we want consists of all of the points on the border of or
2
2
outside the ellipse (2x-aa)
= 1.
R2
R2 -a2
Remark As seen in the diagram, suppose the crease line intersects
OA’at point Q. Then from QA = QA’ we have OQ QA = OA’ = R;
that is, Q is on an ellipse whose foci are 0 and A. And the expression
of the ellipse is
+
(x-;)2
2);(
+
2
=
1.
(+dF7),
+
For any other point P on the crease line IWV, we always have Po
PA =Po PA’ > OA’ = R. So a point of the crease line is either on
+
Mathematical Olympiad in China
24
the border of or outside the ellipse. On the other hand, if from any
point P on the border or outside the ellipse, we draw a line tangent to
the ellipse at point Q , then QO &A = R. Suppose line QO intersects
the circle at point A’. Then QO+QA’ = R , that is, &A =QA’. Then
using the property of the tangent to an ellipse, we have that PQ
bisects LAQA’, and that means the tangent PQ is the crease line as
mentioned above. The arguments above reveal that the set we find
consists of all of the points on every tangent to the ellipse.
+
Popularization Committee of CMS and Hainan Mathematical Society
were responsible for the assignment of the competition problems in
the first and the second rounds of the contests.
Part I Multiple-choice Questions (Questions 1 to 6 carry 6 marks each. )
s@ Let 0 be an acute angle such that the equation 2 4 x 0 s 0
cote=O involving variable x has multiple roots. Then the
measure of 0 in radians is (
).
+
(A)
(B)
5x
x
-
12
12
Solution Since the equation 2
roots, we have
A
x
=
+ 4 x 0 s O+
4~0te(2sin28- 1) = 0.
o < e<
cot 0
16cos2e- 4cot e = 0,
or
BY assumption,
5x
(C)6 or 3
or -
5 , we get
2
1
sin20 = -.
2
(D)
=
+
x
12
0 has multiple
China Mathematical Competition 2004
25
It follows that
20=
-.56x
x
- or20=
6
x
5x
Thus 0 = or 0 = -.
Answer: B.
12
12
@%& Assume t h a t M = {(x, y ) 1x2 +2$ = 3}, a n d N = {(x, y ) I
Y = m+ b ) . If M n N # 0for all m E R, then b takes values
from (
).
Solution For any m E R we have M
2
(0, b) is on or in the ellipsoid 3
n N # 0,which means point
2
+ 2y
3
=
2b2
&
- 3< l , o r - - < b < -2.
1. Therefore
&
2
Answer: A.
@B
The solution set of the inequalityd-+-log+$’
(
1
2
>0 is
>.
(A) [2, 3) (B) (2, 31
(C)[2, 4)
Solution The initial inequality is equivalent to
4 d w - l l o g 23 X + LlOg2X-
Let t =
+2
1
> 0.
d w ,we have
3
1
Jt-,t2+->0,
2
3
2
+-21 > 0,
(D) (2, 41
Mathematical Olympiad in China
26
<
The solution of the above inequalities is 0 t < 1, or 0
1 < 1, which implies that 2 x < 4. Answer: C.
<
<logzx-
+
+
@$@ Let 0 be an interior point of a A B C such that 3 2 3
3 Z =O. Then the ratio of the area of a A B C to the area of
aAocis(
>.
5
3
(A) 2
(€3) 2
(C) 3
(D) j-
Solution In the diagram, let D and E be the midpoints of the sides
AC and B C , respectively. Then we have
S + Z = 2 3 ,
(1)
and
2 ( 3 + Z )
=
4%.
(2)
By equations (1) and (2) we get
S + 2 3 + 3 Z
=2
(3+2%)
= 0.
It follows that 3and % are collinear, and
I+
OD I =2 I +
OE I.
Consequently, SaAEC
~
sapoc
and-
SaABC
-
2
sapoc
&
% Let n
-- 3.
=
3
--
2
e
E
I:
Answer: C.
Zi be a 3-digit number. If we can construct an isosceles
triangle (including equilateral triangle) with a, b and c as the lengths
of the sides. The number of such 3-digit integers n is (
).
(A) 45
(B) 81
(C) 165
(D) 216
Solution If a , b and c are the lengths of the sides of a triangle, all of
9).
them are not zero, it follows that a , b , c E { 1, 2,
(9 If the triangle we construct is equilateral, let nl be the number
of such 3-digit numbers. Since the three digits in such 3-digit number
are equal, we have
..a,
nl =
= 9.
China Mathematical Competition 2004
27
(ii) If the triangle we construct is isosceles but not equilateral, let
n2 be the number of such 3-digit numbers. Since there are only 2
different digits in such a 3-digit number, denote them by a and b. Note
that the equal sides and the base of an isosceles triangle can be
replaced by each other, thus the number of such pairs (a, b) is 2G.
But if the bigger number , say a , is the length of the base, then a must
satisfy the condition b < a < 2b. All pairs that do not satisfy this
condition we list in the following table. There are 20 pairs.
a
b
9
8
7
6
5
4
3
2
4, 3 ,
4, 3 ,
2, 1
2, 1
3 , 2,
1
3 , 2,
1
2, 1
2, 1
1
1
1
On the other hand, there are Cg possible 3-digit numbers with
digits taken from a given pair (a, b). Thus
n2 =
(2G - 20)
Consequently, n = nl
=6(G
+n2 =
-
10)
=
156.
165. Answer: C.
The vertical cross-section of a circular cone with vertex P is an
isosceles right-angled triangle. Point A is on the circumference of
the base circle, point B is interior to the base circle, 0 is the
center of the base circle, AB IOB and intersecting at B, OH I
PB and intersecting at H, PA = 4, and Cis the midpoint of PA.
When the tetrahedron 0 - HPC has the maximum volume, the
length of OB is (
>.
Solution Since AB IOB, andAB IOP, we
have AB IPB , and PAB IFOB. Moreover,
from OH IPB we obtain that OH IHC and
OH I PA. Since C is the midpoint of PA,
O C I P A . Thus, PC is the altitude of the
A
Mathematical Olympiad in China
28
tetrahedron 0 - HPC and PC = 2.
In RtAOHC, OC = 2. Therefore when HO = HC , SABC reaches
its maximum, that is, V o - ~ p =
c V p - ~ reaches
a
its maximum. In this
case, HO
=a,and HO
2&
OP *tan3O0= -.
3
=
1
2
-0P.
Hence, L H P O
=
30°, and OB
=
Answer: D.
Part I1 Short-answer Questions (Questions7 to 12 carry 6 marks each. )
@&&
In a planar rectangular coordinate system d y , the area enclosed
by the graph of function f(x> = asinax cosax (a > 0) defined
on an interval with the least positive period and by the graph of
+
function g(x> =d
m is
Solution w e rewrite function f<x>as f < x >=d m s i n ( a x
+ pi>,
2x
where p = arctan -.1 Its least positive period is , and its amplitude is
a
a
d m .
By symmetry of the figure enclosed by the graphs of the
functions f(x> and g(x> , we can change the figure into a rectangle
with length ?? and w i d t h d m using the cut-and-paste method.
a
Therefore its area is @
a
dW.
&
% Let f: R+R be a function such that f(O>
y
ER, f(xy+l)
= f(x>f(y>-f(y>-x+2
Solution Since for any X , y E R, f(xy
2, we have
x
+
f(yx
+ 1)
= f(y>f(x>
-
=
1 and for any x,
holds. Thenf(x>
+ 1)
= f(x>f(y>
f<x>- y
-
f(y)
+2.
Thus,
f(x>f(y> - f(y)
that is,
-
x
+2
= f(y>f(x>
-
f<x>- y
+2 ,
=
-
China Mathematical Competition 2004
+
f<x> y
Put y
= 0,
=f
29
( Y ) +x.
we obtain f(x> = x+ 1.
In the diagram, ABCD -A1 B1 CI D1 is a cube. The dihedral angle
A - BD1 - Al in degrees is
Solution Draw line segments DIC, and
CE such that CE 1BD1 , Eis the foot of the
perpendicular. Let the extended lines CE
A,
andAIB intersect at F. Draw line segment
AE. By symmetry we have AE 1 BDI.
Therefore L E A is the plane angle of the
dihedral angle A - BD1 - Al. Draw a line A
segment connecting points A and C, and set
AB
=
1, then AC
=
cosLAEC
AD1
=
=
a,BD1 a.In RtAABDl, AE
=
AE2 +CE2 -AC2
2AE.CE
-
=
2AE2 -AC2
2AE2
3
Thus LAEC = 120". But L E A is the supplementary angle of LAEC,
h e n c e L E A = 60".
KES
Let p be an odd prime. Let K be a positive integer such that
d
m is also a positive integer.
Then K
=
Solution S e t d w = n , n E N . ThusK2-pK-n2
P
t
W
, which implies that p2
2
m 2 , wherem E N. So (m-2n>(m+2n>
=O,andK=
+4n2 is a perfect square, says
= p2.
Mathematical Olympiad in China
30
3 , we have
Since p is a prime and p
m-2n
=
m+2n
= p2.
{
1,
Solve the equations above and we get
m=-,
n=-
P2 +1
2
p2-1
4 .
(the negative value is omitted).
-.....-.. Let q , al, a2,
e-i$-s
(6+a,)
%+I)
, a,,
be a sequence of numbers satisfying (3 -
= 18, and% = 3.
Then
c
" 1
- equals
;=o ai
18, namely,
is a geometric progression with common ratio 2. Thus
b,+- 1 = 2 ~ ( b o + - ) =12 " ( - + - ) =1- X 2 "1+ ' ,
3
3
a0
3
b,
Therefore
=
1
-(2"f1
3
-
1).
1
3
China Mathematical Competition 2004
-
1
-(2”f2
3
31
-n-3).
%@% Let M(- 1, 2) and N(1, 4) be two points in a plane rectangular
coordinate system xOy. P is a moving point on the x-axis. When
L M P N takes its maximum value, the x-coordinate of point P is
Solution The center of a circle passing through pointsMandNis on
the perpendicular bisector y = 3 - x of M N . Denote the center by
S ( a , 3 - a ) , then the equation of the circle S is
(x---.)2+(y-3+.)2
= 2(1+a2).
Since for a chord with a fixed length, the angle at the
circumference subtended by the corresponding arc will become larger
as the radius of the circle becomes smaller. When L W N reaches its
maximum value, the circle S through the three points M, N and P will
be tangent to the x-axis at P, which means the value a in the equation
.
the above
of S has to satisfy the condition2(1+a2) = ( ~ - 3 ) ~Solve
equation we have a = 1 or a = - 7. Thus the points of contact are
P(l , 0) and P’(- 7, 0) respectively.
But the radius of the circle through points M, N, and P’ is larger
than that of the circle through points M, N and P. Therefore
L M P N > L M P ’ N . Thus P(1, 0) is the point we want to find, and
the x-axis of point P is 1.
Part I11 Word Problems (Questions 13 to 15 carry 20 marks each. )
@ $ The rule of an “obstacle course” specifies that at the nth obstacle
a person has to toss a die n times. If the sum of points in these n
tosses is bigger than 2” , the person is said to have crossed the
obstacle.
(1) At most how many obstacles can a person cross?
(2) What is the probability that a person crosses the first three
obstacles?
32
Mathematical Olympiad in China
(Note: A die is a fair regular cube, on its six faces there are
numbers 1, 2 , 3 , 4 , 5 , 6 respectively. Toss a die, the point is the
number appearing on its top face after it stops moving. )
Solution Since the die is fair, the probability of any of the six
numbers appearing is the same.
(1) Since the highest point of a die is 6, and 6 X 4 > 24 , 6 X
5 <25 , it is impossible that the sum of points appearing in n tosses is
bigger than 2” if n 5. This means it is an impossible event, and the
probability of crossing the obstacle is 0.
Therefore at most 4 obstacles that a person can cross.
(2) We denote A, the event “at the nth obstacle the person fails to
cross” , the complementary event A, is “at the nth obstacle the person
crosses successfully”.
At the nth obstacle of this game the number of all possible
outcomes is 6”.
The first obstacle: event Al contains 2 possible outcomes (i. e. ,
the outcomes in which the number appearing is 1 or 2 ) . So the
probability of crossing the obstacle is
P(&)
=
1-
The second obstacle: the number of outcomes contained in event
A2 is the total number of positive integer solution sets of the equation
x+ y = a where a is taken to be 2, 3 and 4 respectively. Thus the
C$ = 1 2 3 = 6, and the
number of outcomes equals Ci
probability of crossing the obstacle is
+ +
-
P(A2)
=
1- P(A2)
=
+ +
6
5
1-=62
6’
The third obstacle: the number of outcomes contained in event
A3 is the total number of positive integer solution sets of the equation
x y z = a where a is taken to be 3, 4, 5, 6, 7 and 8 respectively.
Thus the number of outcomes equals
+ +
China Mathematical Competition 2004
a+C3+C$+@+Cg+C'$
33
=1+3+6+10+15+21=56,
and the probability of crossing the obstacle is
P(&)
=
1- P(A3)
=
20
56 = 1- tj3
27'
Consequently, the probability that a person crosses the first three
obstacles is
P(&)
x P ( Z ) x P(&)
2
3
=-
5 20
100
xx - = 243.
6 27
(We can also list all the possible outcomes at the second obstacle and
at the third obstacle. )
Remark Problems concerning probability theory first appeared in
the National High School Mathematics Competition. Problems are not
too difficult. Topics such as derivative and its applications have
already appeared in high school textbooks. These topics will also
appear in mathematics competitions.
@$9 In a plane rectangular coordinate system xOy there are three
pointsA(0,
4
3)'
B(-
1, 0) and C(1, 0). The distance from
point P to line BC is the geometric mean of the distances from
this point to lines AB and AC.
(1) Find the locus equation of point P.
(2) If lineL passes through the incenter (say, D ) of M C , and
has exactly 3 common points with the locus of point P.
Determine all values of the slope K of line L.
4
Solution (1) The equations of lines AB, AC and BC are y = -(x+
3
4
1) , y =--(x-l)
3
andy = 0 respectively. The distances from point P
to AB , AC and BC are respectively
Mathematical Olympiad in China
34
d2
1
I 4~+3y- 4
5
=-
According to the assumption, dl d2
25y2. That is
1 6 2 - ( 3 ~ - 4 ) +25$
~
= 0,
= d?
I , d3
=
I
y
1.
, we have I 16x2-
( 3 ~ -4>2
or 1 6 2 - ( 3 ~ - 4 4 ) -25y2
~
= 0.
By simplifying the above equations, we obtain that the locus
equations of point P consist of
circle S : 2 2 +2y2 +3y-2
=0
and
hyperbola T: 8 2 - 17y2
+ 12y
-
8 = 0.
(2) According to (1) , the locus of point P consists of two parts
circle S : 2 2 +2$
+3y-2
= 0,
0
and
hyperbola T: 8 2 - 17y2
+ 12y
-
8 = 0.
0
Since B(- 1, 0) and C(l , 0) are points satisfying the assumption,
points B and C are on the locus of point P, and the common points of
the curves S and T a r e points B and C only.
The incenter of a A B C is also a point satisfying the assumption.
In view of dl
= d2 = d3
, solving the equations we have D( 0 , -)21
.
Line L passes though D, and has three common points with the
locus of point P. So the slope of L is defined. Suppose that the
equation of L is
1
y=kx+--.
2
i ) If k
0
0, then L is tangent to the circle S, which means
there is a unique common point D. In this case line L is parallel to the
x-axis, which implies that L and hyperbola T have two other common
points different from point D. Hence, there are just three common
(
=
China Mathematical Competition 2004
35
points for L and the locus of point P.
( ii ) If K # 0 , there are two different intersection points for L and
the circle S. In order that L and the locus of point P have 3 common
points, we must have one of the following two cases.
Case 1: Line L passes through point B or point C, which means
that the slope of L is K
=+’ 2 , and the equation of L is x =+(2y-
1).
Substitute it into equation 0we get
y(3y-44)
= 0.
-)
)
5 4
Solving it we have E( or F(- , which means that
3’ 3
3 ’ 3
line BD and curve T have 2 intersection points B and E, and line CD
and curve T have 2 intersection points C and F.
Consequently, if K
=+ 2 for L and the locus of point P there are
exactly 3 common points.
Case 2: Line L does not pass through point B and point C
(i. e. K #+
+).
Since for L and S there are two different intersection
points, there exists a unique common point for L and hyperbola T.
Thus for the following system of equations
there is one and only one real solution. After eliminating y and
simplifying we have
25
=O.
4
(8-l7K2)2-5Kx--
The above equation has a unique real solution if and only if
8-17K2
or
=
0
0
Mathematical Olympiad in China
36
(- 5K)2
+4(8
Solving equation @
we get
IK
we obtain K
-
17K2) 25 = 0.
4
=+
0
=.
17
And solving equation 0
=+.Jz
-.2
Consequently, the set of all possible values of the slope K of line L
is the following finite set
and p are different real roots of the equation
4 2 - 4tx - 1 = 0 ( t E R). [ a , p] is the domain of the function
@@ Suppose that
f < x >=
a
2x- t
x2 1'
~
+
(1) Find g(t> = maxf(x> - minf(x).
(2)ProvethatforuiE (0, ; ) ( i = l ,
sinu3
=
2, 3 ) , i f s i n u l + s i n u 2 +
1, then
2XIX2 - t(X1
+x2)
1
2
--
But
f(X2)-f(X1>
=
2x2 - t
2x1 - t
x;+1
x?+l
< 0.
China Mathematical Competition 2004
f(x2)-
37
f(xd > 0.
Consequently, f(x) is an increasing function on the interval
[a,
la.
Since a +/3
=
( 2 ) g(tanui)
1
4
t and a/3 =- - ,
l6 +24cosui
-(++3)
8
cosui cos ui
=
-
lfi) + 9
cos ui
16 9c0s2ui
+
so
=
Since k s i n u ;
=
-( 1
16&
16 X 3 + 9 X 3 - 9 2 sin2u;).
1, andui E (0,
i= 1
;),
i=l
i= 1
i= 1
i = 1, 2 , 3 , we obtain
Mathematical Olympiad in China
38
Thus
Remark Part (1) of this problem is well-known, we put in an
inequality to increase the level of difficulty.
2005
(Jiangxi)
Popularization Committee of CMS and Jiangxi Mathematical Society
were responsible for the assignment of the competition problems in
the first and the second rounds of the contests.
Part I Multiple-choice Questions (Questions1 to 6 carry 6 marks each. )
@@& Let K be a real number such that the i n e q u a l i t y d a + d z >
K has a solution. The maximum value of K is (
).
(A)&-&
(C)&+&
(B)&
Solution Set
Then
y
= d a + d z ,
3
(D) &
< x < 6.
9= (~-33)+(6-~)+2J(~-33)(6-~)
<2[(~-3)+
So
0 <y
<&, and the maximum value of K is&.
-CS A, B, C, D
=a
( 6 - ~ ) ] = 6.
Answer: D.
are four points in the space and satisfy
IXl-7, 1
3
1= 11 and
(
) values.
=
9. Then
IzI
1
sI
=
3,
has
China Mathematical Competition 2005
(A) only 1
(C)four
Solution Note that
D A 2 . Since
39
(B) two
(D) infinitely many
= 32 +112 = 130 = 72 +92
s2
+s2
=
Z2
+
s+z+s+s=3,
DA2
=s2
(s+Z+s)2
=AB2 + BC2 + a2
+ 2 ( B Z+Z s+S a)
=
=AB2 -BC2
+a2
+2(BC2
+AB *BC+BC*CE+CE AB)
(Z+S)
ISI
=21AB+ZI
IZ+SI
=AB2 - BC2 +a2
+2(AB+Z)
i.e. 2 1 x 1
9
=AD2+BC2-AB2-a2=OS
Thus
I x I IS I
has only one value 0. Answer: A.
@@
MB C is inscribed in a unit circle. The three bisectors of the anglesA,
B and Care extended to intersect the circle at A1 , 4 and C, respectively.
A
A A 1 cos Then the value of
(A) 2
Solution
Then
1
-
-
(C)6
(B) 4
+ $)
= 2sin(
A+B+C
B
>.
(D) 8
Join 841 as shown in the diagram.
A 4 1 = 2sin( B
= 2cos(
+BB cos B + CC1 cos C
+ sin B +2 sin C 2 i s (
2
sinA
A
C
+2-2)
B C
2
2).
-
A,
x
= cos( - - C)+
2
x
cos( - - B ) = sin C+ sin B.
2
Mathematical Olympiad in China
40
Similarly,
B = sinA
BB1cos 2
+ sin C, CC1 cos -C2 = sinA + sin B.
Therefore
+
+ CC1cos -C2 2 (sinA + sin B + sin C) ,
(sinA + sin B + sin C) 2.
and the original expression 2
sinA + sin B + sin C
A BB cos B
AA cos 2
=
2
=
=
Answer: A.
As shown in the diagram, ABCD -A’B’C’D’ is a cube. Construct
an arbitrary plane a perpendicular to the diagonal AC ’ such that a
has common points with each face of the cube. Let S and L
denote the area and the perimeter of the cross-section of a
respectively. Then (
).
(A) S is a fixed number and L is not
(B) S is not fixed and L is fixed
(C) Both S and L are fixed
\
I
\
\
I
\/
”
D‘&’-,. >+- ->
I
‘\
c
(D) Neither S nor L is fixed
,.
Solution After cutting off two regular
A
B
pyramidsA -A’BD and C’ - D ’B’C, we get a
geometric solid V with two parallel planes A’BD and D ’B’C as its
upper and lower bases. Each lateral face is an isosceles right triangle
and each side of the cross-section (denoted by W) is parallel to a side
of the bases of V respectively. Cut the lateral face of V along the edge
A’B’ and stretch it on a plane, we get DA’B’B1 A1 and the perimeter
of W is stretched into a line segment (E’E1 in the figure) which is
parallel to A’Al . Clearly, E’E1 = A’Al . Thus L is a fixed value.
When E’ is the midpoint of A’B’, W is a regular 6-gon. But when
I
/’
E’ is moved to A’, W is an
equilateral triangle. It is easy
to see that the areas of a
B‘
C
\/
A’
B
D’
/\
B,
/\
D
A,
China Mathematical Competition 2005
41
regular 6-gon and an equilateral triangle with the same perimeter L are
&L2 and 43
-L2 respectively. Thus S is not fixed. Answer: B.
24
36
@@ The curve represented by the equation
2
cos 42 - cos 43
=
+
2
sinfi-
sin&
>.
1is (
(A> An ellipse with the foci on the x-axes
(B) A hyperbola with the foci on the x-axes
(C>An ellipse with the foci on the y-axes
(D) A hyperbola with the foci on the y-axes
Solution Sincefi+&>x,
cos(
~ o O < ~ - f i < & - ~ < ~and
2
2
2
5 -&)> cos(43- 5),i. e. sin& > sin&.
x
x
y<&<x,
SinceO<fi<y,
cos 42 - cos 43
> 0.
socosfi>O,
cos&<O,
and
Thus the curve represented by the equation is an
ellipse.
Since
(sinfi-
sin&)-
(cosfi-
cos&)
(*>
and
we get
-
x
42-43
,<,-<o,
L
L
a+&
3x
2
x
y<-
sin fi-&
-< 0 ,
2
3x
-<4
2
<
,,
Mathematical Olympiad in China
42
so the expression ( * ) is less than 0.
That is sinfi- s i n f i < c o d - c o s f i , therefore the curve is an
ellipse with foci on the y-axies. Answer: C.
(0, 1 , 2 , 3 , 4 , 5 , 6 ) a n d M =
@LetT=
ai ET, i
=
(-+-+-+-;
a1
a2
a3
a4
7
72
73
74
1, 2, 3 , 41. Arrange the numbers in M in the
descending order. Then the 2 005-th number is (
).
5
5
6
3
5
5
6
2
(A)?+,+,+,
(my+,+,+,
7
7
7
7
7
7
1
1
0
3
1
1
0
4
(c)-+-+T+Q
(D)-+-+T+Q
7 7 2 7 7
7 7 2 7 7
k]p
a number base p with K digits.
Solution Let [ a l a 2 ~ ~ ~ abe
Multiply each number in M by 74 , and we get
IVf = (
i = 1, 2, 3 , 4)
~ 1 +U272
7 ~ + a 3 7 + ~ 4 ; ai E T,
=
{ [ u I u ~ u ~ u ~I ]ai; E T, i
=
1, 2, 3, 4).
The maximum number inM' is [6 66617 = [2 400110.
In the decimal system, starting from 2 400 in the descending
order, the 2 005-th number is 2 400 - 2 004 = 396. But [396]10 =
[l 10417. Divide this number by 74 , we get a number in M, that is
1
1
0
4
Answer: C.
7 7
73 74
-+,+-+-.
Part I1 Short-answer Questions (Questions7 to 12 carry 6 marks each. )
@$@ Express the polynomial in x f(x) = 1 - x 2 - 2
XI9 +go
into a polynomial in y g (y) = a0 a1y a2y2
a19yI9 a20y20, where y = x - 4. Then a0 a1
a20 =
+
+
+ +
+
+ +
+ + +
Solution The terms in the expression f(x> form a geometric series
with first term 1 and common ratio - x . By the summation formula of
geometric series,
China Mathematical Competition 2005
f<x>=
(-d2I
-1
-x-1
-
2l+1
x+l *
Set x = y + 4 , g(y> = ( Y + ~ ) +'.
~ ~ Lety
Y+5
a0 +a1
@&@
Let f<x) be
+ ...
+a20 =
43
g(1)
=
=
I , we get
521+1
6 '
~
a decreasing function defined on (0, +m). If
then therangeof a is
f(2a2+a+1)<f(3a2-44a+1),
Solution Since f<x>is defined on (0,
~
+m),
from
+x7 > o ,
2(a+iC>
l 2
12a2+.+1=
I
( 3 a 2 - 4 ~ + 1 = (3a-l)(u-1)>0,
we get
a
> 1or a < -.31
(1)
Since f<x>is a decreasing function on (0,
2a2 + a + 1 > 3a2 - 4a+ 1*a2
+
-
00)
, so
5a < 0.
1 or
Thus 0 < a < 5. Combining this with (1) , we have 0 < a < 3
1<a<5.
@@ Assumethata,p, Y s a t i s f y O < a < P < ~ < 2 x . If
co&
+ + c 0 4 x +p> + c 0 4 x + Y )
a>
for arbitrary x E R, then Y - a =
Solution Write f<x>= cos(x+a> +cos(x++P)
f ( x ) = O f o r x € R,
f < - a ) = 0 , f<-Y> =Oandf(-p)
=0
+COS(X+Y).
=O.
Since
Mathematical Olympiad in China
44
cos(P-
That is
cos(a-P>
cos(a-
and
cos(P-
so
+
=-
1,
+cos(~-P)
=-
1,
+ cos(P-
=-
1,
a)
Y)
- a>
COS(Y
P> = C O S ( Y -
a) = C O S ( Y -
~ i n c e 0 < a < p < ~ < 2 x , sop--,
view of P- a < Y 2x
y-P=-,soy-a=-.
3
a,
Y-P<
4x
3
Y-
a,
On the other hand, whenP-a=
Y = a+
-.4x3
Y)
a> =-
Y-P,
-.21
7-aE
(-2x3 ' 4x3 }. In
it is possible only when P-
2x
-,
3
y-P=
For arbitrary x E R, we denote x
we havep=
a=
2x
3
a+-,
+ a = 8. Since three
are the vertices of an equilateral triangle on the unit
circle 2
+y2
=
1 with center at the origin, it is obvious that
cose+cos (e+and that is
c04x
3
+cos
(e+- '3") = o ,
+ + c 0 4 x +P> + cash + Y )
a>
= 0.
1
@$B As shown in the diagram, the volume of tetrahedron DABC is -.
6
AC
Also, L A C B = 45" , and AD
B C - = 3. Then CD =
+
+
42
D
Solution
Since
sin4501 >VDUC
1
, so AD *BC
6
=-
42
>I.
B
China Mathematical Competition 2005
Notethat3 = A D + B C + -
45
2 3J-2
AD*BC*-
42
equality holds if and only if AD = BC
AC
= - = 1.
3. The
It follows that AD is
./z
perpendicular to the face ABC. So DC =dAD2+AC2
=
a.
@3
If one side of square AB CD is on the line y = 2 x - 17, and the
other two vertices lie on parabola y = 2.Then the minimum
area of the square is
Solution Assume that AB is on the line y = 2 x - 17 and the
coordinates of the other two vertices on the parabola are C(xl , y l )
and D ( x 2 , y2). Then CD is on a line L whose equation is y = 2 x b.
+
Combining this with the equation of the parabola, we get 2
*
b
X I ,2 = 1& d
square is a. Then
m .
=
2x+
Assume that the length of one side of the
=5(Xl
= 20(b+
-
1).
(1)
Pick a point (6, - 5 ) on the line y = 2 x - 17, and the distance
from the point to the line y = 2 x b is a.
+
so
(2)
From (1) and ( 2 ) , we get bl
1280, and aLn = 80.
=
3, b2
=
63, so a2
=
80 or a2
=
@@iBA natural number a is called a “lucky number” if the sum of its
digital is 7. Arrange all “lucky numbers” in an ascending order,
If a, = 2 005, then a5, =
and we get a sequence a1 , a2 ,
..a.
Solution Since the number of non-negative integer solutions of
equation X I
x2
X k = m is G + k - l ,
the number of integer
+ + +
Mathematical Olympiad in China
46
solutions, whenxl > l a n d x i > O ( i > 2 ) ,
isC;&.
L e t m = 7 , the
number of lucky numbers with K digits is p(K) = C2+5.
Since 2 005 is the minimum lucky number of the type 2abc and
p(l)
= 1, p(2) = C$ = 7, p(3) =
= 28. Note that the
number of four digits lucky numbers of the type labc is the number of
non-negative integer solutions of a+b+c = 6, i. e.
= 28. Thus
1 7 28 28 1 = 65 and 2 005 is the 65-th lucky number, i. e.
a65 =2 005, SO n = 65, 5n = 325.
~
=a
a
~
a++,
++ + +
cp(K)
5
Furthermore p(4)
= C$ = 84,
p(5)
=C
~=
O 210 and
=
k= 1
330.
Therefore the last six lucky numbers with 5 digits, from the
largest to the smallest, are 70 000, 61 000, 60 100, 60 010, 60 001,
52 000. So The 325-th lucky number is 52 000, i. e. a5, = 52 000.
Part I11 Word Problems (Questions 13 to 15 carry 20 marks each. )
s@.&&Given a sequence {a,} of numbers satisfying a0 = 1, an+l =
7a,
+ d W, n E N .
2
Prove that
(1) for each n E N, a, is a positive integer.
(2) for each n E N, anan+l- 1 is a perfect square.
Proof (1) By assumption, al = 5 and {a, } is strictly increasing with
Square both sides, and we get
7a,a,+l
ai+l
-
a,2
7U,-l a,
-
+
+ +9
Ui-1
+9
= 09
=
0,
0 0: an+l = 7a, U,-l.
It follows from a. = 1, al = 5 and 0that a,
-
for each n E N.
-
0
0
0
is a positive integer
China Mathematical Competition 2005
(2) From
0,
we get (a,+l
+a,>2 =9(U,U,+l
-
47
1>,
so
By (1) , a, , an+l are positive integers and therefore %+I +a, is a
3
rational number. Since
integer, so is %+I +a,
3
(%+I
+any
-
3
. Thus an+lan
-
&5$
- Nine balls, numbered 1, 2,
U,+~U,
-
1 is a positive
1 is the square of an integer.
, 9, are put randomly at 9 equally
spaced points on a circle, each point with a ball. Let S be the sum
of the absolute values of the differences of the numbers of all two
neighboring balls. Find the probability of S to be the minimum
value. (Remark: If one arrangement of the balls is congruent to
another after a rotation or a reflection, the two arrangements are
regarded as the same).
Solution 9 balls with different numbers are placed at 9 equally
spaced points on a circle, one point for one ball. This is equivalent to
a circular arrangement of 9 distinct elements on a circle. Thus there
are 8! arrangements. Considering the reflections, there are
81
2
essentially different arrangements.
Next, we calculate the number of arrangements, which make S
the minimum. Along the circle there are two routes from 1 to 9, the
major arc and the minor arc. For each of them, let XI , x 2 , , Xk be
the numbers of the successive balls on the arc, then
I1--1I+
>I
=
(1 - X I >
I1
-
I..1-~2I+-.+I~k-~I
+
(XI - ~ 2 2 >
+ +
h k
-9)
I
9 I = 8.
The equality occurs if and only if 1<XI <x 2 <
<xk <9, i. e.
Mathematical Olympiad in China
48
the numbers of the balls on each route is increasing from 1 to 9.
Therefore, S- = 2 8 = 16.
From the above analysis, when the numbers of the balls
{ 1, x1 , x2, , Xk , 9 } on each arc are fixed, the arrangement which
gets the minimum value is uniquely determined. Divide the set of 7
balls ( 2 , 3,
8 ) into two subsets, then the subset which contains
+G = 26 cases. Each case corresponds
less elements has +G
to a unique arrangement, which achieves the minimum value of S.
Thus, the number of the arrangements when S takes the minimun
26
1
value is 26 and the corresponding probability is p = - =
8!
315'
2
..a,
a
+G
~
@@ Draw a tangent line of parabola y = 2 at the point A(1, 1).
Suppose the line intersects the x-axis and y-axis at D and B
respectively. Let point C be on the parabola and point E on AC
AE
BF
such that - = A,. Let point F be on BC such that - = A2 and
EC
FC
A,
A2 = 1. Assume that CD intersects EF at point P. When point
Cmoves along the parabola, find the equation of the trail of P.
Solution I The slope of the tangent line passing through A is y' =
2x IZ=l = 2. So the equation of the tangent line AB is y = 2x - 1.
+
Hence The coordinates of B and D are B(O , -1) , D
(y1 , 0). Thus D is
the midpoint of line segment AB.
Consider P(x, y) , C(x0 , 4)
, E(x1 , y1) , F(x2 , y2 >. Then by
AE = A,, we know x1 = l + A I X O , y1 - 1+Al4 . From BE
= A,,
1+ A 1
1+ A 1
FC
we get x 2
=
~
A2 xo
1+A2
-1+A24
'y2=
1+A,
*
Therefore the equation of line EF is
l+AI-d
y-
1+A,
x-
1+AIXO
1+A,
China Mathematical Competition 2005
49
Simplifying it, we get
[(-A2
h o - (1
- -A1
+A2)ly
--Add - 3 ] x + l + x o
=[(-A,
--A2xo.2
(1)
1
When xo # y , the equation of line CD is
24x-4
2x0-1
y=
x = - xo+1
3
From (1) and (2) , we get
(2)
*
’
XO
y=-.
3
Eliminating xo, we get the equation of the trail of point P as y
1
-(3x3
1)2.
When xo
x+
3
2
=
--
1
4
--A2
3 = ( 4-A2
1 - --Al1 - 3)
1 , the equation of EF is - -y
2
2
4
1
, the equation of CD is x = -.2 Combining them, we
=
-
conclude that (x, y >=
(i,&,)
is on the trail of P. Since C and A
2
cannot be congruent, x0 # 1, x # 3.
1
2
- 1)2,x # ?.
3
, the equation of AB is y = 2~ - 1,
Therefore the equation of the trail is y
Solution I[
From Solution
1
B(0, - l ) , D ( y ,
I
= -(3x
0 ) . ThusDis themidpointofAB.
CD
CA
CB
SetY=-,
tl=--=l+Al,
t2=-=1+A2.
Thentl+t2 =3.
CP
CE
CF
Since AD is a median of M C , S a c =
~ 2 S p - c ~ ~= 2saCBD where
Sa denotes the area of A. But
Mathematical Olympiad in China
50
3
so Y = - and P is the center of gravity for AABC.
2
Consider P(x, y > and C(x0 , 4).
Since C is different from A,
xo # 1. Thus the coordinates of the center of gravity P are x
O+l+XO
-
3
we get y
1+xo
3
1
3
= -(3x-
1 > 2 , X#?.
2
9
2
-1+1+4
x#3,y=
3
=
4 Eliminating xo ,
=-
3'
1>2. Thus the equation of the trail is y
1
3
= -(3x-
China Mathematical
Competition (Extra Test)
a@As shown in the diagram, in M
C , L A = 60°, AB >AC, point
0 is a circumcenter and H is the intersection point of two
altitudes BE and CF. Points M and N are on the line segments BH
and HF respectively, and satisfy BM = CN. Determine the value
MH+NH
of
'
OH
Solution We take BK = CH on BE and
join OB , OC and O K .
From the property of the circumcenter
of a triangle, we know that L B O C =
B
c
2 L A =120°. From the property of the
/ /
Mathematical Olympiad in China
52
orthocenter of a triangle, we get L B H C = 180" - L A = 120". So
L B O C = L B H C . Then four points B, C, H and 0 are concyclic.
Hence LOBH = L O C H .
In addition, OB = OC and BK = CH. Therefore, A B O K Z
ACOH. It follows that L B O K = LCOH , and OK = OH.
so ,
LKOH
=LBoC =
LOKH
= L O H K = 30".
120",
In A O K H , by the sine rule, we get KH = a 0 H . In view of
EM =CN and BK = C H , we get KM = N H , and
M + N H = M H + K M = KH = a O H .
Therefore ,
=a.
M+NH
OH
@$@There are real numbers a , b and c and a positive number A such
that f (x)= 2 ax2 b x c has three real roots XI, x2 and x3
satisfying
+ + +
(1) x2
-XI
(2) x3
> +I1
=
A,
+x2).
Find the maximum value of
Solution Let S =
2a3
+27c
2a3
+27c
-
9ab
A3
-
A3
S=
9ab
, then
-
A3
A3
) (- 71a
-
- x2
) (- -a31
-
x3
)
China Mathematical Competition (Extra Test) 2002
53
a .
Writeu~=x~+-(z=1,2,3),thenu2-ul=x2-xI=A,u3>
3
1
-(UI
2
u3
+u2),
andul +u2 +u3
= XI + x 2
+x3 +a
= 0.
So, ul,u2 and
satisfy the corresponding conditions too, and
-
S=
27u1 U2 U3
(u2 - u1)
By u3 =-(uI
+uZ)
>-u1 +u2
2
3’
we get u1 + U Z <O.
Consequently,
at least one of u1 and u2 should be less than 0. We may assume u1 <0.
I f u 2 < 0 , thenS<O.
If u2 > 0, we suppose further
u2
- u1
Vl=-,V=%O2-
u2 -u1
Then V I
u3
+
ZQ
> 0.
=
1, V I and
ZQ
u2
- u1
are both greater than 0 and
V I - ZQ =
So it follows that
u2 -u1
s = 2774 % (Vl
- %)
= 27V(l-
2V)
V)(l-
<27/=
=
3
-a.
2
The equality holds when v
.
.
=
2
1 J 3 = OandA=
equationis2--x+2
18
.
The corresponding cubic
1 x1 =--
2
(1+-
=
Mathematical Olympiad in China
54
L(l-$)
2
andx3 =
3
Consequently, the maximal value is 2
a.
@BBefore
The World Cup tournament, the football coach of F
country will let seven players, Al , A2 , , A7 , join three
training matches (90 minutes each) in order to assess them.
Suppose, at any moment during a match, one and only one of
them enters the field, and the total time (which is measured in
minutes) on the field for each one of A1 , A2 , A3 and & is
divisible by 7 and the total time for each of A5 , & and A7 is
divisible by 13. If there is no restriction about the number of
times of substitution of players during each match, then how
many possible cases are there within the total time for every
player on the field?
Solution Suppose that xi ( i = 1, 2,
7) minutes is the time for
i-th player on the field. Now, the problem is to find the number of
solution groups of positive integers for the following equation:
..a,
XI +XZ
whentheconditions7
are satisfied .
Suppose XI x 2
I
+.**+Q= 270
(1)
x i ( i = l , 2, 3, 4) and13
+ + +
x3
x4 =
7m
7m and x 5
+ 13n
I
+ +
X6
xj(Cj=5, 6, 7)
x7 =
13n. Then
= 270,
andm, n E N+, m > 4 a n d n > 3 .
It is easy to find the positive integer solutions (m,n ) to be
( m , n > = ( 3 3 , 3), (20, l o ) , (7, 17)
which satisfy the conditions above.
When ( m , n ) = (33, 3) , x5 = X6
2, 3, 4), then
= X7 = 13.
Let xi
= 7yi (i = 1,
China Mathematical Competition (Extra Test) 2003
Y1 + Y 2 + Y 3 + Y 4
=
55
33.
We get C ~ =I C?2 = 4 960 solution groups of positive integers
(yl , y2 , y3 , y4 1 and in this case, we have 4 960 solution groups of
positive integers satisfying the conditions.
When(m, n > = ( 2 0 , 1 0 ) , l e t x i = 7 y i ( i = l , 2 , 3 , 4 ) a n d x j =
13yj ( j = 5, 6 , 7). Hence
YI
+Y2
+Y3 +Y4
= 20
andy5 +Y6
+Y7
=
10.
In this case, we have C:9 X
= 34 884 solution groups of positive
integers satisfying the conditions.
When(m, n ) = ( 7 , 1 7 ) , s e t x i = 7 y i ( i = l , 2 , 3 , 4 ) a n d x j =
13yj(j = 5, 6 , 7). Hence
YI
+Y2
+Y3 +Y4
=7
andY5 +Y6
+Y7
=
17.
a
In this case, we have
X c6
:
= 2 400 solution groups of positive
integers satisfying the conditions.
Consequently, for (1) , there are
4 960
+34 884 +2 400
=
42 244
solution groups of positive integers satisfying the conditions.
2003
(Shaanxi)
@BFrom
point P outside a circle draw two tangents to the circle
touching at points A and B. Draw a secant line intersecting the
circle at points C and D , with C between P and D. Choose point Q
on the chord C D such that L D A Q = L P B C . Prove that
LDBQ =/PAC.
Solution Using L D A B = L D C B , L D A B = L D A Q L @ B ,
LDCB =LPBC
L B P Q , and L D A Q = L P B C , we get
+
+
Mathematical Olympiad in China
56
L Q A B = L B P Q , so points P , A , Q , B share a
common circle. Then L B QP = L PAB , that
is, L D B Q +LCDB = L P A C + L C A B . Since
L C D B = / C A B , SO L D B Q = L P A C .
Remark The condition that P A and PB are
tangents to the circle is not necessary. Making
use of this condition intentionally may lead to
further complication.
P
D
{z)={$),
@BLet the three sides of a triangle be integers I ,
satisfyingZ>m
> nand{$)=
m, n, respectively,
where {x}=
x- [x]and [x]denotes the integral part of the number x. Find
the minimum perimeter of such a triangle.
Solution Since
_
3z -
104
["]-3"
lo4
104
["I-
lo4
["I,
3"
104
lo4
we have
3z = 3"
= 3" (mod104 )
3z = 3"
3' = 3"
= 3" ( m 0 d 2 ~ ) ,
= 3" (mod54 ).
(1)
(2)
= 1( m 0 d 2 ~).
Let u be the minimum positive integer satisfying 3" = 1( m 0 d 2 ~).
Then for every positive integer v satisfying 3" = 1( m 0 d 2 ~) , we must
have u I v. Otherwise, if u ! v , then using division with a remainder we
As ( 3 , 2) = 1, we then have from (1) 3""
=3""
+
could get two non-negative integers a and b satisfying v = a u b with
0 <b < u. Then 3b = 3"*
= 3" = 1( m 0 d 2 ~ ) ,contradicting the
definition of u. Therefore u I v.
Notice that
3 = 3 (mod 24 ) , 32 = 9 (mod 24
,
China Mathematical Competition (Extra Test) 2003
57
then u = 4. We may assume that m- n = 4k , where k is a positive
integer.
In the same way, we get from (2) 3""
= 1( m 0 d 5 ~1, that is,
34K=1 (mod 541.
Now, we are going to find number k. As 34K-1 = (1 +5 X 24)' 1 = Om0d5~,i.e.
5k X 24
+ k(k 2
-
= 5k+52k[3+
1)
x 52 x 28 +
(k- 1) X 27]
+
k(k
1)(k
6
k(k-l)(k-2)
3
-
-
2)
x 53 x 212
x 53 x 211
=O ( m 0 d 5 ~ 1 ,
so k
= 5t.
Substituting in the above expression, we get
t+ 5t[3
+ (5t-
1) X 27]
= O(m0d5~1.
Then k = 5t = 53s, and m- n = 500s, where s is a positive integer.
In the same way, we get I-n = 500r, where r i s a positive integer
and r > s since I > m > n.
So the three sides of the required triangle are I = 500r n , m =
500s n and n, respectively, satisfying n > I - n = 500(r- s). When
s = 1, r = 2 the perimeter reaches the minimum which equals (1 000
501) (500 501) 501 = 3 003.
Remark The key to solve the problem is to find the least positive
integer u satisfying 3" = 1(mod104), which, in number theory, is
called the order of 3 with respect to modulus l o 4 .
+
+
+
+
+
+
@& Let a space figure consist of n vertices and I lines connecting these
vertices, w i t h n = ? + q + l , I > ? ( q + 1 ) 2 + 1 ,
4 2 2 , q E N.
Suppose the figure satisfies the following conditions: every four
vertices are non-coplanar, every vertex is connected by at least
one line, and there is a vertex which is connected by at least 4+2
lines. Prove that there exists a space quadrilateral in the figure,
Mathematical Olympiad in China
58
i. e. a quadrilateral with four vertices A , B, C, D and four lines
AB , B C , C D , DA in the figure.
Solution Let V = {&, A1 , A2,
An-l} be the set of all the n
vertices, Bi the set of all vertices adjacent to vertex Ai (i. e. connected
with Ai by a line in the figure) , and the number of the elements in Bi
denoted by I BiI = bi. Obviously,
..a,
n-1
e b i
= 2Zandbi
<(n-l),
i = 0, 1, 2,
.-, n-1.
i=l
If there exists i such that bi = n- 1, without losing generality, we
assumethati=n-1andq+2<n-l1,
andthenwehave
>
< <
That means there exists bj
2 (0 j
n - 2) , so there must be a
space quadrilateral including Aj andA,-I as its vertices in the figure.
Then we consider the case when bi < n- 1, i = 0, 1, 2, , n1, and we may assume that q 2
bo .
We will give the proof by reduction to absurdity. If there is no
such a quadrilateral in the figure, Bi and Bj share no vertex-pair when
i f j , then
+<
n BjJ<i
co<i<j<n-i).
so
I B ~ nBoI>bi-i,
i = i , 2,
-., n-i.
Then we have the number of vertex-pairs inV
n--l
n BO
=
n-I
Ci-b
China Mathematical Competition (Extra Test) 2003
>2 ( n1
-
1)
( n q - q+2-
59
bo)(nq -- q + 3 -
and
(nq-q+2-b0)-((q+l)(n-b0)
>q(q+2)-q-n+2=
As(n-h)(q+l),
= qbo-q-n+2
(3)
1>0.
(n-bo -1)qare positive numbers, we then
get from ( 2 ) and (3),
(nq -4
> q(q+
+2
-
bo ) ( n q - 4 - n
l > ( n - bo)(n-
+3
-
bo )
bo - 1 ) ,
which contradicts to ( 1 ) . This completes the proof.
Remark The question here derives from a research topic in graph
theory, which investigates under what conditions a graph with n
vertices and Z edges contains a quadrilateral.
Mathematical Olympiad in China
60
2004
(Hainan)
In an acute triangle ABC, point H is the intersection point of
altitude CE to AB and altitude BD to AC. A circle with DE as its
diameter intersects AB and AC at points Fand G, respectively. FG
and AH intersect at point K. If BC = 25, BD = 20, and BE = 7,
find the length of AK.
Solution We know that L A D B = L A E C = 90°, therefore
aADBmaAEC,
and
AD
~
AE
-
BD ABCE -AC’
~
(1)
Bu t BC= 2 5 , B D = 2 0 , a n d B E = 7 , s o C D = 1 5 , a n d C E = 2 4 .
From (1) , we obtain
A
D5
~AE
6’
AE+7 5
AD+15
6’
and the solution is
AD
AE
15,
= 18.
=
C
Thus, point D is the midpoint of the
hypotenuse AC of Rt A A E C , and
DE
=
1
-AC
2
=
15.
Draw line DF. Since point F is on the A
-
China Mathematical Competition (Extra Test) 2004
circle with DE as its diameter, L D F E
= 90°,
61
we have
1
AF=-AE=9.
2
Since four points G, F, E and D are concyclic, and four points D,
E, B and C are concyclic too, we get
LAFG
= LADE = LABC.
Thus GF // CB. Extend line AH to intersect BC at point P , then
A
KAF
-AP
-z'
Since H is the orthocenter of AABC, AP
we have
AP
=
(2)
1BC.
From BA
= BC
CE = 24.
Due to (2) , we get
9x24
AK= AF*AP = 8. 64.
AB
25
@@
! In a planar rectangular coordinate system, a sequence of points
{A,} on the positive half of the y-axis and a sequence of points
{B,} on the curve y
1
I OA, I = I OB, I = -.n
a,
=
& (x
The x-intercept of line segment A,B, is
, and the x -coordinate of point B,
(1) a,
0) satisfy the condition
is b, , n E N. Prove that
> an+l > 4, n E N;
(2) There isno E N, such that for a n y n > q ,
b2
b3
-+-+...+bl
+-bn < n-2
bn+l
b2
bn
bn-1
004.
(
Proof (1) According to the stated conditions we have A, 0,
3,
-
Mathematical Olympiad in China
62
2
E+2b,=
( f ).
Thus
b, = / p - l ,
n E N.
On the other hand, the x-intercept a, of line segment A,B,
satisfies the following equation
3
( ~ ~ - 0 &-)
= 0--
(
(
3
(b,-0).
Hence ,
Since2n2b,
=
1- n 2 E
> 0, we have b,
+2
1
n2b,
= -and
China Mathematical Competition (Extra Test) 2004
cn
63
=
1 1 1
Since (2n
\2
+ 1)( n +2)
-
2(n
+ 1>2 n > 0, we obtain
=
1
cn
> n+2
LetSn=cl+c2+.-+cn,
Therefore, if we put
,nEN.
n E N . Ifn=2k-2>1
= 24Oo9
-
(KE
2 , then for any n > no we have
=2 004.
Consequently,
b3
+-bn + bn+l < n +
+
bn
bn-1
bl b2
b2
-
-
-
2 004, n > no.
Mathematical Olympiad in China
64
Remark To prove ( l ) , it is mainly required to determine the
expression of a, which has many different forms, its monotonicity can
be observed directly from the expression of a,. So some students
pointed out that a, >an+l is obvious after writing down the expression
of a,.
About proving (2) one can refer to the test paper of the 19th
Mathematical Olympiad of Soviet Union (1985).
@& For integer n 2 4, find the minimal integer f(n), such that for
any positive integer m ,in any subset with f(n) elements of the set
{ m ,m + 1,
m
n - 1) there are at least 3 mutually prime
elements.
Solution I When n 2 4, we consider the set
..a,
+
M = { m ,m + l , m+2,
. *,
m+n-I}.
If 2 I m , t h e n m + l , m+2, m+3 are mutually prime;
If 2 [ m, thenm, m + l , m+2 are mutually prime.
Therefore, in every n-element subset of M, there are at least 3
mutually prime elements. Hence there exists f(n> and
f(n>
< n.
Let T, = { t I t < n + l a n d 2 I t o r 3 I t } , then T, is a subset of
{ 2 , 3 , , n 1}. But any 3 elements in T, are not mutually prime,
thus f(n> 2
1 Tn I+ 1.
By the inclusion and exclusion principle, we have
+
n+l
n+l
n+l
I Tn I = [I]+
[
T
I
[
T
I
Thus
(1)
2 [I]+
[7
1
[7]+
1.
n+l
f(n)
Therefore
n+l
n+l
China Mathematical Competition (Extra Test) 2004
65
>6, f(8) > 7, f(9) >8.
f(7)
Now we prove that f(6) = 5.
Let X I ,x2 , x3 , x4 , x5 be 5 numbers in { m ,m + l , .-, m+5). If
among these 5 numbers there are 3 odds, then they are mutually
prime. If there are 2 odds among these 5 numbers, then the other
three numbers are even, say x1 , x2 , x3 , and the 2 odds are x4 , x5.
When l < i < j < 3 ,
I xi-xj I E ( 2 , 4). Thus amongxl , x2, x3 there
is at most one which is divisible by 3 , and at most one which is
divisible by 5. Therefore, there is at least one which is neither
divisible by 3 nor by 5 , say, 3 1x3 and 5 1x3. Then x3 , x4 , x5 are
mutually prime. This is to say, among these 5 numbers there are 3
elements which are mutually prime, i. e. f(6) = 5.
On the other hand, { m ,m + l , .-, m+n) = { m ,m + l , .-, m+
n- 1) U { m + n ) implies that
f<n+ 1)
Since f(6)
f(4)
=5
Thus when 4
, we have
f(5)
= 4,
< f(n> + 1.
=
5, f(7)
= 6,
f(8)
= 7,
f(9)
= 8.
< n < 9,
f(n>
[TI+
[3]n+l
=
n+l
n+l
[7]+1.
(2)
In the following we will prove that ( 2 ) holds for all n by
mat hematical induction.
Suppose that equation (2) holds for all n K (K 9). In the case
when n = K 1, since
<
+
{ m ,m + l ,
. *,
m+K)
{m+K-5, m + K - 4 ,
equation (2) holds for n
f(K
=
{ m ,m + l ,
m+K-3,
= 6,
+ 1) < f(K
-
5)
m+K-6)
m+K-2,
n=K
-
>
m+K-1,
5 , we have
+f(6)
-
1
U
m+K),
Mathematical Olympiad in China
66
(3)
By (1) and (3) we obtain that equation (2) holds for n
Consequently, for any n 4, we have
f(n>
=
=k
+ 1.
n+l
n+l
n+l
[T
I
+
[7
1
[7]+
1.
Solution I[ At first, we verify that equations f(4) = 4, f(5) = 5,
f(6) = 5 hold.
Whenn-4, consider{m, m + l , m+2, m+3}. Ifmisodd, then
m, m + l , m+2aremutuallyprime. Ifmiseven, thenm +l, m+2,
m +3 are mutually prime. Thus f(4)
4. But from the set { m, m + l ,
m 2 , m 3) choose a 3-element subset consisting of two evens and
one odd we know these three numbers are not mutually prime, which
implies that f(4) = 4.
W h e n n = 5, consider{m, m + l , m + 2 , m + 3 , m+4}. I f m i s
even, thenm, m+2, m+4 are all even. Then any 3 numbers from the
4-element subset { m ,m + l , m+2, m+4} are not mutually prime, so
f(5) > 4. But from the 5-element universal set we can find out 3
numbers which are mutually prime. Thus f(5) = 5.
Whenn = 6, the elements of the set { m ,m + l , m + 2 , m + 3 ,
m + 4 , m + 5) are 3 odd and 3 even. If we choose a 4-element subset
consisting of 3 evens and 1 odd, then among the subset there are no 3
numbers which are mutually prime. Thus f(6) > 4. Consider the
5-element subset, if among these 5 elements there are 3 odds, then
these 3 numbers must be mutually prime. If these 5 elements are 3
evens and 2 odds, then among these 3 evens, there is at most one
number which is divisible by 3 , and there is at most one number which
is divisible by 5. Hence among these 3 evens there is one number
which is neither divisible by 3 nor 5. The even number and the other 2
odds are mutually prime, which implies that f(6) = 5.
When n > 6, set T, = { t I t n+ 1 and 2 I t or 3 I t}. Then T, is
a subset of { 2, 3,
n
1), and any 3 elements in T, are not
+
<
+
..a,
+
<
China Mathematical Competition (Extra Test) 2005
67
mutually prime. So
f(d2.ITn
n+l
n+l
n+l
1+1= [l]+[T]-[T]+l
Asumen=6K+r,K>l,
r = O , 1, 2, 3, 4, 5, then
n+l
n+l
n+l
[TI+
[TI[TI+
[31-
[Tl+l
= 4K+
r+l
r+l
r+l
[?]+I.
It is easy to verity that
= 0,
r
1, 2, 3,
If r = 0, 1, 2, 3, we can divide n = 6K+rnumbers into K groups:
{ m ,m + l ,
. *,
{m+6(K
m + 5 } , {m+ 6 , m+7,
-
1) m+6K
-
5,
..*)
. *,
m+ll},
m+6K
-
. *,
1)
and left withrnumbersm+6K,
m+6K+r-11. h o n g 4 K + r + l
numbers, there are at least 4K 1 numbers which are contained in the
above K groups. Thus there is at least one group which contains 5
numbers. Since f(6) = 5, there are 3 numbers which are mutually
prime.
If r = 4 , 5 , we can prove similarly that there are 3 numbers which
..a,
+
are mutually prime. Hence, f(n>
2005
In M C , AB
>AC,
=
(Jiangxi)
I is a tangent line of the circumscribed
Mathematical Olympiad in China
68
circle of M C , passing through A. The circle, centered at A
with radius AC, intersects AB at D, and line I at E, F (see the
diagram). Prove that lines DE , DF pass through the incenter and
a escenter of M C respectively.
(Remark : The circle which is tangent
to one side of a triangle and two other
extended sides is called an escribed circle.
The center of an escribed circle is called
an escenter).
Proof First, prove that DE passes through the incenter of M C .
In fact, if we join DE, D C and draw the bisector of LBA C, the
bisector intersects DE, D C at I, G respectively. Join IC. From
AD =AC, we get AG 1DC and ID = IC.
1
Since D, C, Elie on a circle with centerA, /LAC = -1DAC =
2
L I E C . SoA, I, C, Eare on the same circle, andLCIE =/CAE =
B
LABC.
But L C I E
We get
and
=2
LAIC
LACI
1E D , thus LICD
=
lLAB C.
2
= LIGC+LICG
=
'LACB,
2
so I is the incenter of M C .
Secondly, DF passes through an escenter
F
E
A
6
,
;
\ \
\ I
', \',
'41,
of AABC. In fact, the extension of FD intersects the bisector of the
exterior angle of LAB C at II. Join III , BI1 , BI. By (1) , I is the
incenter, we get L I B I l = 90" = L E D I l . So D , B , I1 , I are on the
same circle.
In view of L I B I l = L B D I l = 9 0 " - L A D I
69
China Mathematical Competition (Extra Test) 2005
SOA, I, II are on the same line. Furthermore, II is the escenter
of AABC outside BC.
@@@Assume that positive numbersa, b, c , x, y , zsatisfycy+bz = a ;
a z cx = b and b x ay = c. Find the minimum value of the
+
+
function f(x, y ,
Y2
+-+-.
1+x l + Y
+ cx
Solution By assumption, b(az
bz-a)
= O , i.e. 2bcx+a2-b'-2
the similar reason, y
22
x2
z) = -
=
a2
+2
-
b)
l+z
+ c(bx +ay
= 0 , wegetx=
-
b2
2ac
and z
=
-c) -
a(cy
b2+2-,2
2bc
+
. For
2+@-2
2ab
Since a , b, c, x, y , z are positive, by the above three
expressions, we know b2 2 >a2 , a2 2 >b2 and a2 +b2 >2.Thus
there is an acute triangle AB C with the lengths of its sides a , b, c. So
x = cosA, y = cos B and z = cos C. The problem is now changed to
finding the minimum value of the function
+
+
~ ( c o s A COSB,
,
COSC)=
cos2A
1 cosA'
+
cos2B
cos2c
1 cosB+ 1 cosC'
+
+
+
cot C, then u , v , w E Rf, uv
vw+wu = 1, u 2 + 1 = ( u + v ) ( u + w ) , v2+1 = (u+v)(v+w) and
w2+1= ( u + w ) ( v + w ) .
Set u = cotA, v
=
cot B , w
=
U2
We get
cos2A
+ cosA
1
-
u2+1
I+-
U
-
U2
m ( d u 2 f l f U )
Mathematical Olympiad in China
70
.+)'
&-u3
1
2L+v
u+w
By a similar argument,
cos2c
1+cosc
and
Hence
f
u2
cos2B
1+cosB
+
+
~
v2+w2-
vw
(v2-
+-
-
'1
-
2
(u2 -uv
+w2>+
(u2
1
2
= -(uv+vw+uw>
) +' v+w
-
.+)' v+w
w3 1
2 iu+w
2
2.--
+ v2+ w2-
=u2
1
2 L + v
2
2.-v 3
+
v2)
- uw2+
w2
>j
1
2
=-9
the equality sign is valid if and only if u = v = w, i. e. a = b = c , x =
y=z=--,
1
2
so
@& For each positive integer, define a function
lol
if n is the square of an integer,
-
f(n)=i
L&l
,
if n is not the square of an integer.
(Here [x] denotes the maximum integer not exceeding x , and
xf(K>.
200
{x}= x- [x].) Find the value of
k= 1
Solution For arbitrary a , K E Nf
, if K2 < a < (K + 1>2, we set
China Mathematical Competition (Extra Test) 2005
a
=
k2 + m , m
=
1, 2,
..a,
71
2k,
&=k+O,O<O<l,
2k+O
0<--m
For
2k
<1,
2k
m
2k+O
m
if there exists an integer t between - and -, then
2k
m
2k+O
m
-<t<-.
<mt , thus 2k + 1 <mt. On the other hand,
mt < 2k + O< 2k + 1 , a contradiction.
On one hand 2k
Thus
26
Next, we calculate
;=I
*
2k
[-1.
z
Draw a 2k X 2k table, and put a star
at each place in the i-th row where it is a multiple of i. Then in this
row there are
[71stars and the total number of stars in the table is
5[71.On
the other hand, in j-th column, if j has T ( j ) positive
,=l
factors, then there are T ( j ) stars in this column. Thus the total
2k
T ( j )and
number of stars is
j=l
For example:
26
;=I
2k
[T]
z
26
=
j=l
T(j).
Mathematical Olympiad in China
72
Thus
a=l
k=l
j=l
+ T(2)]+
=n[T(I)
+ ...+ [T(2n16’
a
1
g
2
3
5
3
4
6
6
(16 - k) [T(2K - 1)
5
7
6
8
6
+ T(2k) 1.
(3)
7
8
9
8
9 1 0 1 1 1 2 1 3 1 4 1 5
8
8
1
0
7
1
0
1
0
15
(16 - k>ak
=
k= 1
, 15. It is easy to see that
= 1, 2,
cf(k) c
256
Therefore,
(2)
k= 1
Set U k = T(2K-1) +T(2K), K
takes the following values.
k
+ T(2n)l
15
=
k= 1
ak
1)
cf(K) c
From (2),
+ T(4)I
( n - 1)[T(3)
=
783.
(4)
k= 1
Note that f(256) = f(162> = 0 by definition. When K E
(241, 242,
255}, denotek = 1 5 2 + r ( 1 6 < r < 3 0 ) , then
..a,
China Mathematical Competition (Extra Test) 2005
r
-<
31
r
r
< - 9
d m + 1 5
30
l<-<
1
31
<--2.
30
Thus
(5)
c
200
Therefore,
73
f(K)
k= 1
cf(K)
256
= 783 -
k=U)l
= 783 - 15 = 768.
China Mathematical
Olympiad
T h e China Mathematical Olympiad, organiEd by the China
Mathematical Olympiad Committee, is held in January every year.
About 150 winners of the China Mathematical Competition take
part in it. The competition lasts for 2 days, and there are 3
problems to be completed within 4.5 hours each day.
2003
(Changsha, Hunan)
2003 China Mathematical Olympiad and 18th Mathematics Winter
Camp was held on January 11- 18 in Changsha, Hunan Province, and
was hosted by Hunan Mathematical Committee and Changsha No. 1
middle school.
China Mathematical Olympiad 2003
75
The Competition Committee consisted of the following: Xu
Yichao, Li Shenghong, Shen Wenxuan, Chen Yonggao, Su Chun,
Leng Gangsong, Huang Yumin , Huang Xuanguo .
First Day
8:OO- 12:30January 15, 2003
-.=-s
i
~2
Suppose points I and H are the incenter and orthocenter of an acute
triangle AB C respectively, and points B1 and Cl are the midpoints
of sides AC and AB respectively. It is known that ray B1 I intersects
side AB at B2 (B2 # B ) and ray Cl I intersects the extension of AC
at C, : B2 C, and B C intersect at K and A1 is the circumcenter of
ABHC. Prove that three points A, I and A1 are collinear if and
only if the areas of ABKB2 andACKC2 are equal. (posed by Shen
Wenxuan)
A
Proof First, we will prove that
three points A, I and A1 are
collinear H L B A C = 60".
As shown in the figure, assume
that 0 is the circumcenter of
A A B C. We join BO and CO , then
\
LBHC
=
180"- L B A C ,
L B A l C = 2(180" - L B H C )
Hence, L B A C
=
\
'A ,
= 2LBAC.
60"HLBAC+ L B A l C = 180"
H A1 is on the circumcircle 00 of A A B C
H Al andAA1 coincide (becauseA1 is on the perpendicular
bisector of BC. )
H Three points A, I and Al are collinear.
Secondly, we will prove S~JKB = SacKcz#LBAC
= 60".
Mathematical Olympiad in China
76
Construct IP
1A B at point P , and IQ 1AC at Q , then
SMB,B,
=
1
-IP
2
Note that
SMB,B,
therefore
IP A B 2
=
1
*AB2+--IQ*ABl.
2
1
2 A B 1 AB2
+ IQ
AB1
sinA,
= AB1
sin A.
AB2
Assume IP = r , where r is the radius of the inscribed circle of
U B C . Then IQ = r. Furthermore set BC = a, C A = b a n d A B = c,
then r
=
2sMBC
a+b+c'
b
FromAB1 = - and
2
we have
AB2
2AB1 sinA
=
=
~SAABC
9
C
(2syc
-2
2sAABc
a+b+c
AB2
=
bc
a+b+c'
Similarly,
AC2
=
bc
a+c-bb'
Hence ,
sABKB,
so
h,
]
=b
2sMBC
a
c,
+
+
= SACKC,
H SAABC
= SMB,C,
H bc
=
bc
a+b-c
bc
a+c-b
Ha2=b2+2-bc
HLBAC
= 60" (By
the law of cosines).
Therefore, the proposition is true.
@@
Suppose
&
a set S satisfies the following conditions:
(1) every element in S is a positive integer and not greater than
100 ;
China Mathematical Olympiad 2003
77
(2) for any two different elements a and b in S, there is an
element c in S such that the greatest common divisor of a and
c is equal to 1, and the greatest common divisor of b and c is
also 1; and
(3) for any two different elements a and b in S, there is an
element d , which is different from a and b, such that the
greatest common divisor of a and d , and that of b and d are
greater than 1.
Find the maximum number of elements in S. (posed by Yao
Jiangang)
Solution The maximum number of elements is 72.
A positive integer not greater than 100 can be written as
n
= 2"1
.3% . .7"4 .11%.
5"3
q,
where q is a positive integer and not divisible by 2 , 3 , 5 , 7 and 11, and
a1 , a2 , a3 , a4 and a5 are nonnegative integers.
We pick out those positive integers n with just one or two nonzero
among a1 , a2 , a3 , a4 and a5 to form set S. In this case, S contains 50
even numbers (2 , 4, , 98 and 100) except the following seven: 2 X
3x5, 22X3X5, 2X32X5, 2X3X7, 22X3X7, 2X5X7and2X
3 X 11, 17 odd numbers that are multiples of three ( i. e. 3 X 1, 3 X
3,
, 3 X 33) , 7 odd numbers with the least prime divisor 5 ( i. e.
5x1, 5x5, 5 x 7 , 5x11, 5 x 1 3 , 5 X 1 7 a n d 5 X 1 9 ) , 4oddnumbers
with the least prime divisor 7 ( i. e. 7 X 1, 7 X 7, 7 X 11and 7 X 13),
and the prime number 11.
+ +++
Consequently, S contains (50 - 7) 17 7 4 1 = 72 numbers
tot ally.
In what follows, we will prove that S constructed above satisfies
the given condition.
Obviously, it satisfies condition (1).
For condition (2) , we note that, at most, four prime divisors
among 2, 3, 5, 7 and 11 will occur in [a, b]. We write the prime
which does not occur, as p. Obviously, p E S and
Mathematical Olympiad in China
78
Hence, we take c = p.
For condition (3) , we take the least prime divisor of a as p and
the one of b as q when (a, b) = 1. It is easy to see that p # q and p,
q E ( 2 , 3, 4, 7, 11). Hencepq E S, and
(pq, a ) > p
> 1and (pq, b) > q >
1.
Being coprime to each other for a and b ensures that pq is different
from a and b. Thus we take d = pq.
When ( a , b) = e > 1, we take p as the least prim divisor of e , and
q as the smallest prime number satisfying q / [a, b]. It is easy to see
t h a t p f q , a n d p a n d q E (2, 3, 5, 7, 11). Hencepq E S, and
(pq, a ) > ( p , a ) = p > 1 ,
(pq, b)
q ! [a, b]
d
> (p, b)
=p
> 1.
ensures that pq is different from a and b. Thus, we take
= pq.
In what follows, we prove that the number of elements in S,
which satisfies the conditions described in the problem, will not be
greater than 72.
Obviously , 1 @ S. For arbitrary two prime numbers p and q which
are both greater than 10, since the least number which is not prime to
neither p nor q is pq, it must be greater than 100. So we know,
according to condition (3) , that there is at most one among 21 prime
89 , 97) , occurring in S. We
numbers between 10 and 100 (11, 13,
write the set consisting of all natural numbers not greater than 100
except 1 and the above-mentioned 21 prime numbers as T, and there
are 78 numbers in the set. We can conclude that there are at least 7
numbers in T a r e not in S. Thus S contains at most 78 - 7 1 = 72
elements.
(9 When a prime number p is greater than 10 and belongs to S,
..a,
+
China Mathematical Olympiad 2003
79
every number in S can only have 2 , 3 , 5 , 7 and p as its least prime
divisor. By condition (2) , we have the following conclusions.
0If 7 p E S, because ( 2 X 3 X 5, 22 X 3 X 5, 2 X 32 X 5, 7p)
contains all the least prime divisors, we know from condition (2) that
2 X 3 X 5 , 22X3X5and2X32X5donotbelongtoS.If7p@S,noting
2 X 7 p > 100, but p E S, so from condition (2) we know that 7 X 1,
7 X7, 7 X 11and 7 X 13 do not belong to S.
0If5p E S, t h e 1 1 2 X 3 X 7 a n d 2 ~ X 3 X 7 d o n o t belong toS. If
5p @ S, then 5 X 1and 5 X 5 do not belong to S.
02 X 5 X 7 and 3p do not belong to S at the same time.
@ 2 X 3 p and 5 X 7 do not belong to S at the same time.
0If 5p, 7 p @ S, then5 X 7 @ S.
Whenp = 11or 13, from 0,
0,
0 and @, we can get at least
3 , 2, 1 and 1 numbers in Trespectively which do not belong to S, and
in total 7 numbers. When p = 17 or 19, from 0,
0 and 0,
we can
get at least 4 , 2 and 1 numbers in T respectively, which do not belong
to S and in total 7 numbers. When p > 20 , from 0, 0and 0, there
are at least 4 , 2 and 1 numbers in T respectively, which do not belong
to S and in total 7 numbers also.
(ii) If there is no prime number greater than 10 belonging to S,
then the least prime numbers in S can only be 2 , 3 , 5 and 7. Hence ,
each of the following 7 pairs of numbers can not belong to S at the
same time:
(3,2X5X7),
(5,2X3X7),
(7,2X3X5),
(2X3,5X7),
( 2 x 5 , 3X7), ( 2 x 7 , 3X5), (22X7, 32X5).
Thus, there are at least 7 numbers in T that are not in S.
Consequently, the answer for this problem is 72.
@& Given a positive integer n, find the least positive number A such
that cos 01 cos 02
cos 0, is not greater than A provided
+
+ +
Mathematical Olympiad in China
80
n). (posed by Huang Yumin)
When n = 2, we can prove
cos
and when 01
In fact,
=
e2 = arctan&,
0
the equality holds.
+cos~e2+~cose1.cose2<-, 34
am
that is,
43
el + cos e2 < 2,
3
+ tan2el + 1+ tan2e2
1
1
0
From tan el
0w
tan 0, = 2, we get
+ tan2el + tan2e2
5 + tan2el + tan2e2
2
that is,
36(5+x)
Obviously,
Hence, A
When n
< 196+28x++,
> 0.
@ W x - 8x+ 16 = (x- 4>2
243
3 .
=-
> 3 , there is no loss of generality in supposing 0, > 0,
> > en, then
tan 01
tan O2
tan e,
> 2&.
China Mathematical Olympiad 2003
Since cos 0,
=2/-
From tan2el
<1
> tan2
1
cos2
8
02
1
-sin2ei
2
tan2&
02
tan 02
2/8
, so
, we have
8 + tan2
>
el
tan282
cosel G
that is,
tan203
-
81
+ tan2
’
tan2e3
tan 03
02
tan2 e3
Hence
cos e2
+ cos e3 + cos el
r
1
1
Note that
8cos202 cos2O3
~8
+ tan202 tan2
03
+ sin2&
> cos2e,
sin2e3
1
cos2e3
+ tan2e2) (1 + tan20, )
H t a n 2 + tan2 < 7.
= (1
02
03
Thus, we get that, when @ holds,
el + cos e2 + cos e3 < 2.
If @ does not hold, then tan20, + tan20, > 7,
cos
>1
Mathematical Olympiad in China
82
hence
tan2
el > tan2e2 > 72 ,
-
so
Thus
cos
el + cos e2 +cos e3 < @+
3
1< 2,
that is, 0 holds too.
Hence we can get
cos
el + cos e2 + cos e3 + ...+ cos en <
On the other hand, if we take 0,
= 0, =
m.0
-
I.
= 0, = a
>0, a-
0,
then
Obviously, 0,
cos
-
iT
-
2
, thus
el + cos e2 + cos e3 + ...+ cos en
Consequently, we get A
=
-
-
I.
n- 1.
Second Day
8:OO-12: 30
January 16, 2003
>
@.$@Find all ternary positive integer groups (a, m, n>satisfying a 2
and m 2 such that an 203 is a multiple of a"
1. (posed by
Chen Yonggao)
Solution We will discuss the following three cases for n and m.
(i) In the case when n < m , from an 203 a" 1, we have
>
+
202
Therefore ,
>a"
-
an
>an( a
+
-
+ > +
1) > a ( a 1).
2<a<14.
-
China Mathematical Olympiad 2003
83
Whena = 2, we can taken to be 1, 2, .-, 7.
Whena = 3, we can taken to be 1, 2, 3 and 4.
Whena = 4, we can taken to be 1, 2 and 3.
W h e n 5 < a < 6 , w e c a n t a k e n t o b e l and2.
W h e n 7 < a < 14, n = 1.
From a"
1 I an 203 , we can deduce that the solutions are (2,
2, 0 , (2, 3, 2) and(5, 2, 1).
(ii) In the case when n = m , a" +1 I 202. Since 202 has only four
divisors 1, 2, 101 and202, b u t a > 2 , m > 2 a n d a m + l > 5 , SOU" =
100 or 201. Since m
2, therefore the solution is (10, 2, 2).
(iii) In the case when n >m,from a"
1 I 203(a"
1), we have
+
+
>
U"
that is,
+
+
(203~"+ 203)
+ 1 I an +203
a" + 1 I ~ " ( d - "
-
-
Since (a"
+ 1, a")
=
203).
1, so
a m + l I ~"-"-203.
0 Ifan-"<203,
thensetn-m=s>l,
wehavea"+l
1203-
as. Hence
203-as > ~ " + 1 ,
202
thus
> as +a" >a" + a
=a(arn--l + 1) > a(a+ l ) ,
2 < a < 13.
Using the same orgument as in Case ( i ) , we show that the
solutions for ( a , m, s) are:
(2, 2, 3), (2, 6, 3), (2, 4, 4), (2, 3, 5), (2, 2, 7),
(3, 2, l ) , (4, 2, 2), (5, 2, 3) and(8, 2, 1).
Hence, ( a , m, n ) are
(2, 2, 5), (2, 6, 9 > , (2, 4, 8>, (2, 3, 8 > , (2, 2, 9 > ,
(3, 2, 3), (4, 2, 4), (5, 2, 5) and(8, 2, 3).
Mathematical Olympiad in China
84
@ If an-"
= 203,
then a = 203 and n-m
= 1, and
the solution is
(203, m, m + l > , 7 x 2 2 .
~Ifan-">203,setn-m=s>1,
thena"+l
Since as -203 >a"
1, so s > m. By
a"
+
+ 1 I as+203~"
= (as-"
-
and
a")
(a"+l,
we have
a"
(an-2m
=
I as-203.
+203)~"
+203)~",
1,
+ 1 I an-'" +203.
Now s > mWn - m > mWn > 2mWn - 2m > 0. In this case, the
solutions can only be derived from the preceding solutions, that is,
from ( a , m , n) ( a , m , n+2m)
( a , m, n+2Km) , and each
1 I an 203.
solution derived also satisfies a"
Summarizing what described above, we obtair all solutions ( a , m,
n) to be:
-
-..a-
+
+
(2, 2, 4K+1), (2, 3, 6K+2), (2, 4, 8K+8), (2, 6, 12K+9),
(3, 2, 4K+3), (4, 2, 4K+4), (5, 2, 4K+1), (8, 2, 4K+3),
(10, 2, 4K+2) and(203, m, ( 2 K+l)m +l),
where K is any nonnegative integer, and m
>2 is an integer.
$$&& A certain company wants to employ one secretary. Ten personsapply.
The manager decides to interview them one by one according to
the order of their applications. The first 3 applicants should not
be employed. From the fourth onward an applicant will be
compared with the preceding ones. If he exceeds in ability all the
procedings applicants, he will be employed. Otherwise he will
not, and the interview goes on. If the preceding nine persons are
not employed, the last one will be employed.
Suppose that the 10 persons are different from each other in
ability, and we can arrange them according to their ability rating
China Mathematical Olympiad 2003
85
from superior to inferior, such as lst, 2nd,
10th. Obviously,
whether an applicant will be eventually employed by the company
depends on the order of the applications. As it is known, there are
l o ! such permutations in all. Now denote by Ak the number of the
permutations such that the applicant with the k t h ability rating is
Ak
employed and the probability for him to be employed is -.
(posed
..a,
lo!
by Su Chun)
Prove that under the policy given by the manager, we have the
following properties:
(1)Al >A2 > . . * > A s =A, =Ale.
(2) The probability for the company to employ one of the
persons with the ability among the three is over 70% , and to employ
one of the persons with the ability among the bottom three is not over
10%. (provided by Su Chun)
Proof We denote by a the ability rating of the applicant with
the highest ability among the first three interviews. Obviously, a<8.
Now we denote by Ak ( a ) the set of permutations for which the person
with the k t h ability rating is employed and we denote the
corresponding number of permutations by I Ak ( a ) 1.
(1) It is easy to see that, when a = 1, a definite result is to give
up the first nine persons and to employ the last interviewee. Now the
chance is equal for every person except the one who has the highest
ability rating. It is not difficult to see that
I &(I)
I = 3 X 8 ! : = r l , k = 2 , 3,
. *,
10,
where “ :=” denotes “writter as”.
When 2 a 8, a person who is placed K-th in ability rating will
have no chance to be employed for a K 10, and the chance is equal
for 1 K < a.
In fact, a person with the a-th ability rating is among the first
three interviews and there are three ways to choose one out of the
three interviews. Those people with their ability rating from the 1-st
<<
<
<<
Mathematical Olympiad in China
86
to ( a - 1)-th are among the later seven interviews, and the first one
having the highest ability rating among them is employed. There are
C7-I ( a -2) ! ways of such arrangement. The other 10-a persons may
be arranged arbitrarily on the remainder positions, and we have (10a ) ! ways of arrangement. Hence, we have
The result above shows :
As=Ag=Alo=rl
=3x8!>0;
+ cr , , k
0
8
Ah
= r1
=
2,
..a,
7;
0
a=ktl
cr,.
8
A1
and
=
0
a=2
From
0and 0,
we obtain
and from 0and
>..*>&=A9
>A3
A2
=A10
>O;
0,
we obtain
A1 -A2
= r2
- r1
=3X7
X 8! - 3 X 8! > 0.
Consequently, (1) is proved.
(2) From 0,
we know
So the probability for the company to employ one of the bottom
three is equal to 10%.
we can show that
From 0and 0,
9
9
a=2
a=2
=3X7!):
a=2
(9-a)(lO-a)
a-1
China Mathematical Olympiad 2003
87
(8-s)(9-s)
=3X7!):
S
S l
12
56+21+10+5+-+l+5
=3 X 7 ! x 9 5
24
35
>3 X 7 ! x 9 5
-
2
3
-
cru
8
A2 =r1+
a=3
=3X8!+3X7!X
12
21+10+5+-+l+5
cru
8
A3 =r1+
a=4
=3X8!+3X7!X
12
10+5+-+l+5
24
2
= 3 ~ 7 !X 2 6 - > 3 X 7 ! X 2 6 - = 8 0 X 7 ! .
35
3
Therefore,
Ai+&+&
lo!
>287+143+80
720
-
510
720
~
17
24
=-
> 70%.
That is, the probability for the company to employ one of the top
three, is greater than 70%.
+
Suppose a , b , c and d are positive real numbers satisfying ab
cd =1and Pi (xi, yi> (i = 1, 2 , 3, 4) are four points on the unit
Mathematical Olympiad in China
88
circle which has the origin as its center. Prove that:
<
(y
+-1.
(a~i+b~2+~~3+d~4)~+(ax4+bx3+cx:2+dx1)~
2
2+&
(posed by Li Shenghong)
cd
Proof I Set u = ayl +by2, v = cy3 +d y4 ,
d x l . Then
cx2
u1 = ax4
+
u2
< ( a yl
by^)^
+ ( a x1
-
b2
< a2 +2ab
b x ~ ) ~
-
that is
XlX2
-
YlY2
YlY2 -x1x2
+bx3 and vl =
u2
-V?
< 2 + &2cd
we get
0+ 0,
u2
a2+@
-+< -+-*
ab cd
ab
2+d2
cd
2+&
+-.a’+@
ab
V?
that is
v2
u?
-+-<cd ab
Similarly
cd
By Cauchy’s inequality, we have
+ d 2+ +
v2
< ( a b + c d ) ( -+a b c d )+ ( a b + c d )
(u
(u1
u2
V l >2
I:+:[
a2+b2 + -21.+ d 2
-+-+-+pv?
ab cd ab
ab
cd
= u2
212
u?
0
0
China Mathematical Olympiad 2003
89
Proof II By Cauchy’s inequality, we can show
(aYl +by2 +CY3 +dY4)2
a
b
d
+ L Y 5 + CdY ? +-Y$
C
= -Y?
b
Similarly,
( a x4
+2(y1y2 +y3y4).
+b x3 + cx2 +d x l
b
< TaX $ +-x?
+-xi
a
d
C
)2
d
+-x? +2(XIX2
C
+X3X4).
So we subtract the right-hand side (RHS) from the left-hand side
(LHS) in the original inequality and get
a 2
-bx 4
-
d
c
+ ab 2 + -x;
d
+ -x?
a X 2I
-b
-x3
b 2
a
- -XI
c 2
d
-Y2
d
--Y?C
<
-
C
c 2
d
--x3
+2(XIX2
+2x1x2+2x3x4
d 2
--x4
C
+x3x4
-
a
-y$
b
=0.
The proposition is proved.
b
-y?
a
-
+y1y2 + y 3 y 4 )
2XIX2 - 2X3X4 - 2Yl Y2 - 2Y3Y4
2(XIX2 + x 3 x 4 +YlY2 +Y3Y4)
-
-
+
Mathematical Olympiad in China
90
2004 ChinaMathematical Olympiad and 19th Mathematics Winter
Camp(for secondary school students) was held on January 6 - 11 in
Macao, and was hosted by Education and Youth Affair Bureau of
Macao Special Administrative Region of PRC.
The Competition Committee consisted of the following: Chen
Yonggao, Liang Yingde, Huang Yumin, Li Shenghong, Yu
Hongbing, Li Weigu, Qin Hourong, Xiong Bin, Wang Jianwei, and
Xu Jiangxiong.
First Day
8 :30 - 13 :00 January 8, 2004
Let EFGH ,ADCD and El Fl GI HI be three convex quadrilaterals,
satisfying: (a) Points E , F , G and H lie on sides AB , BC , CD
A E BF CG DH
and DA , respectively, and - = 1; (b) points
E B FC GD HA
A, B , C and D lie on sides H l E l , El FI , FIGI and GIHI,
respectively, andElF1 / / E F , FIGI / / E ,GIHI //GH, HIEI //
~
~
Fl C
El A
= A , find the expression of
in terms of A.
HE. supposeCG1
AH1
(posed by Xiong Bin)
~
Solution (1) If EF
so
// AC,
then
BE
BF
-~
DG using condition ( a ) . Then
HA
GC
HG //AC, giving EIFl // AC // HlGl. That
~
FI
C
China Mathematical Olympiad 2004
91
F l C - E1A
means - -- A.
CG1 AH1
(2) If EF is not parallel to AC, extend
CF
FB
AT
BE
EA
Menelaus' Theorem, we have
T
t\,
;i i
1 1 '
!
B
I
' \ 'I II
\\ I I
AT
CG D H
=1, and then
1 using
GI
TC
GD HA
=
E;
C
condition ( a ) . By the inverse of Menelaus'
Theorem, we know that points T, H and G are collinear. Suppose
lines TF and TG meet line El HI at M and N respectively. As EB1 //
BA
AD
EF, we get E1A = - AM. In the same way, we get H1A = EA
AH
EiA - AM AB AH
AN. Then
0
T A P A N A E AD'
~
~
\I I
,I"I I
~
.-.-
On the other hand,
AM EQ AAEC - AABC AE AD
= -=
AN QH a A H C - A A D C * A B * A H '
~
~
~
0
From
EIA
0,
0we get -
EQ
H ~ A-QH
-
-
AH
AABC
.--*AE AD m c . In the
AB
-
same way ,
so
FlC
CG1
-
E1A
AH1
-
A.
Let c be a positive integer, and a number sequence XI
satisfy x1 = c and
, x2 ,
where [x] denotes the largest integer not greater than x.
Mathematical Olympiad in China
92
Determine the expression of x, in terms of n and c. (posed by
Huang Yumin)
Solution
Let
u,
Obviously, X,
=
+ [2(xn-A
= xnp1
A (n+ l)(n+2>
,
2
nonnegative integer. Since
=A
(n
n
=
-
1, 2,
+ 1)(n+ 2)
2
'1
..a,
= u,,
for n 2 2. Let a,
where A is a
forn22,
the sequence { u , } satisfies 0.
Let y, = n , n = 1, 2,
Since
..a.
r
9 1
the sequence {y,} satisfies 0 too.
Letz,=[( n
+2>2] , n = l , 2 ,
..a.
Thenwehave, f o r n = 2 m
a n d m 2 1,
=[+(m+l)
For n = 2m+ 1 andm 2 1,
1
=
(m+1>2
= z,.
China Mathematical Olympiad 2004
So, the sequence {y,} satisfies 0 too.
For any nonnegative integer A, let v,
(n
+ 1)( n + 2 ) + n ,
2
=A*
W, = u,+z,
n
= u,
93
+ y,
1, 2,
A
9
2
=
=
.a*.
Obviously, both {v,} and {w,}satisfy 0.
Sinceul
a,
= 3A,
y1
=
1,
z1 =
[$1=2, then for3 I
a1
we have
a1
= -(n+l>(n+2>.
6
Forq
= l(mod 3),
a,
a, =
a1 -1
~
6
(n
+ l ) ( n + 2) + n. For a1 =
a1 -2
=
~
6
In summary,
c-1
x, = - ( n + l ) ( n + 2 ) + 1 ,
6
c-2
x, = -(n+
1)(n+2)
6
c-3
x, = -(n+
l ) ( n 2)
6
f o r c = l ( mod3);
+ n+ 1, for c = 2(mod 3);
+ + [( n :2)2]+
1, for c = O(mod 3).
94
Mathematical Olympiad in China
@@ Let M be a set consisting of n points in the plane, and satisfying:
(1) there exist 7 points in M which constitute the vertices of a
convex heptagon; (2) if for any 5 points in M which constitute
the vertices of a convex pentagon, then there is a point in M
which lies in the interior of the pentagon.
Find the minimum value of n. (posed by Leng Gangsong)
Solution First, we prove that n 11. Suppose a convex heptagon
has its vertices in M given by A I A ~ A ~ A A ~ AUsing
~ A ~Condition
.
(l), we get that there exists one point PI belonging to M in the
interior of convex pentagon A1A2A3A4A5.Connecting PIAl and
P1A5, we obtain that there exists one point P2 in convex pentagon
Al P1A5&A7 so that P2 belongs to Mand is different from PI. Then,
there are at least 5 points in {Al , A2 , A3 , A4 , A5 , A6 , A7 } which do
not lie on line PIP2. By the Pigeon Hole Principle, there exist at least
3 points on one side of line PI P2 , and these 3 points together with PI
and P2 constitute a convex pentagon which contains at least one point
P3 belonging to M.
Now, we have three lines PI P2 , P2P3 and P3P1 , which form a
triangle APl P2P3. Let x1 denote the half-plane on one side of line
PIP2 which is opposite to APl P2P3 and contains no points on PIP2.
In a similar way, we define x2 and x3. Areas XI , x2 and x3 cover the
entire plane except APl P2P3. By the Pigeon Hole Principle, there is
one area of x1 , x2 and x3 which contains at least 3 points belonging to
{A1 , A2 , A3 , A , A5 , A6 , A7} , Without loss of generality, we
assume that the area x1 contains points Al , A2 , A3 , then there exists
one point P4belonging to M within the convex pentagon constituted by
A1 , A2, A3, PI and P2. So, n 11.
Now, we give an example to illustrate that
n =11 is attainable. As seen in the figure, set M
consists of integral pointsAl , A2 , A3 , A4 , A5 ,
&, A7, and four integral points within the
heptagon A1A2A3A4A5&A7.Obviously, M
satisfies Condition (1). We are going to
>
>
China Mathematical Olympiad 2004
95
prove that M also satisfies Condition ( 2 ) .
By reduction to absurdity, assume that there is a convex pentagon
with its vertices belonging to M which contains no point of M in its
interior. Then among such pentagons there must be one, denoted by
A B C D E , which has the least area, since the value of the area of a
polygon with integral vertices is always in the form of ? ( n E N).
2
There are only 4 cases concerning the odd/even property of the
xycoordinate of a integral point: (odd, even) , (even, odd) , (odd,
odd) , (even, even). So there must be two vertices among A , B, C ,
D , E which have the same odd/even property, and the midpoint of
the segment formed by these two vertices, say P, is also an integral
point and belongs to M. By definition, P is not in the interior of
pentagon ABCDE, then it must be on one side of the pentagon.
Assume that P is on the side A B , then it must be the midpoint of A B ,
and PBCDE is a convex pentagon with strictly less area than that of
ABCDE.
So, the minimum value of n is 11.
Second Day
8 :30 - 13 :00 January 9, 2004
a- For a given real number a and a positive integer n , prove that:
.--
(1) there exists exactly one sequence of real numbers xo , x1,
x, , x,+1 , such that
( 2 ) the sequence xo , x1 , , x, , x,+1 in (1) satisfies
I a 1 , i = 0, 1,
n + l . (posed by Liang Yingde)
I
xi
..a,
I<
..a,
+
Solution (1) Proof of existence: From xi+l = 2xi 2 d - 2a3 xi-I,i = 1, 2 ,
n andxo = 0, we get that xiis a polynomial of x1
..a,
Mathematical Olympiad in China
96
<
with degree 3i-1 and real coefficients, for 1 i < n+ 1. Specifically,
xn+l is a polynomial of XI with degree 3" and real coefficients. As 3" is
an odd number, there exists a real number x1such that xn+l= 0. Then
from this XI and xo = 0 we can calculate xi. The sequence xo , XI , ,
xn , xn+l obtained in this way satisfies the required condition.
Proof of uniqueness: Suppose there are two sequences w o ,
v, , v,+l , both of which satisfy
WI .-, w n wn+l and vo , vl ,
..a,
1
Condition (1). Then-(wi+l +wi-l)
2
~ i - 1 ) = ~ l i v?- u3. Thus,
=
+
1
(wi+l
-
2
=
1
wi+w?-u3 and-(vi+l
2
+wi-1
- Vi+l
+
- Vi-1)
(wi- v i ) ( 1 +w:+wivi + v : > .
Suppose I wio - vi0 I is the greatest. Then
Iwi - v i
I<
I(l+w:+wi
Iwi - v i
< y1 I Wi0+l
<Iwi
So, either I wi
either I wi
-vi
I
vi0+1I
I = 0 or (l+w:
0
0
+ 1 I wio-l
- "io-l
I
I.
-vi
I = 0 or w: +v:
vi + v : >
0
+ w i0 vi0 +v: ) = 0, that is,
+(wi +vi
)2 = 0.
However it must
Since I wi -via I is the greatest. Then
I wi - vi I = 0 for every i = 1, 2 , , n. This completed the proof of
(1).
(2) Suppose that I xio I is the greatest. Then we have
be
I wi
- vi
- vi
-
0
=
I"io
0.
I+
I"io 13 = I"io
I(1+
1
<y~xio+l
I"io
12)
I +1 ~ I+
I ~ ~ ~ - ~
1 4 3
!n
China Mathematical Olympiad 2004
%&@ For a given positive integer n
ai(i =1, 2,
..a,
n> satisfy a1
> 2,
suppose positive integers
< a, and " -1 1.
c
< al <
;=I
Prove that, for any real number x
holds,
,
ai
<
the following inequality
(posed by Li Shenghong)
Solution For 2
>al (al
c
n
-
1) , from
;=I
r
1
-
ai
< 1we have
For 2 < al (al - 1>,using the Cauchy Inequality, we have
ai
Further, for positive integers a1 <a1 <
1 and
+
2a;
2a;
<a,,
we have ai+l
>
Mathematical Olympiad in China
98
1
<
(Ui
for i = 1, 2,
..a,
-+I2
1
-
+2
(%+I
-y 12+2'
So
n-1.
%!&Prove
B
that every positive integer n , except a finite number of
them, can be represented as a sum of 2 004 positive integers: n =
< a20049 and
a1 a2
a 2 0 0 4 9 where 1
a1 < a2 <
ai I ai+l , i = 1, 2,
2 003. (posed by Chen Yonggao)
Solution We are going to prove a more general result: For any
positive integer r 2, there exists N ( r ) E N such that for every n
N ( r ) , there are positive integers al , a2 , , a, satisfying
+ + +
<
..a,
>
n
>
= a1 +a2
<a,,
ai
I
+
Ui+l,
+a,, 1
i
=
< a1 < <
1, 2,
a2
..a,
r-1.
F o r r = 2, we haven= l + n - 1 , thenN(2) = 3.
Suppose it is true for r = K , then for r = K 1 let N(K 1) =
4N(K)3. For any positive integer n = 2"(21
1)
N(K
1) =
4N(K)3 , we have either 2"
2N(K)2 or 21 1 > 2N(K).
If 2" 2N(K)2 , there exists an even positive integer 2t a such
that 22t N(K>2, then 2t 1 N(K). By induction we get positive
integers bl , b2 , , bk such that
+
>
>
>
+ >
+
+ >
+
+
<
2005 China Mathematical Olympiad and 20th Mathematics Winter
Camp was held on January 20 - 25 in Zhengzhou, Henan Province,
and was hosted by Olympiad Committee of CMS, the editorial
deboard of (( Zhong Xue Sheng Shu Li Hua ( %&
' & H f i ) )) and
Zhengzhou Foreign Language School.
The Competition Committee consisted of the following: Xiong
Bin, Chen Yonggao, Leng Gangsong, Li Shenghong, Su Chun,
Wang Jianwei, Wu Xihuan, Ye Zhonghao, Zhang Zhengjie, Zhu
Huawei.
Mathematical Olympiad in China
100
First Day
8 :00 - 12 :30 January 22, 2005
Let 0, E
(--,2"
2"), i
-
=
1, 2, 3, 4. Prove that there existsx E
R such that the following two inequalities
el C O S ~e2
cos2
e,
C O S ~0, C O S ~
el sin e2
-
(sin
-
(sin 0, sin 0,
>0,
d 2> 0,
-
-
x)2
0
0
hold simultaneously if and only if
(posed by Li Shenghong)
Proof Clearly, 0and 0are equivalent to
sin 01 sin e2 - cos
sin 0, sin 0,
-
el cos 0, < x < sin 0, sin 0, + cos 0, cos e2 ,
@
<x < sin 0, sin 0, + cos 0, cos 0, ,
0
cos 0, cos 0,
respectively. It is easy to show that there exists x E Rsuch that @ and
0hold simultaneously if and only if
sin
el sin 0, + cos el cos 0,
sin 0, sin 0,
+ cos 0, cos 0,
+ cos 0, cos 0, > 0,
sin el sin 0, + cos 0, cos 0, > 0.
-
-
sin 0, sin 0,
On the other hand, using sin2a = 1 -
0
cos2a, we can simplify 0
into
+2c0s el cos e2cos e, cos e, +
sin2el sin2e2 + 2sin el sin e2sin 0, sin 0,
sin20, sin2e, > 0,
cos2el
-
cos2e2
cos2e3 cos2e4
-
or
(cos
el cos 0, + cos 0,
cos 04>2- (sin 0, sin 0,
-
sin e, sin e4)2
That is,
(sin
el sin 0, + cos el cos 0,
-
@
sin 0, sin 0,
+ cos e, cos e,)
> 0.
.
China Mathematical Olympiad 2005
(sin 0, sin 0,
+ cos 0, cos 0,
-
sin
101
el sin 0, + cos el cos 0,)
0. @
If there exists x E R such that @I and 0 hold simultaneously,
then from @ and 0 we can get @ immediately. It follows that 0
holds.
Conversely, if 0holds, or equivalently @ holds, but @ and 0
do not hold, then we have
sin
el sin e2 + cos el cos 0,
-
sin 0, sin 0,
+ cos 0, cos 0, < 0,
-
sin 0, sin 0,
+ cos 0, cos 0, < 0.
and
sin 0, sin 0,
+ cos 0, cos 0,
Adding the last two equations, we obtain
2(c0s
el cos e2 + cos e3 cos e4) < 0,
which contradicts to the fact ei E
(-
2
,
2
i
=
1, 2, 3, 4. So @
and 0hold simultaneously. Hence there existsx E Rsuch that @I and
0hold simultaneously.
@ A circle intersects sides B C , C A , A B of A B C at two points for
each side in the following order: {Dl , D2 } , {El , E2 } and {FI,
F2 } . Line segments D1El and D2E2 intersect at point L , El Fl and
E2D2 intersect at point M,FlDl and F2E2 intersect at point N.
Prove that A L , B M and CN are concurrent. (posed by Ye
Zhonghao)
Proof Through point L draw perpendicular lines to A B and to A C ,
the feet are L’ and L” respectively. Let L L A B = a1 , L L A C = a2 ,
L L F z A = a3 , a n d L L E I A = a4. We have
sin a1
sin a2
~~
-
LL’
L F 2sin a3
LL” = L E sin
~ a4.
0
Draw line segments DlF2 and D2El (see Figure 1). Since A L
DlF2 c/)A L D z E l we get
102
Mathematical Olympiad in China
Figure 1
Figure 2
0
Draw line segments D2F1 and D1E2 (see Figure 2). By using the
sine rule we obtain
sin a3
D2F1
D1 E2 '
0
-- -
sin a4
Substituting 0and 0into 0,
we have
sin al
--
-
sin a2
Similarly, write L B M C
L N C B = Y2 , and we get
.-
DlF2 D2F1
D2El DIE2'
=
PI,
LMBA
0
=
shyl
Yl,
0
.-
@
FIE2 F2El
F2D1 FlD2'
~-
sin Y2
=
.-
sins - ElD2 E2D1
sins E2F1 ElF2'
--
8,LNCA
Multipling 0,
0and @, we obtain
sin al
sin a2
sin y1
.-sinpl
.-s i n 8 sin y2
-
1.
Finally, according to the inverse of Ceva's Theorem, we know A
L, BM and CN have a common point.
China Mathematical Olympiad 2005
a As seen in Figure 3, a circular pool is divided into 2n ( n
103
5)
‘grids’ . Two grids are said to be neighbors if they have a
common side or an arc. It is easy to see that every grid has three
neighbors.
4n 1 frogs jump into the pool. It is difficult for frogs to be
quiet together. Whenever there is a grid where at least three
frogs live together, sooner or later there must be three frogs in
the grid jumping out simultaneously into the three neighboring
grids respectively. Show that after a few times, the distribution
of frogs in the pool will become uniform.
Here “uniform” means, for every grid of the pool, either
the grid itself or each of the three neighboring grids has at least
one frog. (posed by Su Chun)
Proof We call an event that there are three
frogs in the same grid simultaneously jumping
into three different neighboring grids “ an
eruption ”. A grid is said to be “ in
equilibrium” if there are frogs in it or there
are frogs in all its three neighbors.
Figure 3
It is easy to see that a grid will be in
equilibrium after a frog jumps into the grid. Infact, frogs in this grid
never move if there is no eruption. So, the grid is in equilibrium. If
an eruption occurs, then there is at least one frog in each of its three
neighbors. Further, it will be in equilibrium provided no eruption in
each of its three neighbors. However, no matter which neighbor
erupts, there will be frogs jumping into the grid, and there are frogs
in it, so it will remain in equilibrium.
According to the above discussion, it suffices to prove that for
any grid sooner or latter there is a frog jumping into it.
Given any grid, say Grid A. We call the sector in which the grid
is situated Sector 1, and Grid B, for the other grid in Sector 1 (as
shown in Figure 4). We have to prove that sooner or latter there are
frogs jumping into Grid A.
+
104
Mathematical Olympiad in China
Figure 4
Number the remaining sectors 2 to n in the clockwise direction.
At first we show that sooner or latter there are frogs jumping into
Sector 1. Suppose that in Sector 1 there are no frogs coming. Then
there are no frogs crossing the wall between Sector 1 and Sector n.
Consider the sum of the squares of the labeling numbers of sectors
having frogs. Since there is no frog entering Sector 1 (especially,
there is no frog crossing the wall between Sector 1 and Sector n ) , it is
only possible that three frogs jumping from some Sector K (3 K
n- 1) into Sector K - 1, K , and K
1, respectively. So the change in
the sum of the square numbers is
< <
+
(K-l)2
+K2 + ( K + 1 ) 2
-3K2
= 2.
That is increasing by 2. On the one hand, since frogs cannot stop
jumping because there is at least one grid in which there are at least
three frogs, hence the increasing of the sum does not stop. On the
other hand, the sum cannot increase forever (it cannot be greater
than (4n+ l > n 2 >, a contradiction. Therefore, sooner or latter there
are frogs that cross the wall between sector 1 and sector n, and enter
Sector 1.
Next, we prove that sooner or latter there are three frogs
jumping into Sector 1. If there are at most two frogs jumping into
Sector 1, then they do not jumpout , and the above sum of the square
numbers decreases at most twice (it happens only when two frogs
cross the wall between Sector 1 and Sector n). And after that the sum
increases continuously, again a contradiction. Hence, sooner or latter
China Mathematical Olympiad 2005
105
there are three frogs that jump into sector 1.
If among these three frogs some are in Grid A, then there are
already frogs jumping into Grid A. Otherwise, these three frogs are
all in Grid B , thus an eruption happens in Grid B and there is one frog
that jumps into GridA.
Second Day
8:OO-12:30
.-
- Let
January23, 2005
{a,} be a sequence such that al
21 and
=-
16
0
Let m be a positive integer and m
2. Prove that for n
(posed by Zhu Huawei)
Proof By Equation 0,
we have
2%,
=3
Setbn=2"a,,n=l,2,
Sincebl
= 2al =
b,
it follows that
2n--la,-l
3
+T.
.-,then
21
-,
8
+8
3 = 3"-1
(b1+%)=
3",
< m,
Mathematical Olympiad in China
106
Therefore, in order to prove Equation
0,it
suffices to prove
that
or equivalently,
n
m+l'
At first, we estimate the upper bound of 1 - - By using
Bernoulli's inequality, we get
so that
1
n
(1+ilrn
(Note: By the mean inequality, we can also have the same result:
(
n
(1--)
m
m+l
+n 1 "
.1.1.
.... 1
y
-''-
m 1--)+mn-m]
n
m+1
mn
mn
(
Since m
1
2, in view of the binomial formula, we obtain
China Mathematical Olympiad 2005
m
1
m2
5
2
107
1
2m
> -.49
It follows that
or
z_.
1 - - < ( $n)
m+l
112
Hence, if we want to prove Equation
.
8,we only need to prove
that
that is,
Set
($1
112
=t,
then 0 < t < 1 , and Equation 0now becomes
t(m-tm-l) < m - I ,
or
( t - 1 ) [m- (trn-l
+
+ + I ) ] < 0.
The above inequality clearly holds, so does the initial inequality.
$2&%
There are 5 points in a rectangle AB C D (including its boundary)
with unit area such that any three of them are not collinear. Find
1
the minimun number of triangles with areas no more than - and
4
vertexes chosen from these 5 points. (posed by Leng Gangsong)
Solution At first, we give a lemma without proof.
108
Mathematical Olympiad in China
Lemma The area of a triangle inscribed in a rectangle is no more than
half of the area of the rectangle.
In the rectangle A B CD if there are 3 points such that the area of
1
4
the triangle with these points as the vertices is no more than - , then
these 3 points are called a good triple or good.
Denote E , F, H and G the midpoints of A B , CD , BC and AD ,
respectively, and 0 the intersection of line segments EF and GH. E F
and GH divide the rectangle A B C D into 4 small rectangles, it follows
that there exists certain small rectangle, say A E O G , in which there
are at least two points (say, M and N) out of those 5 points, see
Figure 5.
(1) If there is no more than one given
point in the rectangle OHCF, consider any
given point X which is different from M and
B
Nand not in the rectangle OHCF. It is easy A
E
to verify that triple ( M , N, X ) is either in
Figure 5
rectangle A B HG , or in rectangle AEFD. By
the above lemma ( M , N, X ) is good. Since there are at lease two such
points X, we have at least two such good triples.
(2) If there exist at least two given points in rectangle OHCF , we
suppose that P and Q are the given points in rectangle OHCF. Consider
the final given point R. If R is in the rectangle OFDG, then ( M , N ,
R ) is in the rectangle AEFD, and ( P , Q, R ) is in the rectangle
GHCD. So they are all good. It follows that there are at least two
good triples. Similarly, when point R is in the rectangle EBHO there
are at least two good triples too. If point R is in the rectangle OHCF
or the rectangle AECG , suppose that R is in the rectangle OHC F.
Consider the smallest convex polygon containing the 5 points M, N,
P, Q and R. The polygon must be contained in the convex hexagon
AEHCFG (see Figure 6). But the area
1 -1 =3
SAEHCM;= 1- 8
8
4'
China Mathematical Olympiad 2005
109
We divide the case into three subcases as
follows.
i) Suppose the convex polygon generated
by M, N, P, Q and R is a convex pentagon,
A
(no loss of generality) say MNPQR (as seen
in Figure 7). In this case
SMQR
N'
E
B
Figure 6
3
+ SAMNQ+ SANPQ< 7
9
it follows that there is at least one good triple among ( M , Q, R ) , ( M ,
N, Q) and (N,P, Q). Moreover, ( P , Q, R ) is clearly good for it is
in the rectangle OHCF. Thus there are at least two good triples.
R
Figure 7
Figure 8
ii) If the convex polygon generated by M, N, P, Q and R is a
convex quadrilateral, say A1A2A3A4, and the fifth point is A5 (as
seen in Figure 8) where Ai E { M , N,P , Q, R } (i = 1, 2, 3, 4, 5).
Draw line segments A5Ai(i = 1 , 2 , 3 , 4) , then
+
+
+
<3
S ~ A ~ A ~S~A,A,A,
A,
S~A,A,A, SAA,A~A,
= SAA,~,PSA,
-.4
Therefore, there are at least two good triples among (A1 , A2 ,
A5), (A2, A3, A5), (A3, A4, A5) and(&, A4, As).
Figure 9
Figure 10
Mathematical Olympiad in China
110
iii) If the convex polygon generated by M, N, P, Q and R is a
triangle, say aAlA2A3 , and the remaining two points are & and A5
(as seen in Figure 9 > , whereAi E {M, N, P , Q, R } (i = 1, 2, 3, 4,
5). Draw line segmentsA4Ai(i = 1, 2, 3), then
+
+
<3
S ~ A ~ AS~ ~AA~ ~ AS~A,A,A,
~ A ~ = S ~ A ~ A 7.
~A~
Therefore, there is at least one good triple among (Al , A2 , A4) ,
(A2 , A3 , &) and (A1 , A3 , &). Similarly, A5 with two of A1 , A2
andA3 constitutes a good triple. Consequently, in this case there are
at least two good triples.
Thus, in any case there are at least two good triples among these
5 points.
In the following we will give some examples to show that the
number of good triples may be just two. Pick a point Mon the side AD
of the rectangle AB CD and a point N on the side AB such that AN :
NB =AM : M D = 2 : 3 ( as shown in Figure 10). Then among 5
pointsM, N, B, CandD there are just two good triples. In fact, ( B ,
C, D> is clearly not good. Suppose that a triple containing exactly one
of two points M and N, say M. Let E be the midpoint of AD , then
It follows that ( M , B, D> is not good. Thus
Hence ( M , B, C) and ( M , C, D> are not good. If a tripe does
contain two points M and N, then
SAMNC
= 1 - SANBC
- SAMCD- SAAMN
China Mathematical Olympiad 2005
111
1
1
So ( M , N, C) is not good. But SAMNB = SAMND= - < -, thus
5
4
among the triples there are only two good triples ( M , N, B ) and ( M ,
N,0).
Consequently, the minimun number of triangles with area not
1
4
great than - is 2.
*
--*
=B
Find all non-negative integer solutions (x, y ,
following equation
2"
.3Y
-
.
5" 7"
=
z,
w) of the
1.
(posed by Chen Yonggao)
Solution Since 5" 7" 1 is even,we have x 1.
Case 1: y = 0. The equation to be solved becomes
+
2"
>
-
.
5" 7"
=
1.
If z # 0, then 2" = l(mod 5). It follows that 4 I x. Thus 3 I 2"
1, which contradicts to 2" - 5" 7" = 1.
If z = 0, then
2" -7"
=
-
1.
When x = 1, 2 , 3 , a direct computation shows that (x,w) = (1,
0) , (3, 1) are the solutions.
When x 4, 7" =-l(mod 16). By direct computation we know
that this is impossible.
Consequently, when y = 0 all non-negative integer solutions of
the equation are
>
(x,y , z , w) = (1, 0, 0, O), (3, 0, 0, 1).
Case 2 : y > 0 and x = 1. Thus the equation to be solved becomes
Mathematical Olympiad in China
112
2 . 3 ~-5".
Hence-5" *7"=l(mod3),
that z is odd.
Thus
2 3Y
7"
=
1.
i . e . , (-1)"=-l(mod3).
It follows
= l(mod 5).
So y = 1(mod 4).
When w # 0, we have 2 3Y = l(mod 7). Thus y = 4(mod 6),
which contradicts to the fact y = 1(mod 4). Hence w = 0 and
2 3y-5"
=
1.
>
Wheny = 1, we have z = 1. If y
2, then 5" =- l (mo d 9 ),
which implies z = 3(mod 6). Thus 53 1 I 5" 1, so 7 I 5" 1, which
contradicts to 5" 1 = 2 33'. Hence in this case we have only one
solution
+
+
+
+
(x,y , z , w) = (1, 1, 1, 0).
Case 3 : y > 0 and x
5"
7"
=-
> 2. Thus
l(mod 4), and 5"
7"
=-
l(mod 3).
That is,
(-1)"
and(--)"
=-l(mod4),
=-l(mod3).
Thus z and w are odd. It follows that
2 " * 3 Y = 5 " * 7 " +1 = 3 5 +1 = 4 (rno d 8 ).
Hence, x
= 2,
and
4 3Y - 5"
7"
=
1 (where z and w are odd).
Thus,
4 3Y
= l(mod5),
and4 3Y = l(mod7).
From the above two congruencies we have y = 2(mod 12).
S e t y = 12m+2, m>O, then
5"
Since
.7"
=4
.3Y
-
1 = (2 .36"+l
-
1)(2 .36"+l
+ 1).
0
China Mathematical Olympiad 2006
113
0
If m > 1, by Equation 0we have 5" =-1(mod9) , and from Case
2 we know that this is impossible.
If m = 0, then y = 2, z = 1andw= 1. Thus in this case, we have
only one solution
(x,y , z , w) = (2, 2, 1, 1).
Consequently, all non-negative integer solutions are
(x,y , z , w) = (1, 0, 0, O), (3, 0, 0, l ) ,
(1, 1, 1, O), (2, 2, 1, 1).
2006 China Mathematical Olympiad and 21th Mathematics Winter
Camp was held on January 10 - 15 in Fuzhou, Fujian Province, and
was hosted by Olympiad Committee of CMS and Fuzhou No. 1 middle
school.
The Competition Committee consisted of the following: Li
Shenghong, Li Yonggao, Xiong Bin, Leng Gangsong, Li Weigu,
Wang Jianwei, Zhu Huawei, Lin Chang, Luo Wei, Ji Chungang.
First Day
8:OO- 12:30 January 12, 2006
Suppose that the real numbers al , a2 ,
, a,
satisfy al
+a2 +
Mathematical Olympiad in China
114
+a,
= 0.
Prove
-1
maxa!
l<i<n
< 3n C (ai
-
- ai+l
>2.
(posed by Zhu Huawei)
;=I
Solution It is sufficient to prove that: For every K E { 1 , 2 , .-, n} ,
we have
i= 1
i=l
I 1
<[ X i 2 [ C &
-1
i=l
-1
i=l
i= 1
=
n(n
-
1) (2n -1)
6
i= 1
China Mathematical Olympiad 2006
&
%
.
*
s
Suppose that positive integers al , a 2 ,
..a,
115
a2 006 (some of them
may be equal) satisfy the condition: any two of
a2 005
~
-
a1
, a2 ,
a2
a3
-
,
are unequal. At least how many different numbers are
a2 006
there in { a1 , a2 , , a2 006 } ? (posed by Chen Yonggao)
Solution The answer is: there are at least 46 different numbers in
a29
'"9
a2006).
+
With 45 different positive integers we can only get 45 X 44 1 =
1 981 fractions. So there are more than 45 different numbers in { al ,
a29
"')
a2006).
On the other hand, let p l
a1 9 a29 "*, a2006 to be:
, p2 , , p46 be 46 different prime.
Then the 2 006 positive numbers satisfy that any two of
a1
~
, a2 , ,
- a2
a2 005
Set
a3
are unequal.
a2 006
So the answer is 46.
++
Suppose positive integers m, n , K satisfy mn = K2 K 3. Prove
that at least one of the following Diophantine equations
2 + 1 1 9 = 4m and xz + 11y2 = 4n
has a solution (x,y ) with x, y being odd numbers. (posed by Li
Weigu)
Solution First, we prove a lemma.
Mathematical Olympiad in China
116
Lemma The following Diophantine equation
0
2+11$= 4m
has a solution (XO , yo) , such that either xo, yo are odd numbers or
xo, yo are even numbers with xo = (2K l)yo (modm).
0
Proof of the lemma: Consider the expression x+ (2K+l)y, where
+
&
x, yareintegers, a n d O < x < 2 & , O < y < - .
There are ([2
f i ]+ 1)
E [O, 2 &I
, x2
that(x1,
yz),and
yl)f(x2,
XI
1
6+ 1 > m such expressions. So
“ 2 1
there exist integers XI
, y1 , y2
+ (2K + l ) y l = + (2K +
where
x2
x
= XI - x 2
2
9
y
= y2
I]
E [0, &
l)y2 (mod m)
-
, such
,
y1.
This means
2 = (2K + I ) ~ $=- l l y 2(mod m) ,
2 + y 2 = tm.
that is,
2 + 1ly2 < 4m+-m11 < 7m.
4
so
1<t<6.
+
A m is an odd number, obviously the equations 2 113 = 2m,
and 2 l l y 2 = 6m have no integer solution.
(1) If 2 l l y 2 = m, thenxo = 2x, yo = 2y is a solution of 0
satisfying 0.
(2) If 2 l l y 2 = 4m, then xo = x, yo = y is a solution of 0
+
+
+
China Mathematical Olympiad 2006
117
satisfying 0.
(3) If 2 119 = 3m, then (x& 1 1 ~ ) 11(xT
~
y>2 = 32 4m.
First, assume that3 /m. Ifx+O(mod3), y+O(mod3), andx+
y(mod 3) , then
+
+
xo
=
x- l l y
~
3
X+Y
9
Y
o
=
0
T
is a solution of 0satisfying 0.
If x = y O(mod 3 ), then
+
is a solution of 0satisfying 0.
Now suppose 3/m. Then 0 and 0 are still integer solutions of
0.If equation 0has an even integer solution xo = 2x1 , yo = 2yl ,
then
$
+ 1ly:
=m W
36m = (5x1 & l l y l > 2
+ 11(5y1 7
XI >2.
Since one of XI , y1 is even and the other is odd, so 5x1 & l l y l ,
5yl Txl are odd numbers.
5x1 - l l y l
5 ~ +1X I is a
If x = y(mod 3), then xo =
9 Yo =
3
3
solution of 0satisfying 0.
5x1 +11y1
5 ~ -1X I is a
If XI
y1 (mod 3) , then xo =
9 Yo =
3
3
solution of 0satisfying 0.
(4) If 2 + l l y 2 = 5m, then 52 4m = ( 3 x 7 1 1 ~ +) 1~ 1 ( 3 y & ~ ) ~ .
When 5 1 m, if
+
x =& l(mod 5) , y =T 2(mod 5) ,
or x =& 2(mod 5) , y =& l(mod 5) ,
then
xo
=
3~ - l l y
3Y+X
, Y O = T
5
@
Mathematical Olympiad in China
118
is a solution of 0satisfying 0.
If x =& l(mod 5), y =& 2(mod 5) , or x =& 2(mod 5) , y =T
1(mod 5) then
xo
=
3x+ l l y
3y-x
9 y 0 = 5
5
0
is a solution of 0satisfying 0.
When 5/m, then @and 0are still are integer solutions of 0.
If
Equation 0has an even integer solution xo = 2x1 , yo = 2yl , then
d + lly:
= m, XI
+ y1 (mod 2) ,
+
and we have lOOm = (XI T 33y1> 2 ll(y1 & 3x1>2.
Ifx1 = y l =O(mod5), orxl =&l(mod5), y1 =&2(mod5), or
XI
thenxo
XI
=
=& 2(mod 5), y1 =T l(mod 5)
-33Y1
,Yo=
5
YI
+3x1
is a solution of
0satisfying 0.
is a solution of 0satisfying 0.
The lemma is proved.
From the lemma, if 0has a solution (x, y) with x, y being odd
numbers, then it has a solution (xo , yo) with xo , yo being even
numbers satifying 0.
Let I = 2K 1, the solution of the quadratic equation
+
mx2+Zyox+nyi-1
-1~o+JFyi!-4mnyi!+4m
2m
has at least an integer solution XI , i. e.
1s x =
-
=o.
@
- ~ y ~ & x ~
. So Equation @
2m
China Mathematical Olympiad 2006
mxf
+ ~ y o x l +ny8
-
119
8
1 = 0.
This indicates that x1 is an odd number. Now, from 8,
it follows
that
(2nyO
+
+
1x1> 2
+ IIZ+
= 4n.
This means 2 1 1 9 = 4n has a solution (x,y> with x, y being
odd numbers, where x = 2ny0 Zxl y = XI.
+
Second Day
8:OO- 12:30 January 13, 2006
.=.s3 Let a A B C
be a right-angled triangle with L A C B
=
90". The
inscribed circle OOof A A B C is tangent to BC, C A , AB at D , E ,
Frespectively. AD intersects @Oat P, L B P C = 90". Prove that
A E A P = PD. (posed by Xiong Bin)
Proof Let A E = AF = x, BD
+
= B F = y , C D = C E = z , A P &AE
= m, PD = n.
_--- B
SinceLACP + L P C B = " \ D L
___--___--\,
90" = L P B C L P C B , then
LACP = LPBC.
Let Q be a point on the line AD, such that LAQC = LACP =
L P B C. Then P , B , Q , C are concyclic.
Let OQ = I , then by the intersecting chord theorem and the
tangent secant theorem, it follows that
_/---
\
+
\
\ \
;----
yz
=
nl,
2 = m(m+n>.
As A A C P
c/)
AC AP
A A Q C , we have - = -. So
A Q AC
( ~ + z 2 )=
~ m(m+n+I>.
0
In Rt A A C D and Rt A A C B , using the Pythagorean theorem, we
Mathematical Olympiad in China
120
get
( x + + ) +2
~
= (rn+d2,
(y+z)2
0-0:
+(z+x)2
2 +2zx
=
(x+yY)2.
= ml,
0
@X
0
together with 0, we get 2 + 2 Y z
z2 + 2 z x
=
(m+d2 = ( x
+ z > ~+2,i.e.
2y
+2x
--
z
From
0,
x+z
-
2z(x+z).
2XY
=-
8
y +z'
From @ and 8,
we have
X
-
z+2x
Also from 0,
y+z=
Y
+
z,
y+z'
2xz
X-z
~
42
. Substituting into @ and eliminating
we abtain
3 2 -2xz-22
giving the solution x = .J?+1 z , y
3
implies
~
m+n
=
=
= 0,
(22/?+5)z. Together with
2(47+ 1)
3
z.
0,
it
China Mathematical Olympiad
From 0,
m=
AE+AP
=
2
-
.J?+l
z,
-- -
6
m+n
n=-
121
2006
.J?+l z.
2
So x + m
= n,
i. e.
PD.
@ A sequence of real numbers {a,} satisfies the condition that a1
1
1
, K = 1, 2 ,
-9
ak+l
+-2
2
-
=
..a.
ak
Prove the following inequality:
n
n
(posed by Li Shenghong)
Proof First, weuseinduction toprovethato<%<-,
1
2
n = l , 2,
..a.
When n = 1, it is obvious.
Nowsupposeitistrueforn ( n > l > , i . e . O<a,<-.
Let f ( x > =-x+-
1
function. So
1
>f
-
1
6
=-
>o,
i. e. it is true for n+ 1.
Back to the problem, it is sufficient to prove:
n
( a , +a2 +...+a,
2
. Then f ( x >is a decreasing
2-x
an+l =f(a,>
1
n
2(a1 +a2
+ ...+a,>
Mathematical Olympiad in China
122
Let f<x) = In
(:
--
( 4 .)
1
1 for x E 0
function, i. e. for every 0
1
< x l , x2 < -,
2
Then f<x)is a concave
we have
f[e]
<
2
In fact,
is equivalent to
w
(XI -x22>2
> 0.
So f<x>is a concave function.
Using Jenson’s Inequality, It follows that
x1 +x2
+.-+x,
f(x1) + f ( x 2 )
n
+-.+f(x,)
9
n
so
(a1 +a2
+ ...+a,
On the other hand, by using Cauchy’s Inequality, we have
i=l
China Mathematical Olympiad 2006
i=l
123
i=l
so
lIn
Let X be a set of 56 elements. Find the least positive integer n
such that for any 15 subsets of X, if the union of every 7 sets of
these subsets contains at least n elements, then there exist 3 of the
15 subsets whose intersection is nonempty. (posed by Leng
Gangsong)
Solution The minimum value of n is 41.
First, we prove that n = 41 satisfies the condition.
Suppose there exist 15 subsets of X, the union of every 7 of these
15 subsets contains at least 41 elements, and there exists no 3 of the 15
subsets whose intersection is nonempty .
Since every element of X can only belong 2 of the 15 subsets, we
can suppose that every element of X belongs to exactly 2 of the 15
subsets. Otherwise we can add a few more elements to some sets of
the 15 subsets, and the condition still holds.
By the Pigeonhole Principle, there is a set of the 15 subsets
(suppose it is A) , such that I A
I
+I"!'[
>
15
sets be A1 , A,, .-, A14.
The union of every 7 sets of A1 , A2 ,
elements. So the total number y 41CT4.
<
1 = 8. Let the other 14
, A14 contains at least 41
Mathematical Olympiad in China
124
We use another way to compute the value of y.
For a E X, if a @ A, then a belongs to exactly 2 sets of A1 ,
, , A14.
So a is counted C74 - C72 times. If a E A, then a belongs
A14. So a is counted CT4 - CT3 times. It
to only one set of A1 , A2 ,
follows that
A2
..a,
41c74<~<(56-1A
I)(C74-C72)+IA
=56(C74
-
C72) - I A I (C73 - C72)
<56(C74
-
C72) - S(C73 - C72) 9
I
(C74-C73)
<
that is, 196 195, a contradiction.
Next, we prove n 41.
I f n < 4 0 , supposeX= (1, 2, .-, 56). let
Ai
=
{i, i + 7 , i+14, i+21, i+28, i+35, i+42, i + 4 9 ) ,
i =1, 2,
..a,
7,
Bj = { j , j + 8 , j + 1 6 , j + 2 4 , j + 3 2 , j + 4 0 , j + 4 8 ) ,
j =1, 2,
.a*,
8.
It is easy to see that
7), IAi n A j I = O ( l < i < j < 7 ) ,
IAi1-8 ( i = l , 2,
..*)
I B j I = 7 ( j = i , 2,
-., s),
IAi
n Bjl=
nBjI=o(i<i<j<8),
1(l<i<7,
l<j<S).
For every 3 of the 15 subsets, there are 2 sets both being Ai, or
both being Bj. So the intersection of the 3 sets is empty.
But for every 7 of the 15 subsets, for example,
Ai1 Ai2
Ai
Bj, Bj2
Bj ( s + t
t
= 7),
we have
IAil U Ai2 U
= Mi1
I+
Mi2
I+-.+
U Ais U Bj, U Bj2 U
1 % I+
lBjl
I+
U Bj, I
lBj2I+-.+
lBjt I--t
China Mathematical Olympiad 2006
=8
~ 7t
+ - st
=8
~ 7(7
+ - S>
= (~-33)~+40>40.
>
So n 41.
There fore the answer is 41.
-
~ ( 7 - S>
125
China Girls’ Mathematical
Olympiad
T h e China Girls’ Mathematical Olympiad, organized by the China
Mathematical Olympiad Committee, is held in August every year.
Participants from Russia, Philippine, Hong Kong, Macau and the
mainland are invited to take part in the competition. The
competition lasts for 2 days, and there are 4 problems to be
completed within 4 hours each day.
2002
(zhl-i,Guarrgdong)
The 1st China Girls’ Mathematical Olympiad was held on August 15 19, 2002 in Zhuhai, Guangdong Province.
The Competition Committee consisted of the following: Pan
China Girls’ Mathematical Olympiad
2002
127
Chengbiao , Qiu Zonghu, Li Shenghong, Xiong Bin, Wu Weichao ,
Qian Zhanwang and Qi Jianxin.
First Day
8:OO- 12:OO August 16, 2002
+
+
@@@Find all positive integers n such that 20n 2 can divide 2 003n
2 002. (posed by Wu Weichao)
Solution It is easy to see that n is an even number. Let n = 2m,
then from
40m+ 2 12 003 X 2m+2 002
we can get
But
20m
+ 1I 2 003m + 1001.
2003m+1001
so
=
100(20m+l)+3m+901,
20m+1 I3m+901.
And when
+
3m 901
20m+1
=
1, 2, 3 , 4, m is not a positive integer.
Therefore ,
+
+
3m 901
2 5,
20m 1
896
m<-<10.
97
<
Hence m 9. But after checking one by one form = 1, 2, 3 , ,
9, we know that20m+1[ 3m+901 form = 1, 2, 3,
9.
Therefore, there is no positive integer n satisfying the condition.
..a,
@& 3 n ( n is a positive integer) girl students took part in a summer
camp. There were three girl students to be on duty every day.
When the summer camp ended, it was found that any two of the
3 n girl students had just one time to be on duty on the same day.
(1 ) When n = 3 , is there any arrangement satisfying the
requirement above? Prove your conclusion.
Mathematical Olympiad in China
128
(2) Prove that rz is an odd number. (posed by Liu Jiangfeng)
Solution (1) When n = 3, there is an arrangement satisfying the
requirement. Now the concrete arrangement is given as follows
(Denote the nine girl students by 1, 2 , , 9) :
(2) We take arbitrarily one girl student. Since she is on duty just
one time with every one of other girl students, and three people are
on duty every day. So all other girl students can be paired up two by
two. Hence
2 I3n- 1,
therefore rz is an odd number.
Remark Problem (1) is an open-ended problem. It asks whether
there is an arrangement satisfying the requirement of the problem. If
an arrangements exists, we must construct a concrete example (see the
example we have given). If not, we will prove it. ( I t is usually
proved by contradiction. )
@ $ Find all positive integers K such that , for any positive numbers a ,
b and c satisfying the inequality K ( a b bc ca 1> 5(a2 b2
2),there must exist a triangle with a , b and c as the length of
+ +
+ +
its three sides respectively. (posed by Qian Zhangwang)
Analysis First, we try to determine the range of K. Then by
assumption that K is a positive integer, we may get the values for K.
Afterward, we check that K can take really these values.
Solution From(a-b)2+(b-cc)2+(c-ua)2
2.0,weget
a 2 + b 2 + 2 >ub+bc+ca,
so K
> 5.
>
Hence K 6.
Since there is no triangle with length of its sides to be 1, 1, 2
China Girls' Mathematical Olympiad 2002
129
respectively, by the assumption in the problem, we have
K(lxl+lx2+lx2~<5~l~+l~+2~~,
<
that is, K
6.
We will prove that K = 6 satisfies the requirement below. There is
no harm in assuming a b c.
Since
so
<<
6 ( a b + bc + c a ) > 5(a2 + b2 + c?) ,
52 -6(a+b)c+5a2
A = [6(a+b)12 -4
+5b2 - 6 ~ b< O ,
5(5a2 +5@ - 6 d )
=64[ab- ( ~ - b ) ~ ]
< 64 ( a +4 b I 2
=16(a + bI2.
<64ab
=a+b.
Hence, there exists a triangle with a , b , and c being the length
of three sides.
Circles 0 1 and 0 2 interest at two points B and C , and BC is the
diameter of circle 0 1 . Construct a tangent line of circle 0 1 at C
and interesting circle O2 at another point A . We join A B to
intersect circle 01 at point E , then join C E and extend it to
intersect circle 0 2 at point F . Assume H is an arbitrary point on
line segment A F . We join H E and extend it to intersect circle
01 at point G , and then join B G and extend it to intersect the
extend line of A C at point D. Prove:
AH
HF
-
AC
CD'
(posed by Xiong Bin)
130
Mathematical Olympiad in China
Analysis To prove
AH
HF
~
=
AC
AH
,
we need only to prove
CD
AF
~
~
=
AC
AD'
~
that is,
A H * A D = AC O A F .
First, we try to prove M H E c/)a A B D 1.
and then using the tangent secant theorem,
we can prove the result.
Proof Since B C is the diameter of 00 1
and ACD is a tangent line, so BC 1A D , L A C B
the diameter of 00 2 .
Also, L B E C = 90", that is, A B 1F C , so
= 90".
Hence AB is
LFAB =/CAB.
Join CG, then CG 1BD. Hence
LADB
therefore
Thus
that is,
=LBCG
M H E
=/BEG
=L
AEH,
MBD.
c/)
AH
AB
AE =AD'
AH*AD=AB*AE.
By the tangent secant theorem, we can get
AC2 = A E * A B ,
A H * A D = AC2 = AC O A F ,
so
AH
AC
AF =AD'
We can get
AH
HF
-
AC
CD'
Second Day
8:OO- 12:OO August 17, 2002
@@
! Assume PI , P2 , .-, P, ( n
2) is an arbitrary permutation for
China Girls' Mathematical Olympiad
1, 2,
..a,
131
n. Prove that
1
PI
2002
+ ...+
+p3
1
+p2 ' p 2
1
Pn-2
+
pn-1
+
1
pn-1+
n-1
>pn
n + 2'
(posed by Qiu Zonghu)
Proof By Cauchy's inequality, we can get
[(PI +P2)+
(P2 +P3)+"'+
1
Therefore
PI
(Pn-I
+ ...+
+p3
1
+p2 ' p 2
+Pi%)]*
1
pn-1
+pn
Find all pairs of positive integers ( x , y ) satisfying xY = y-y.
(posed by Pan Chengbiao)
Solution If x = 1, theny = 1. If y = 1, thenx = 1.
Ifx=y, thenxY=l,sox=y=l.
We will discuss the circumstances when x > y 2 2 below. By
assumption
$$@@
Y
1<(5)= y x - - 2 Y ,
so
x
> 2y, and y I x.
Mathematical Olympiad in China
132
Assume x = ky then k
> 3, and
kY
= y<k-2>Y.
k =ykP2.
Therefore
>
>
Since y
2 , so 9”
2k-2. By mathematical induction or the
binomial theorem, it is easy to prove that when k 5 , 2’ >4k. Hence
k can only be 3 or 4.
When x = 9, y = 3 , we have k = 3. When y = 2, x = 8, we have
k = 4.
Therefore, all pairs of positive integers to be found are (1 , 1) ,
(9, 3) and (8, 2).
>
@@& An acute triangle A B C has three heights A D , B E and CF
respectively. Prove that the perimeter of triangle D E F is not
over half of the perimeter of triangle
ABC. (posed by Qi Jianxin)
Analysis To prove
DE
+ EF +DF < 1 ( A B +BC + C A ) ,
B
we need only to prove
n
c
+DF < B C ,
EF + DF <A B ,
EF + DE <AC.
DE
Proof Since L A D B = L A E B = 90°, so four points A , B , D and E
are concyclic, and furthermore, A B is the diameter. Hence, by the
sine rule, we can get
DE
=AB=c,
sin L D A E
so
DE
=
csinLDAE.
In addition, L D A C + L D C A
= 90°,
therefore,
China Girls’ Mathematical Olympiad
DE
Similarly, we can get DF
Therefore ,
=
2002
133
ccos C.
= b cos B.
DE+DF
=
ccosC+bcosB
= (2RsinC)cosC+
(2RsinB)cosB
+ sin 2B)
=2Rsin ( B + C)cos ( B
= R (sin 2C
-
C)
=2RsinA cos ( B - C)
=acos(B--)<a,
where R is the radius of the circumcircle of A A B C .
DE +DF
a.
That is,
<
Similarly,
Therefore,
< b and EF +DF < c.
1
DE +DF +EF < 2 ( a +b+ c ) .
DE +EF
Assume that A l , A 2 ,
A8 are eight points taken arbitrarily
on a plane. For a directed line 1 taken arbitrarily on the plane,
assume that projections of A l , A2 , .-, A8 on the line are P I ,
P2,
P8 respectively. If the eight projections are pairwise
disjoint, they can be arranged as Pil , Pi , , Pi8 according to
..a,
..a,
the direction of line 1 . Thus we get one permutation for 1,
2,
8, namely, i l , i 2 , .-, is. In the figure, this
permutation is 2, 1, 8, 3, 7, 4, 6, 5. Assume that after these
eight points are projected to every directed line on the plane, we
get the number of different
permutations as N8 = N(A1 ,
A2 , , A8). Find the maximal
value of N 8 . (posed by Su
Chun)
Solution (1) For two parallel and
..a,
Mathematical Olympiad in China
134
directed lines with the same direction, the order of projections of A1 ,
A2, , A8 must be the same. So, we need only to discuss all directed
lines passing through a fixed point 0.
(2) If a directed line taken is perpendicular to a line joining two
given points, then the projections of these two points must coincide,
and a corresponding permutation will not be produced. When the
directed lines taken are not perpendicular to any line joining two given
points, any two projections of A1 , A2 ,
A8 must not coincide.
Hence there is a corresponding permutation.
(3) Suppose that the number of lines through point 0 and
perpendicular to a line joining two given points is K. Then K
Ci =
28. Then there arise 2K directed lines placed anticlockwise. Assume
12k. For an arbitrary directed line
that they are in order of I1 , 12 ,
1 (different to I1 ,
I2k ) , there must be two consecutive directed
lines Zj and Z j + 1 such that Z j , Z , Zj+ 1 are placed anticlockwise. It is
obvious that for given j , the corresponding permutations obtained
from such Z must be the same.
(4) For any two directed lines 1 and I’ different from I1 , , 12k ,
if we cannot find j such that both Zj , Z , Zj+ 1 and Zj , I’ , Zj+l satisfy
(3). Then there must be j so that 1’, Z j , I are placed anticlockwise.
Assume that Z j is perpendicular to the line joining Aj, and Aj , it is
..a,
<
..a,
..a,
obvious that the orders of the projections of points Aj, and Aj on the
directed lines Z and I’ must be different, so the corresponding
permutations must also be different.
(5) It follows from (3) and (4) that the number of different
permutations is 2K. Note that K = is obtainable, so N8 = 56.
2003
(Wuhan, Hubei)
The 2nd China Girls’ Mathematical Olympiad was held on August 25 -
China Girls' Mathematical Olympiad
2003
135
29 , 2003 in Wuhan, Hubei Province, and was hosted jointly by CMO
Committee, Wuhan Education Bureau, Wuhan Iron and Steel
Company Limited (WISCO) , and the No. 3 Middle School of WISCO.
The Competition Committee consisted of the following: Chen
Yonggao, Qian Zhanwang, Li Shenghong, Xiong Bin, Leng
Gangsong, Li Weigu, Feng Zuming, and Zhang Zhenjie.
First Day
8 :30 - 12:30 August 27, 2003
@BLet ABC be a triangle. Points D and E are on sides A B and A C ,
respectively, and point F is on line segment DE. Let
AE
DF
- = y , - = z. Prove that
AC
DE
AD
AB
~
=
x,
136
Mathematical Olympiad in China
-=
There are 47 students in a classroom with seats arranged in
6 rows X 8 columns, and the seat in the i-th row and j-th
column is denoted by ( i , j ) . Now, an adjustment is made for
students’ seats in the new school term. For a student with the
original seat ( i , j ) , if his/her new seat is ( m , n ) , we say that
the student is moved by [a, b] = [i - m , j - n] and define the
position value of the student asa+b. Let S denote the sum of the
position values of all the students. Determine the difference
between the greatest and smallest possible values of S. (posed by
Chen Yonggao)
Solution Add a virtual student A so that every seat is occupied by
exactly one student. Denote S’ the sum of the position values in this
situation. Notice that an exchange of two students occupying the
adjacent seats will not change the value of S’. Every student can
return to his/her original seat by a finite number of such exchanges of
adjacent students. Then S’ = 0. Since S’ = S U A b~ , where U A
b~ is the position value of student A , then we have S is the greatest
when student A occupies seat (1, 1) , and S is the smallest when A
occupies seat (6, 8). So the difference between the greatest and the
smallest possible values of S is 14.
+ +
+
@& As shown in the figure, quadrilateral A B C D is inscribed in a
circle with AC as its diameter , BD 1AC , and E the intersection
of AC and BD. Extend line segment DA and BA through A to F
and G respectively, such that DG //
G
BF. Extend GF to H such that CH 1
H
GH. Prove that points B, E, F and H
lie on one circle. ( posed by Liu
Jiangfeng)
B
Solution As shown in the figure, connect
BH, EF and CG. S i n c e A B A F w A G A D ,
we have
C
China Girls' Mathematical Olympiad
FA
AB
2003
137
DA
- AG'
0
~-~
Furthermore, L U B E
A A C D , then
c/)
AB
AC
-
0
EA-DA'
Multiplying 0 by
0,
we get
FA then A F A E c/)A C A G
EA - AG
a s L F A E = L C A G , and thusLFEA = L C G A .
As is known that L C B G = L C H G = 90", then points B , C , G
and H lie on one circle. So,
LBHF
+L B E F
~
~
+90" +L B E F
= / B E + 90" + L B E F
=LFEA + 90" +L B E F
=LBHC
=
180".
That means that points B , E, F and H lie on one circle.
@@@ (1) Prove that there exist five nonnegative real numbers a , b ,
c , d and e with their sum equal to 1 such that for any
arrangement of these numbers around a circle, there are
always two neighboring numbers with their product not less
1
than -.
9
(2) Prove that for any five nonnegative real numbers with their
sum equal to 1, it is always possible to arrange them around a
circle such that there are two neighboring numbers with their
1
product not greater than -. (posed by Qian Zhanwang)
9
1
b = c = -, d = e = 0, it is easy to see that,
3
when arrange them around a circle, we can always get two
Solution (1) Let a
=
1
and their product is -.
9
(2) For any five nonnegative real numbers a , b , c , d and e
neighboring numbers of
1
-,
3
Mathematical Olympiad in China
138
with their sum equal to 1, without loss of
generality , we assume that a b c d
e
0. Arrange these numbers around a
circle in such a way as seen in the figure, d
we are going to prove that this arrangement
meet the condition.
Sincea+b+c+d+e= 1, we havea+
3 d < 1 , and
u*3d<(-) a + 3 d < T1, thenad<-. 1
12
>>> >
>
Furthermore, a + b + c < l ,
thenb+c<l-u<l--,
<1
2
i.e.
b+c
2
< <
<
-.9 Since ce a e ad and bd bc ,
Sobc< (b+ c)2
4
then any neighboring numbers in this arrangement have their product
b+c<j-.
1
less than -.
9
Second Day
8:30-12:30August 28, 2003
*s=
zz=Let {a,}T be a sequence of real numbers such that al = 2,
a~-u,+l,forn=l,2,
1+
-
a2
...+
Provethatl-
=
1
1
<-+
2 0032003 ai
< 1. (posed by Li Shenghong)
-
a2 003
Solution We have adl
1
a,-1
..a.
adl
l . so,
a,
-
1 = a, (a,
-
1) , and then
1
aTtt1-1
-
-
China Girls' Mathematical Olympiad
2003
139
It is easy to see that { a, } is strictly monotonic increasing, so
a2004 > 1, and that means
1 +-1
a1
a2
1
+ ...+ < 1.
a2 003
In order to prove the left inequality, we only need to prove that
a2004
-
1
> 2 0032003.
By induction, we have a,tl
a1 >n"(n
+ 1 and
*a1
=
> 1).
This completes the proof.
>
Let n 2 be an integer. Find the largest real number A such that
the inequality
holds for any positive integers a1 , a2,
a2
< a,. (posed by Leng Gangsong)
..a,
a , satisfying a1
<
<..a
Solution F o r a i = i , i = l , 2 ,
..a,
n,wehaveA<(n-2)+-=
n-1
2
2n-4
n--3
Now, we are going to prove that a;
> 2n-4
n-1
~
a,-l) +2a, for any positive integers a l , a2 ,
a,.
a2
< ...<
Sinceak<a,-(n--R),
K=l,
2,
..a,
n-1,
(a1
, a,
a,>n,
+ + ... +
a2
satisfying a1 <
thenwehave
Mathematical Olympiad in China
140
That is, a,2 >,
2 n--l
n - 4 (al
+ a2 + + a,-l) + 2a,.
So the largest
2n- 4
value of h is n-1'
Let the sides of a scalene triangle A A B C be A B = c , BC = a ,
C A =b, D , E , F be points on B C , C A , A B , such that A D ,
B E , CF are angle bisectors of the triangle, respectively. Assume
that DE = DF. Prove that
pi,
a
b
C
(1)b+c
c+a
afb'
(2) L s A C > 90".
-+-*
I '\
I
(posed by Xiong Bin)
B
c
D
Solution Using the sine rule, we have that
sinLAFD
sinLFAD
-
AD
FD
-
AD
ED
-
sinLAED
sinLFAD'
then sin L A F D = sin L A E D. So, either L A FD = L A E D or
L A F D + L A E D = 180".
If L A F D = L A E D , then a A D F Z a A D E , and we get A F =
A E . Then M F Z M E , and L A F I = L A E I . So a A F C Z
U E B , then AC = A B . It contradicts the condition given. So
L A F D + L A E D = 180", and points A , F, D and E lie on one
circle.
Then L D E C = L D F A > L A B C . Extend C A through A to point
P such that
LDPC
=L
0
B , thenPC = PE+CE.
Since L B F D
=
L P E D and FD
A P E D , then PE
=
BF
=
E D , we have that A B F D Z
ac
Furthermore, APC D
a +b'
=-
c/)
ABCA,
PC
CD So
then - = -.
BC C A
P C = a * - ba
b+c
1
.-=b
b+c'
U2
0
China Girls' Mathematical Olympiad 2003
From
a2
0and @ we get-b+c
=
ac
-+*
a+b
c+a'
141
a
b+c
then-
-
-+
b
c+a
C
-
a +b'
This completes the proof of (1).
As to the proof of (2) , we have from (1) that
a(a
+b) ( a + c )
a2(a+b+c)
= b(b+a)
=
+c(c+
a ) ( c + b)
@(a+b+c) +2(a+b+c)
>@ ( a + b+
Then a2 > @
(b+ c )
c)
+ 2 ( a +b+
+abc
c).
+ 2 , and that means L B A C > 90".
Let rz be a positive integer, and S, be the set of all positive
integer divisors of rz (including 1 and itself). Prove that at most
half of the elements in S, have their last digits equal to 3. (posed
by Feng Zuming)
Solution We are going to consider the following three cases.
(1) If 5 I r z , let d l , d 2 ,
d, be the elements in S, with their
last digits equal to 3, then 5 d 1 , 5d2 , .-, 5 d , are elements in S,
1
with their last digits equal to 5 . So m - I S, I . The statement is true
2
in this case.
(2) If 5 [ rz and the last digit of every prime divisor of rz is either
1 or 9, the last digit of any element in S, is either 1 or 9. The
statement is also true in this case.
(3) If 5 1 rz and there exists a prime divisor p in S, such that the
last digit of p is either 3 or 7. Let n = p'q , where q and I are positive
integers and p is prime to q , and let S, = { a1 , a2 , , ak } be the set
of all positive integer divisors of q . Then the elements in S, can be
written in the following way:
..a,
<
a l , a l p , a l p 2 , .-, a l p ' ,
a2,
a z p , a2p2,
.-, a 2 p r ,
.................................
ah, a k p , a k p 2 , ..., akp'.
Mathematical Olympiad in China
142
= a s p z E S,
, we choose ei =
jaspz+’
I<r,
thenei E
I = r,
S, and we call ei the partner of di . If the last digit of di is 3 , then
that of its partner ei is not, since that of p is either 3 or 7. If di and
dj in S, are different, and their last digits are both 3, then their
partners ei and ej are also different. Otherwise, suppose ei =
For any di
ej =aspz , we may assume that { di , di }
(aspz-’
=
{aspz-’
, aspz+’ } , then
di = d j p 2 . As the last digit of p is 3 or 7, then that of p 2 is always 9,
and that means the last digits of di and dj cannot be the same. It
leads to a contradiction.
We then see that every di E S, with its last digit equal to 3 has a
partner ei E S, with its last digit not equal to 3 , and different di has
different partner. That means that at most half of the elements in S,
have their last digits equal to 3. This completes the proof.
The 3rd China Girls’ Mathematical Olympiad was held on August 10 15, 2004 in Nanchang, Jiangxi Province, China, and was hosted
jointly by CMO Committee, Jiangxi Mathematical Society and
Nanchang No. 2 middle school.
The Competition Committee consisted of the following: Tao
Pingsheng, Su Chun, Chen Yonggao, Xiong Bin, Li Shenghong, Li
Weigu, Shi qiyan, Yuan Hanhui and Feng Zuming.
First Day
8:OO- 12:OO August 12, 2004
@@@We say a positive integer rz is “good” if there is a permutation
China Girls' Mathematical Olympiad
2004
143
a,) of 1, 2,
rz such thatak+Kis a perfect
n. Determine all the good numbers in the
square for all 1 K
( a l , a2,
..a,
..a,
< <
set { 11, 13, 15, 17, 19). (posed by Su Chun)
Solution The good numbers are 13, 15, 17 and 19. However 11 is
not.
11, 4 K is a perfect square if and only if
Note that for 1 K
K = 5. Likewise, 11+ K is a perfect square if and only if K = 5. Hence
11 is not good.
Note that 13 is good because
< <
K:
ah:
+
1 2 3 4 5 6 7 8 9 1 0 1 1 1 2 1 3 ,
8 2 13 12 11 10 9 1 7 6 5 4 3.
Similarly, 15, 17 and 19 are good because
K : 1 2 3 4 5 6 7 8 9 10 11 12 13 14 15,
ah:
15 14 13 12 11 10 9 8 7 6 5 4 3 2
1;
K : 1 2 3 4 5 6 7 8 9 10 11 12 13 14 15 16 17,
ah:
3 7 6 5 4
10 2 17 16 15 14 13 12 11 1 9 8;
K : 1 2 3 4 5 6 7 8 9 10 11 12 13 14 15 16 17 18 19,
ah:
8 7 6 5 4 3 2 1 16 15 14 13 12 11 10 9
19 18 17.
&
%
p
%
& Let a , b and c be positive real numbers. Determine the
minimum value of
a+3c
4b
a+2b+c+a+b+2c
-
Solution The answer is 12&-
I;I::r :%1::
x
8c
a +b+3c'
(posed by Li Shenghong)
17. Set
= a+2b+c,
..
It is easy to see thatz-y = c a n d x - y = b-c, givingx-y =
b-h-y),
orb=x+z-2y.
Wenote t h a ta+3c=2y-x.
By the
AM-GM Inequality, it follows that
Mathematical Olympiad in China
144
a+3c
4b
a+2b+c+a+b+2c
-
-17+2
-
8c
a+b+3c
-Y+ 4 - + 4X- + 8 z
X
Y
Y
2- 17 + 2 & + 2 & T
The equality holds if and only if
=-
2y
-
Y
-
z
17
+1 2 a .
4x
42
= - and - =
"
Y
Y
2y2 = 2.Hence the equality holds if and only if
+
a b+ 2c
a+b+3c
= JZ(a
+2b+
C)
8y
-
z
, or 4
2
=
,
= 2(a+2b+c).
Solving the above system of equations for b and c in terms of a
gives
b = (1+ f i > a ,
c = (4+3&)a.
We conclude that
a+3c
4b
a+2b+c+a+b+2c
-
8c
a+b+3c
has minimum value 1 2 a - 1 7 if and only if (a, b , c) = ( a , ( l + f i ) a ,
(4+3.JZ)a).
@3
Let ABC be an obtuse triangle inscribed in a circle of radius 1.
Prove that triangle A B C can be covered by an isosceles right-
angled triangle with hypotenuse f i + 1. (posed by Leng
Gangsong)
Solution Without loss of generality, we
may assume that L C > 90". Since L A
L B <90", we may assume without loss of A
0
e D
+
China Girls' Mathematical Olympiad
2004
145
generality that L A < 45".
We can then construct a semicircle w with A B as its diameter
such that point C lies inside w . Let 0 be the center of w . ThenAO =
BO. Construct rays AT and OE such that LBAT = L B O E = 45" with
E lying on w . Let 1 be the line tangent to w at E, and let 1 meet rays
AT and A B at F and D respectively. It is not difficult to see that
triangle AFD is an isosceles right-angled triangle, with L F = 90",
that covers triangle ABC. It suffices to show that A D <&+1. Note
that
AD=AO+OD=AO+&OE=(&+~)AO.
Applying the sine rule to triangle ABC gives A B
= 2sinC<
2, or
A0 < 1. It follows that A D <&+1, as desired.
A deck of 32 cards has 2 different jokers each of which is
numbered 0. There are 10 red cards numbered 1 through 10 and
similarly for blue and green cards. One chooses a number of
cards from the deck. If a card in hand is numbered K , then the
value of the card is 2' , and the value of the hand is the sum of
the values of cards in hand. Determine the number of hands
having the value 2004. (posed by Tao Pingsheng)
Remark The answer of the problem is 1 0032 = 1 006 009. We
provide two approaches as follows.
Solution I For each hand h having the value 2004, let j h , i h ,
b h and g h denote the total values of jokers, red, blue, and green
cards in hand. Then ( j h , i h , b h , g h ) is an ordered quadruples of
nonnegative integers such that
j , + %+ b h +g,
where 0
< j , < 2 and
i h
, bh
and
gh
= 2 004,
(*>
are even. Note that 2 004 <
2 048 =211 , each even nonnegative integer e less than 2 004 can be
written uniquely in the form of
Mathematical Olympiad in China
146
< <
whereai = 0 or 1 for 1
i
10. (This is basically the binary
representation of e . Hence each ordered quadruple of nonnegative
integers ( j h , rh , bh , g h ) satisfying equation ( * can be mapped oneto-one and onto a hand having the value 2 004. It suffices to count the
number of ordered quadruples of nonnegative integers in equation
( * that satisfy the conditions. Since i h , b h and g h are even, j h is
also even. Set j h = 2 j , rh = 2 r , bh = 2b, and gh = 2g. It suffices to
count the number of ordered quadruples ( j , I , b , g> of nonnegative
integers such that
>
>
>
j+r+b+g
=
( **
1002,
with j = 0 or 1. We consider two cases.
In the first case, we assume that j = 0. Then r+b+g
(' ","")
=
Oo4
2
integers such that r
>
= 1002 has
Oo3 ordered triples ( I , b , g > of nonnegative
+b +g
=
1 002 , that is, there are
1004 1 003
ordered quadruples (0, I , b , g> of nonnegative integers satisfying
equation ( ** .
In the second case, we assume that j = 2. Then r + b+ g = 1001
>
has
(' Oo3)
2
=
Oo3
integers such that r
2
+b
, b , g> of
nonnegative
1 001 , that is, there are
1003 1 002
2
Oo2 ordered triples
=
g
=
( I
ordered quadruples ( 1, I , b , g> of nonnegative integers satisfying
equation ( ** . Therefore, the answer for the problem is
>
1004 1003 + 1003 1002 = 10032.
2
2
Solution I[ For a particular hand, a card in hand numbered K
contributes either 0 or 2' points to the value of the hand. We consider
the following generating function
China Girls' Mathematical Olympiad
f(..)
=
(1
+2'
)2
[(1
2004
147
+2 ) (1 +g2) (1 +2')13.
That is, if a , denotes the number of hands having a value rz , then
, where [x,]
h ( x ) denotes the coefficients of x n in
the expansion of the polynomial h ( x ) . We compute a2004 =
[xz004]f(x>. Note that
a, =[x,]f(x)
Since 2 004 < 2"
, we have [ x 2 0 0 4 I f ( x )
= [x2004Ig(x)
9 where
Note that
It follows that for any positive integer K ,
[x~:zK]~
=( 1
x+) 3 + 5 + . . * + ( 2 K + l )
In particular,
a2004
=
(/~+1)~.
= [x2004]g(x)= 10032.
Second Day
8:OO- 12:OO August 13, 2004
@@& Determine the maximum value of constant A such that
u+v+w>A,
Mathematical Olympiad in China
148
where u , v and w are positive real numbers with u
w&
6
> 1. (posed by Chen Yonggao)
+ v&+
Remark The maximum value of A is&. It is not difficult to see that
for u = v = w
a, u
=-
3
6 +
&+
6= 1 and u +v+ w = a.
Hence the maximum value of A is less than or equal to
a.It suffices
toshow t h a t u +v + w > &.
Solution I By the AM-GM inequality and the given condition, we
have
or
uv+vw+wu>l.
Since
+ (v-w)2 + (w-u)2 >0 ,
u2 + z? +a? > uv + vw + wu > 1 ,
(u-
v)2
it follows that
(u+v+w)2
or u
=
u2+z?
+a? +2(uv+vw+wu)
+v +w >&. The equality holds if and only if u
2 3 ,
=v =w =
a
-.
3
Solution I[
By the AM-GM inequality and then by Cauchy's
Inequality, we have
(u
+v9+w)
= ("+;+")3
3(u+v+w)
>3uvw(u+v+w)
= (uvw
+ vwu +wu v )( u + v +w)
China Girls' Mathematical Olympiad 2004
>(u4/Gi+v&+wz/uu)2
which implies that u+v+w>&.
Solution a Set x
condition reads
Note that
u =
=&,
y
ZX
-,
Y
=
v
149
=
1,
The equality hold if and only if u =
=6,
and z =&.
g,
and w
z
=
Then the given
E. By the AM-GM
X
inequality, we have
2(u+v+w)
=
(=+=)+(:+:)+(:+y)
>2x
Y
+2y +22 ,
oru+v+w>x+y+z.
y + ~ >) 3~( x y + y z + z r ) > 3 .
3 , or u + v + w
that is,
z
As shown in Solution
I , we have ( x +
>
H e n c e ( ~ + v + w )>
~(x+y+~>~
>&, and the equality holds if and only if x
= y = z,
&
u =v =w=-
3'
Given an acute triangle ABC with 0 as its circumcenter. Line
A0 and side BC meet at D. Points E and F are on sides AB and
AC respectively, such that points A , E , D and F are on a
circle. Prove that the length of the projection of line segment EF
on side BC does not depend on
the positions of E and F. (posed
by Xiong Bin)
Solution Let M and N be the feet
of the perpendiculars from D to lines
A B and AC respectively. Let Eo ,
F o , M O and NO be the feet of the
A
C
150
Mathematical Olympiad in China
perpendiculars from E, F, M and N to side B C respectively. It
suffices to show that EoFo= MONO.Without loss of generality, we
may assume that E lies on line segment BM. Then L A E D < 90".
Since A , E, D , F are concyclic, L D F C = L A E D < 90", and so F
lies on line segment A N . It follows that we need only to consider the
figure above. It suffices to show that EoMo = FoNo. Note that
EoMo=EM cosLB and FoNo = FNcos L C . We need only to show
that EMcosLB = FNcosLC, or
EM - cosLC
FN
cosLB'
--
(1)
Since L M E D = L A E D = L D F C = L N F D , the right-angled
triangles E M 0 and FNO are similar, which implies that
EM - MD
FN
ND'
(2)
In the right-angled triangles A M 0 and A N O , MD = AD
sinLDAM =ADsin LOAB and N D = A D sin L D A N = AD
sinLOAC. Substitute these equations into equation (2) and we have
EM
FN
~-
sinLOAB
sinLOAC'
(3)
Since 0 is the circumcenter of triangle A B C , LAOB = 2LCand
LOAB = LOBA = 90"-LC, and so sinLOAB = sin(90"-LC) =
cos L C . Likewise, sin LOAC = cos L B . Substitute the last two
equations into equation (3) and we get the desired equation (1).
Let p and q be two coprime positive integers, and let rz be a
nonnegative integer. Determine the number of integers that can
be written in the form ip j q , where i and j are nonnegative
integers with i + j
n. (posed by Li Weigu)
Solution Define a set
S ( p , q , n) = { ip + j q I i and j are nonnegative integers with i
<
+
+
China Girls’ Mathematical Olympiad
j
2004
151
<n } .
Lets, = I S ( p , q , n> I , where I X I denotes the number of
elements in set X. The answer of the problem is
(*>
where r = max{p, q } .
Now we establish the equation ( * . Without loss of generality,
we assume that r = p > q. It is easy to see that so = I S ( p , q , 0) I =
I { 0) I = 1 satisfying equation ( * . Note that
>
>
S ( p , q , n>\S(p, q , n - 1 )
c { i p + ( n - i > q I i = 0 , 1 , ..., n } .
Note also that
ip+(n-ii)q=
(i+q>p+(n-p-ii)q,
with
(i+q)+(n-p-ii)
=
+
n+q-p<n--l.
Hence number ip ( n - i > q belongs to both sets S ( p , q , n>and
S ( p , q , n- 1 ) if and only if n- p - i 0, or i n- p. Therefore,
=i
{ip+(n-ii)q
{ip+(n-ii)q
I
I
>
<
i = n-p+l, n-p+2,
i = o , 1,
n},
..a,
..a,
It implies that
if n > p ,
if n < p.
If n < p , we conclude that
n } , if n > p ,
if n p.
<
Mathematical Olympiad in China
152
=1+2+-.+(n+l)
=
In particular, spP1 = p ( p + '). If n
2
s,
= sp-1
+( s p
- sp-1)
(n+ 1)( n
2
+2)
p, we conclude that
+ ...+ (s,
-
)
Remark If p = 4, then
S ( p , q , n) = S ( p , p , n ) = { (i + j ) p I i and j are nonnegative
n}.
integers with i j
It is easy to see that I S ( p , p , n ) I is equal to the number of
ordered triples of nonnegative integers (i, j , K) such that i+j+K = n ,
which implies that I S ( P , P, n ) I = (n+ l > ( n 2)
2
+<
+
When the unit squares at the four corners are removed from a
three by three square, the resulting shape is called a cross.
What is the maximum number of non-overlapping crosses placed
within the boundary of a 10 X 11 chessboard? ( Each cross
covers exactly five unit squares on the board. ) (posed by Feng
Zuming)
Solution The answer is 15.
We first show that it is impossible to place 16 crosses within the
boundary of a 10 X 11 chessboard. We approach indirectly by assuming
that we could place 16 crosses.
The centers of the crosses (denoted by * ) must lie in the 8 X 9
subboard in the middle. We tile this central board by three 8 X 3
boards, and label these three boards (a) , (b) and (c) , from left to
right. We consider the number of centers placed in the three boards.
It is easy to see that in a 2 X 3 board or a 3 X 3 board, we can
place at most two centers.
China Girls’ Mathematical Olympiad
2004
153
Note that we can place at most two centers on each 3 X 3 subboard:
*
*
We can tile a 8 X 3 board one 2 X 3 board sandwiched by two 3 X 3
boards. Hence we can place at most 6 centers on a 8 X 3 board, with
each two centers placed on each subboard. Since there are two centers
placed in the middle 2 X 3 subboard, no centers can be places in the
third and sixth row of the 8 X 3 board. We can only have the
following two symmetric distributions.
154
Mathematical Olympiad in China
Therefore, there are 16 centers placed in board (a>, (b) and (c) ,
with each board holding at most 6 centers. Hence at least one of the three
boards must contain 6 centers. We consider the following cases.
Case I Board (b) has 6 centers. By symmetry, we can assume that
the following scheme for placing the centers (see left-hand side
diagram in the following figure). It is not difficult to see that no
centers can be placed on the third and the seventh columns of the 8 X
9 board. Then it is easy to see that we can place at most 4 centers in
board (a> or (c) , which implies that we can place at most 4+6 +4 =
14 centers on the 8 X 9 board. It contradicts the assumption.
China Girls’ Mathematical Olympiad
2004
155
Case I[ Both boards (a> and (c) have six centers. By symmetry,
we discuss with the right-hand side diagram in the following figure. In
this case there is no centers can be placed in the forth and the sixth
columns of the 8 X 9 board, which implies that board (b) can hold at
most 3 centers, and so the 8 X 9 board can hold at most 6 3 6 = 15
centers, which is again a contradiction.
Case
Exactly one of the boards (a> and (c) has six centers. In
this case we assume that (a> contains 6 centers and board (c) contains
at most 5 centers. Then no center can be placed in the fourth column
of the 8 X 9 board. It follows that board ( b ) contains at most 4
centers. Hence there are at most 6 4 5 = 15 centers on the 8 X 9
board, which is again a contradiction.
Combining the above argument , we conclude that it is impossible
to place 16 centers on a 8 X 9 board.
We complete our solution by providing two different ways to
place 15 centers on the board.
++
+ +
or
Remark The example shown on the right-hand side comes from
the fact that the crosses can tile the whole plane without any gaps.
The example shown on the left-hand side shows that it is possible to
have holes ( or gaps) between the crosses for this particular
problem , which makes the other approaches for this problem
difficult .
Mathematical Olympiad in China
156
2005
(Changchun, Jilin)
The 4th China Girls' Mathematical Olympiad was held on August 10 15, 2005 in Changchun, Jilin Province, China, and was hosted by
CMO Committee.
The Competition Committee consisted of the following: Wang Jie
(the president) , Zhu Huawei, Chen Yonggao, Su Chun, Xiong Bin,
Feng Zuming, Zhang Tongjun, Feng Yuefeng, Ye Zhonghao and
Yuan Hanhui.
First Day
8:OO- 12:OO August 12, 2005
a@@As shown in the following figure, point P lies on the circumcicle
of triangle A B C . Lines A B and CP meet at E , and lines A C and
BP meet at F . The perpendicular bisector of line segment A B
meets line segment A C at K , and the perpendicular bisector of
line segment A C meets line segment A B at 1. Prove that
CE
(m)= AAJK * KJ EF . (posed by Ye Zhonghao)
Solution Set L s A C = X. By the given condition, we have
A K =BK and AJ = CJ , and so L A B K = L A C J = x and L E J C =
L B K F = 2x. Note also that L E CJ =
A
L A C P - L A C J = L A C P - x. In triangle
A B F , we have L A F B = 180" - L A B P L A B F = 180"- L A B P - x. Since A , B ,
P , C are concyclic, we have L A C P =
180"-LABP. Combining with the above
E
F
China Girls' Mathematical Olympiad
2005
157
equation, we conclude that L B F K = L B F A = L A C P -x = L E C J .
Thus, in triangles CEJ and FBK, we have L C J E = L B K F and
L E C J = L B F K . So the two triangles are similar. It follows that
CE-- CJ- EJ
FB F K - E '
Consequently, we have
as desired.
%@@Find all ordered triples ( x , y , z > of real numbers such that
]5(xf$)=
1 2 ( y + y1) = 1 3 (z + -),
1
z
Ly+yz+zx
Remark
=
(posed by Zhu Huawei)
1.
The solutions are
2
( 17 ,7,
1)
and
1
2
(-7,
-7,
-1).
Assume that ( x , y , z > is a solution of the given system. Clearly, x y z
# 0. If x > 0 , then by the first given equation, we have y >0 and z >
0. Ifx<O, thenitisclearthaty<Oandz<O,andso(-x, - y , z > is a solution of the given system.
Solution There are angles A , B, and C in the interval (O", 180")
such that
x
=
A y
tan-,
2
=
B
tan-,
2
C
z = tan-.
2
By the addition and subtraction formulas, the second equation in
the given system becomes
A
B
1 = tan- tan2
2
=tan-
implying that
A tan-B
2
2
+ tan-B2 tan-C2 +tan- C2 tan-2A
+ tan-C2 tan-' + '2( I
-
t a n A tan'),
2
2
Mathematical Olympiad in China
158
(1-tan-
2
tan-A +
2
Note that tan-A tan-B
2
2
z =
")
(I -tan-
A
2
tan2
")
=
# 1, otherwise z(x+y>
= xy
0, which is impossible. Therefore, tan
0.
= 0,
implying
C
A+B
tan - = 1, or
2
2
-
+
C
- = 90". In other words, A , B, C are the angles of a
2
2
triangle. Let ABC denote that triangle.
We rewrite the first equation in the given system as
Note that, by the double-angle formula, we have
A
X
--
x2+1
tan y
-
2A
tan - + I
2
A
- ~A
tan y
-
. A
A
s i n y cos-
se22
sinA
2 '
=-
2
and analogously for the expressions of y and z . We conclude that
sinA
5
-- -
sinB
12
-
sinC
13
By the sine rule, the sides of triangle ABC are in ratio 5 : 12 : 13
5
12
X
5
with sinA = -, sin B = - and sin C = 1. Hence
- -, or
13
13
x2+1
26
~
5 2 -26x
+5
= 0,
1
implying that x = 5 or x = -. Likewise, we have
5
3
2
y = - or y = - , and z = 1. Substituting z = 1into the second equation
2
3
in the given system leads to xy + x + y = 1, implying that (x, y , z> =
1 2
1) is the only solution with x > 0. Hence (7,3,1) and
(+, $,
1
(-7,
-3'
1) are the solutions of the problem.
China Girls' Mathematical Olympiad
2005
159
@@ Determine if there exists a
convex polyhedron such
that
(1) it has 12 edges, 6 faces
and 8 vertices;
(2) it has 4 faces with each
pair of them sharing a
common edge of the
polyhedron. (posed by
Su Chun)
Solution The answer is yes,
as shown in the figure.
@@$ Determine all positive real numbers a such that there exists a
positive integer rz and sets A1 , A2,
A , satisfying the
following conditions :
(1) every set Ai has infinitely many elements;
(2) every pair of distinct sets Ai and Aj do not share any
common element ;
(3) the union of sets Al , A2,
A , is the set of all integers;
(4) for every set Ai , the positive difference of any pair of
elements in A i is at least a ' . (posed by Yuan Hanhui)
Solution The answer of the problem is the set of all positive real
numbers less than 2. We consider two cases.
Case I We assume that 0 < a < 2. Then there is a positive rz such
that 2"-' > an. We define A, = { m I m is a multiple of 2"-' } and
..a,
..a,
I m is an odd integer} ,
for 1< i < n - 1. Then Al , A 2 , , A , is a partition of
Ai
=
{ 2"lm
the set of
positive integers satisfying the conditions of the problem.
Case I[ We assume that a
2. We claim that no such partition
exists. To prove by contradiction, we assume on the contrary that
A1 , A2 ,
A , is a partition satisfying the conditions of the
..a,
Mathematical Olympiad in China
160
<
problem. Let N = (1, 2,
2"). For every i with 1 i < n, let
Bi =Ai N. We assume that Bi = {bl , b2 ,
bm} with bl <
b2 < < bm. We have
..a,
n
-a,
2"
> bm
-
+.a*+
bl
=
(bm - bm-l>+ (bm-1
(bZ-bl)>
-
bm-2
(~~-1)2~,
A, i s a
implyingthatm-l<2"-i,
0rrn<2"-~. SinceAl, A2,
partition, Bi B j = 0for 1 i <j
n and N = B1 U B2 U
U
B,. It follows that
..a,
n
<
<
which is impossible. Hence our assumption was wrong and such a
partition does not exist for every positive integer n.
Second Day
8:OO- 12:OO August 13, 2005
&
@ Let x and y be positive real numbers w i t h 2 +y3 = x-y. Prove
that 2 4 3 < 1. (posed by Xiong Bin)
Solution In view of 2
y3 > 0, it suffices to show that
(2 4y2) ( X- y ) < 2 +y3. Expanding the left-hand side of the last
inequality and canceling the like terms we obtain 4xy2 < 2 y 5 9 .
+
+
+
+
By the AM-GM inequality, we have 2 y
>
+ 5y3 >2 &xy2 > 4xy2.
An integer n is called good if n 3 and there are n lattice points
P I , P2 , .-, P , in the coordinate plane satisfying the following
conditions: If line segment PiPj has a rational length, then there
is pk such that both line segments PiPk and PjPk have irrational
lengths; and if line segment PiPj has an irrational length, then
there is pk such that both line segments PiPk and PjPk have
rational lengths.
(1) Determine the minimum good number.
China Girls’ Mathematical Olympiad
2005
161
(2) Determine if 2005 is a good number.
(Apoint in the coordinate plane is a lattice point if both of its
coordinate are integers. ) (posed by Feng Zuming)
Solution We claim that the minimum good number is 5, and that
2 005 is good.
We say PiPj is rational (irrational) if segment PiPj has a
rational (an irrational) length. We say an ordered triple ( P i , Pj , Pk)
of lattice points to be good with 1 i <j
n and 1 K
n if PiPj
is rational (irrational) and both PiPk and PjPk are irrational
(rational). In the figure shown below, if PiPj is rational (irrational)
then Pi and P j are connected by darkened solid (regular solid) line
segment.
It is not difficult to see that n = 3 is not a good number. Note that
n = 4 is also not a good number. Assume on the contrary that there
are lattice points P I , P2 , P3 , P4 satisfying the conditions of the
problem. Without loss of generality, we assume that P I P2 is rational
and ( P I , P 2 , P3 ) is good. Then ( P 2 , P 3 , P4 1 must be good.
Neither ( P 2 , P 4 , P I ) nor ( P 2 , P 4 , P31 is good, violating the
condition of the problem. Hence our assumption was wrong and n = 4
is not a good number.
<
<
P2 =
(1, 01,
< <
For n = 5, we define the set
s
5 = {PI =
(0, 01,
P4 =
(8, 71,
P5 =
P3 =
(5, 31,
(0, 71).
It is not difficult to see that all triples ( P i , Pj , P k ) are good.
Hence n = 5 is minimum good number.
Mathematical Olympiad in China
162
Consider the sets
A = { ( 1 , 0) 9 ( 2 , 0) 9
B = { ( l , 11, (2, 11,
C = { ( l , 21, ( 2 , 21,
. *,
. *,
. *,
(669, 0) } 9
(668, I ) } ,
(668, 2 ) ) .
We claim that the 2 005 points in the set S2005 = A U B U C
satisfies the conditions of the problem. Consider a pair of distinct
points Pi = ( x i , yi and Pj = ( x j , y j in the set S2005. If either
xi = x j or yi = y j , then clearly PiPj is rational. Otherwise, either
~ x i - x j ~ = l o r 2 or
, l y i - y j I = l o r 2 , orboth. Hence IPiPjI2
can be written in the form of either n2 1 or n2 4 for some positive
integer n . For a pair of positive integers n and m with n > m , we
have n2 - m2 n2 - ( n - 1 l2 = 2n - 1. It follows that for a positive
integer n , n2 + 1 and n2 + 4 are not squares of an integer. We conclude
that ( P i , Pj is rational if and only if line PiPj is parallel to one of
the axes.
If PiPj is rational with yi = yj , then we set Pk = ( X k , Y k 1, with
X k # xi , X k # xj and yk # y i . (We can do so because there are 668
distinct x values and three distinct y values. ) Then ( P i , Pj , pk is
good. In exactly the same way, we can deal with the case when PiPj
is rational with xi = x j .
If PiPj is irrational, then xi # xj and yi # y j . We set pk =
( x i , yj 1. Then ( P i , Pj , Pk is good. Therefore, set S2005 satisfies
the conditions of the problem and n = 2 005 is good.
Remark Set S6 = S5 U {P6 = ( - 2 4 , 0 ) ) and S7 = s6 U {P7 =
(- 24, 7) }. It is not difficult to see that both 6 and 7 are good. For
every positive integer n 8 , we can easily generalize the construction
of part ( 2 ) to show that n is good. Hence all integers greater than 4
are good.
+
+
>
>
@Let
if&
m and n be positive integers with m
> n > 2.
Set S =
( 1 , 2 , .-, m } ,a n d T = { a l , a2, .-, a,} is a subset of S such
China Girls' Mathematical Olympiad
2005
163
that every number in S is not divisible by any two distinct
numbers in T. Prove that
l+-+...
1
1
m+n
a1
an
a2
(posed by Zhang Tongjun)
Solution For every i with 1 i
< < n, we define set
Si
=
There are
{ b I b is an element in S and is divisible by a i } .
[El elements in Si. Since every element in S is not
divisible by any two distinct elements in T, it follows that Si
0for 1 i < j n. Thus
<
<
2[3=PISiI< I S I = m .
< [El+ 1. It follows that
,=1
Note that
ai
n Sj =
i=l
So the desired result is obtained.
@@ Given an a X b rectangle with a > b > 0, determine the minimum
length of a square that covers the rectangle. (A square covers the
rectangle if each point in the rectangle lies inside the square.)
(posed by Chen Yonggao)
Solution Let R denote the rectangle, and let S denote the square
with minimum length that covers R . Let s denote the length of a side
of S. We claim that R is inscribed in S, that is, the vertices of R lie
on the sides of S. We also claim that R can only be inscribed in two
ways, as shown below. Let S1 ( S 2 1 denote the square shown on the
left-hand side (right-hand side). For S2 , the sides of R are parallel to
the diagonals of S 2 . It easy to see that s = a if S = SI.
Mathematical Olympiad in China
164
It is not difficult to see that s = .lZ(u+b) if S = S2. Taking the
2
minimum value of
(
U,
b,
] , we conclude that
Now we prove our claim that these are only two ways. Let R =
ABCD and S = XYZW. Without loss of generality, we place XY
horizontally. By the minimality of S, we can assume that at least one
vertex, say A , of R lies on one side of S , say WX (see the left-hand
side figure shown below). If neither B nor D lies on the sides of S,
we can then slide R down (vertically) , so that one of them, say B lies
on side XY (see the middle figure shown below). If neither C or D
lies on the sides of S , then we can apply an enlargement, centered at
X with scale less than 1, to S such that the image of S still covers R .
This violates the minimality of S. Hence at least one of C and D lies
on the sides of S, that is, three consecutive vertices of R lie on the
sides of S (see the right-hand side figure shown below). Without loss
of generality, we assume that they are A , B and C. If any of these
three vertices coincide with any of the vertices of S, then we clearly
have S = S1. Hence we may assume that A , B and C are on sides
WX, X Y and YZ,respectively. By symmetry, we may also assume
thatAB = u > b = B C .
If D does not lie on line segment ZW, then we can slide R up a
bit so both B and D lie in the interior of S (see the left-hand slide
China Girls’ Mathematical Olympiad
z
2005
165
w,
figure shown below). Let 0 be the center of R . We can then rotate
R around 0 with a small angle so that all four vertices lie inside S
(see the middle figure shown below). It is easy to see that we can use
a smaller square to cover R (by applying an enlargement centered at
0 with a scale less that 1) , violating the minimality of S. Thus our
assumption was wrong, and D must lie on side ZW (see the righthand side figure shown below) , which is the case when S = S2.
x
We finish our proof that if S = S 2 , then the sides of R are
parallel to the diagonals of S 2 . By symmetry, it suffices to show that
A X = X B . It is not difficult to see that L X A B = L Y B C = L Z C D
= L W D A . So triangles A B X , BCY, CDZ and DAW are similar. Set
A X = u x and X B = uy. Then B Y = bx and C Y = by. Also, DW = bx
and WA = by. Hence by u x = WA A X = W X = X Y = X B
B Y =uy+bx, implyingthat(u-b)x= (u-b)y, o r x = y , asdesired.
+
+
+
China Western Mathematical
Olympiad
T h e China Western Mathematical Olympiad, organiEd by the
China Mathematical Olympiad Committee, is held in November
every year. Students of grade 10 and 11, from Kasakshtan, Hong
Kong, Macau and West China, are invited to take part in the
competition. The competition lasts for 2 days, and there are 4
problems to be completed within 4 hours each day.
2002
(Lanzhou, Gansu)
The 2nd (2002) China Western Mathematical Olympiad was held on
December 3 - 8, 2002 in Lanzhou, Gansu, China, and was hosted by
Gansu Mathematical Society and Lanzhou University.
China Western Mathematical Olympiad
2002
167
The Competition Committee consisted of the following: Pan
Chenbiao , Fan Xianling, Li Shenghong, Leng Gangsong, Xiong Bin,
Feng Zhigang, Wang Haiming and Zhao Dun.
First Day
8 :00 - 12 :00 December 4, 2002
~2
-.=-s
i
Find all positive integers rz such that
n4 - 4n3
+22n2
-
36n
+ 18
is a perfect square. (posed by Pan Chengbiao)
Solution We write
A = n4 -4n3 +22n2 -36n+18
+
= (n2 - 2 ~ 2 ) ~18(n2 - 2n)
+ 18.
Let n2 -2n = x, A = 9, where y is a nonnegative integer. Then
( ~ + 9 -63
) ~
That is, ( x + 9 - y ) ( x + 9 + y )
= y2.
= 63.
It can only be (x+9-y, x+9+y) = (1, 63) , (3, 21) or (7, 9).
We obtain, respectively, (x,y) = (23, 31), (3, 9) or (-1, 1). But
only when x = 3 and - 1, n2-2n = x has solutions in positive integers
and we get n = 1 or 3.
Therefore, n = 1 or 3 satisfies the condition.
Remark We can also deal with this problem by using inequality.
@ Suppose 0 is the circumcenter of an acute triangle A A B C , P is
a point inside A A O B , and D , E , F are
A
the projections of P on three sides B C ,
C A , A B of A A B C respectively. Prove
that a parallelogram with FE and FD as
adjacent sides lies inside A A B C . (posed
by Leng Gangsong)
Mathematical Olympiad in China
168
Proof As shown in the figure, we construct a parallelogram DEFG
with FE and FD as adjacent sides. To prove the proposition to be
true, we need only to prove that L F E G < L F E C , and L F D G <
L F D C . It is equivalent to proving: L B F D <L B A C , and L A F E <
LABC.
In fact, we construct OH with OH 1B C , and H is the foot of
the perpendicular. From PD 1BC and PF 1A B , we know that four
points B, F, P and D are concyclic. Thus L B F D = LBPD. But
L P B D >LOB H , hence 90" - L P B D < 90" - L O B H , and that is,
L B P D < L B O H . Moreover, 0 is the circumcenter of A A B C , so
1
L B O H = - 1 B O C = L B A C . Therefore, L B F D = LBPD <
2
L B O H = L B A C , that is, L B F D < L B A C .
Similarly, we can prove that L A F E < L A B C. Therefore, the
proposition holds.
Remark If 0 is not the circumcenter of A A B C , but the incenter or
orthocenter, does the conclusion given in the problem still hold? The
reader may wish to think over it.
@@& Consider a square on the complex plane. The complex numbers
corresponding to its four vertices are the four roots of some
equation of the fourth degree with one unknown and integer
coefficients x4 + p x 3 +qx2 + r x + s = 0. Find the minimum value
of the area of such square. (posed by Xiong Bin)
Solution Suppose the complex number corresponding to the center
of the square is a . Then after translating the origin of the complex
plane to a , the vertices of the square distribute evenly on the
circumference. That is, they are the solutions of equation ( x - u ) ~=
b , where b is a complex number. Hence,
x4+px3+qx2+m+s=
(x-a)2-b
=x4
-
4m3
+6 a 2 2
-
4a3x
+a4
-
b.
Comparing the coefficients of terms for x with the same degree,
China Western Mathematical Olympiad
we know that
-
a
=
P
-,
4
2002
169
and it is a rational number. Combining
further t h a t - 4 ~ ~= r i s an integer, we can see that a is an integer. So
by using the fact that s = a4 - b is an integer, we can show that b is
also an integer.
The above discussion makes clear of a fact that the four numbers
corresponding to the four vertices of this square are roots of integer
coefficients equation ( x - a>4 = b. Hence, the radius of its
is not less that 1 . Therefore, the area of this
circumcircle ( =
m)
= 2. But the four roots of the equation
square is not less than
x4 = 1 are corresponding to the four vertices of a square on the complex
plane. Hence, the minimm value of the area of the square is 2.
Remark By using the method above, we can prove: If the
corresponding complex numbers of vertices of a regular n-gon are n
complex roots of some equation with integer coefficients xn
an-1x-'
a0 = 0, then the minimum value of the area of a
+
+ +
n
2
regular n-gon is -sin
.=.-
-.2nx
Assume that n is a positive integer, and A1 , A2, ,An+l are n+
1 nonempty subsets of the set { 1 , 2,
n ). Prove that there are
two disjoint and nonempty subsets { il , i2,
ib ) and
{ j l , j 2 , . *, j, ) such that
..a,
..a,
Ai IJ Ai IJ
IJ Aik = Aj, IJ Aj, IJ
IJ Aj
112
.
(posed by Zhao Dun)
Proof I We prove by induction for n .
When n = 1 , A1 = A2 = { 1 ) , the proposition holds.
Suppose it holds for n . We consider the case of n 1.
Suppose A1 , A2 , .-, An+2 are nonempty subsets of { 1 , 2,
n + l ) . Let& = A i \ { n + l ) , i = 1 , 2,.-, n+2. We willprove the
following cases.
Case I There exist 1 i < j n+2 such that Bi = B j = 0,
+
..a,
<
<
Mathematical Olympiad in China
170
+
then Ai = Aj = { n 1} . The proposition is proven.
Case II There exists only one i such that Bi = 0.
There is no
loss of generality in supposing Bn+2 = 0,
and that ~ S A , +=
~ { n 1} .
Now by the inductive assumption, for { 1, 2,
n 1 } , there exist
two disjoint subsets { il , , ik } and { j l , , j , } such that
+
..a,
Bil
+
u ... u Bik = Bj, u ... u Bj,.
We write C = Ail U
U Aik, D = Aj, U
0
U Aj
112
. Then C and
D differ at the most by the element n + 1. (This can be shown by 0
and the definition of Bi. ) In this case, we can make the proposition
to hold true by putting&+2 into C or D.
Case Ill No Bi is empty. Now B1, B 2 , , Bnflare nonempty
subsets of { 1, 2 , , rz } . By the inductive assumption, we show that ,
for { 1, 2,
n 1 } , there exist disjoint subsets { il ,
ik } and
{jl,
j , } such that
+
..a,
..a,
..a,
Bil
u ... u Bik = Bj, u ... u Bj .
0
112
In addition, B 2 , B 3 ,
Bn+2 are also nonempty subsets of
{ 1, 2 , , rz } . By the inductive assumption, we can show that, for
2 } , there exist disjoint subsets { r1,
r u } and
( 2 , 3, .-, n
{tl,
t o }such that
..a,
+
..a,
..a,
Brl U
U Br,
= Btl
U
U Bto-
0
Again, we write C = Ail U U Aik, D = Aj, U U Aj , and
write E = A,, U U AT, , F = A,, U U Ate. By using 0 , 0and
112
the definition of Bi , we see that C and D differ at the most by the
element n+ 1, and so do E and F. If C = D or E = F, then the
proposition holds. Hence we need only to consider the case when C #
D and E # F. There is no loss of generality in supposing C = D U
{ n + l } , but E = F\{n+l}.
Now C U E = D U F. After
amalgamating the sets occurred repeatedly in C and E, as well as in D
and F, we get two subsets { P I , p,} and { q l ,
q y } of (1,
, n 2 } such that
2,
+
..a,
..a,
China Western Mathematical Olympiad
u ... u Ap,
Ap,
= A,,
u ... u
2002
171
A p y 9
where G = Ap, U U Ap, = C U E, H = 4, U U 4Y = D U F.
Now, if { P I ,
p, } ( 4 1 ,
qy } = 0,
then the proposition
p,}
(41,
q y } , we write C =
holds. If there is i E { P I ,
{Ail 9 ..*) A i k } )D = {Aj, 9 ..*) Aj } , E = { A r l ,
A r y } ,F =
n
..a,
..a,
..a,
n
N
..a,
N
.a*,
112
{A,, ,
, At0} . And
there is no loss of generality in assuming that
Ai does not belong to andEat the same time, and it does not belong
to B and F at the same time too. Hence there are only two
possibilities.
(a) Ai E andAi E F. If there are two sets in containing n+
1, then we take away set Ai from the left side in @I. Now since all
elements except n + l in Ai belong to E (in view of
and there are
two sets on the left side in @I containing n + l . Thus after taking away
Ai, the number of elements in G does not reduce and @I is still an
equality. In the same way, if there are two sets in containing n 1,
then we take away Ai from the right side in @I, and @I still holds.
Of course, if there is only one set in and F containing n 1,
then after taking away Ai from both sides in @I, it remains to be an
equality. (Now, by 0and 8,
we can see that the two sides of @I will
not become empty sets. )
(b) Ai E B and Ai E E , then n+ 1 @ Ai. Now after taking away
Ai from both sides in @I, the resulting expression is still an equality.
In view of the above operation, we have a method to make the
two sets of subscripts { p l , , p , } and { q1 , , qy } in @I disjoint.
Therefore the proposition holds for n 1.
As a consequence of what is described above, we show that the
proposition holds.
Proof II Here we need to use a fact from linear algebra that n 1
vectors in the n-dimensional linear space are linearly dependent.
If element i is in set Aj , we write it as 1, otherwise write it as 0.
Then Aj corresponds to an n-dimensional vector, which is nonzero
a j , ) , where
and contains 0 and 1. We write aj = (ajl, a j , ,
c
c
c
a),
+
c
+
+
+
..a,
Mathematical Olympiad in China
172
+
Since a1 , a2 , .-, an+l are n
1 vectors in the n-dimensional
space, so there exists a group of real numbers, not every one of them
xn+l such that
to be zero, x 1 , x2 ,
..a,
+
n
1 } , there exist two disjoint and
Hence, for { 1, 2,
nonempty subsets { il , .-, ik } and { j l , .-, j , } such that
..a,
xik> 0 , yjl = ( - x j 1 >, yjm= ( - x j m ) > O ( Here,
wherexi, ,
it is essential to put the terms with coefficients greater than zero in 0
to one side, and those with coefficients less than zero to another
side).
We conclude that
..a,
Ail IJ Ai2 IJ
..a,
.
0
(1 < a < n ) belongs to the left side in 0,
IJ Aik = Aj, IJ
IJ Aj
m
In fact, if element a
then the a-th component of the sum of the vectors from the left side
in @ must be greater than zero. Thus it makes the a-th component of
the sum of the vectors from the right side in @ to be greater than
zero. Hence, there is a j , , and its a-th component is 1, that is, a E
Aj,. Conversely, it is also true, that is, 0holds.
Therefore, the original proposition holds.
Remark Proof II is easy and fundamental. But it needs to use some
knowledge of linear algebra. Although such knowledge is elementary,
but it is not faught in the middle schools. In Proof I , after taking
away n+ 1and obtaining conclusion by using inductive assumption, we
may not put n+ 1 back simply. Professor Pan Chengbiao obtains this
proof after putting in a lot of hard work.
China Western Mathematical Olympiad
2002
173
Second Day
8 :00 - 12 :00 December 5, 2002
@@& In a given trapezium A B C D , A D I B C . Suppose E is a variable
point on A B , 0, and 0, are circumcenters of A A E D and A B E C
respectively. Prove that the length of 010, is a fixed value.
(posed by Leng Gangsong)
Proof As shown in the figure, we join
E ---__
E01 and E02, then LA E O l = 90" L A D E , /BE02 = 90"-LBCE. Hence
LO1 EO2
=LADE
+LECB.
LZl
B
Since A D / / B C , through E constructing a
line parallel to AD, we can prove L D E C = L A D E
0 2
c
+ L B CE , so
LOlE02 = LDEC.
Further , by the sine rule , we can show
DE
EC
- --
201EsinA OI E
202EsinB 02E'
Thus A D E C c/)AOl E02. Therefore
0102
DC
so0102
Remark
-
OIE - 01E -~1
DE
201EsinA - 2sinA'
DC
, which is a fixed value. The proposition is proven.
=zA
It is natural to think that the distance between the
1
projections of 0, and 0, on A B is -AB , which is a fixed value. Thus
2
we need only to prove the included angle between 010,and A B is a
fixed value. This is an idea for another method to prove the problem.
The reader may wish to try it.
Assume that rz is a given positive integer. Find all of the integer
Mathematical Olympiad in China
174
groups (a1 , a2 ,
, a,)
satisfying the conditions:
2.’;
(1) a1 + a 2 + . . * + a n
(2) a: +a; +...+a:
< n3 +I.
(posed by Pan Chengbiao)
Solution Suppose ( a l , a 2 ,
a,) is an integer group which
satisfies the conditions. Then by Cauchy’s inequality we have
..a,
1
a~+...+a~~-(~l+...+a,)~2
.’.
n
0
+-+
<
Combininga:
2 +1, we see that it can only bea:
& = 2 or.:
= 2 +I.
If it is the former, then by the condition for Cauchy’s inequality
to take the equality sign, we can obtain a1 =
= a,. This requires
a: = n 2 , 1 i < n. Combining al
+a, 2 n 2 , we have al = =
+..a+&
+..a+&
<
a,
+..a
= n.
If it is the latter, then let bi
= ai
n
g+b;+.-+b:
=
-
n , then we have
n
xa:-2nxai+n3
i=l
i=l
=2n3+1-2nxai<1.
i=l
Thus @ can only be 0 or 1, and there is at the most one among g ,
@,
..a,
b“, to be
1. If
n
g,
b2, all are zero, thenai
..a,
=
n, x u :
=
i=l
n3 #n3
+ 1. It leads to a contradiction. If there is just one among
g,
g t o b e l , thenxa?=n3&2n+l#n3+1.
..a,
Again, itleads
i=l
to a contradiction.
Consequently, we obtain that it can only be ( a l ,
(n,
..a,
a,)
=
.-, n).
@$@Assume that
a,
P are two roots of the equation 2 -xa,
=
an- p”
~
a-P
, n = 1, 2,
..a.
1 = 0. Let
China Western Mathematical Olympiad 2002
175
(1) Prove that for any positive integer n , we have an+2
an
-
= an+l
+
<
(2) Find all positive integers a and b , a
b, satisfying that b
divides a, - 2n an for any positive integer n. (posed by Li
Shenghong)
Solution (1) Noting a + p
#+2-P+2=
=
1 and ap =- 1, we have
-P+l)-ap(a”-p”)
(a+P)(an+l
= (#+I
-P+l)+ (an-p”).
+
Dividing two sides by a - p, we have a n f 2 = anfl a,.
(2) Byassumption, wehaveblal --a, t h a t i s , b I 1 - 2 ~ . Butb)
a , so b = 2a - 1. Moreover, for an arbitrary positive integer n , we
have
b I a,
-
+ 1)an+’ , b I
an+l + a, and b
2a
2nan , b I an+l - 2(n
Combining an+2
have
=
bI (n
But ( b , a )
=
an+2
=
+2)anf2
-
(n
-
+ l)an+l
-
-
nun.
1, SO
Taking n as n
0
+ 1 in 0,we have
bl (n+3>a2- ( n + 2 > a -
Subtracting the right side of
(n+l>.
1,
that is,
2a-112
so
2a - 1I 2a2 - 2a - 2.
so
-a-1,
2a2 = a (mod 2a - 1).
2a-l~-a-2,2a-l~-22a-4.
2a-11-5,
0
0from the same side of 0,
we have
bl a2 - a -
Therefore
+2)anf2.
1 an odd number, we
bl ( n + 2 ) a 2 - ( n + l ) a - n n .
But
2(n
2a-1
=
101-5.
Mathematical Olympiad in China
176
But 2a - 1 = 1implies b = a , a contradiction. So 2a - 1 = 5, a =
3 andb = 5.
We will prove in the following that when a = 3 and b = 5, for any
positive integer rz , we have b I a, - 2nan , that is 5 I a, - 2n X 3".
W h e n n = l , 2,sinceal =1, a 2 = a + ~ = l , w e h a v e a l - 2 X 3 =
-5,
5 I a,
2 X 2 X 32 =- 35. So, when n = 1, 2, we have
- 2n x 3,.
Assume that , when n = K ,K 1, this conclusion is true , that is ,
a2
-
+
51ak-2kX3',
Hence
51 ( a k + l
That is,
To prove 5 I a h f 2
+ah)
5 I Uk+2 -2
5 I Uk+2
so
51~k+l-2(k+1)
-
2(K
-
X3'+'.
-2?~X3~-2(K+1)3~+~.
x 3k(K+3(K+ l ) ) ,
2 x 3k x (4K + 3).
+2) X 3kf2, we need only to prove
2(K +2) X 3k+2 = 2 X (4K +3) X 3'(mod 5).
0
It is equivalent to9(K+2) =4K+3, that is, 5K+15=0 (mod 5).
Obviously the latter holds, Hence 0holds. Thus the conclusion holds
forn = K+2.
Consequertly, we show that ( a , b) = (3, 5) satisfying the
conditions.
Remark A sequence of numbers { a , } occurring in the problem is a
Fibonacci sequence. This problem came from a discussion with regard
to the properties of Fibonacci sequence. Problems concerning
Fibonacci sequence often appear in mathematical contests.
@'@ Assume that S = (a1 , a2 ,
..a,
a,) consists of 0 and 1 and is the
longest sequence of number, which satisfies the following
condition: Every two sections of successive 5 terms in the
sequence of numbers S are different, i. e. , for arbitrary 1 i <
j
n - 4, (ai , ai+l , ai+2 , ai+3 , ai+4) and ( a j , aj+l , aj+2 ,
aj+3 , aj+4) are different. Prove that the first four terms and the
<
<
China Western Mathematical Olympiad
2003
177
last four terms in the sequence are the same. (posed by Feng
Zhigang)
Proof Noting that S is the longest sequence of numbers satisfying
the condition. Hence, if we add a term, 0 or 1, after the last term of
S , there will occur two identical sections of successive 5 terms in S,
and that is, there exist i # j such that
, Ui+l , ..., Ui+4)
( U j , Uj+l , ..., U j + 4 )
If (a1 , a2 , a3 , a,) # ( ~
(Ui
, a,-2 , ..., a, , 0) ,
= (an-3 , a,-2 , ..., a, , 1
).
~ -, 3an-2 , a,-1 , a,) , then
= (G-3
l<i, j<n-3,
and
(Ui
, Ui+l , ai+2
9
Ui+3)
i#j,
= (Uj
-
,Uj+l,
(4-3
9
4-2
aj+2
9
, Uj+3)
4-1
9
a,).
Now, consider ai-1 , aj-1 and an-4, among which there must be
two identical terms. This causes two sections with successive 5 terms
respectively in S to be identical. It leads to a contradiction.
Therefore, the proposition holds.
Remark Note that there are at the most 32 different sections of
successive 5 terms (consisting of 0 and 1). According to this, we
know that the length for S is less than or equal to 36. The reader may
wish to try to construct one such S with length 36 and satisfying the
requirements.
2003
(Urumqi, ~injiang)
The 3rd (2003) China Western Mathematical Olympiad was held on
September 25 - 30 , 2003 in Urumqi, Xinjiang, China, and was hosted
by the Research Center of Education of Urumqi.
The Competition Committee consisted of the following: Chen
Mathematical Olympiad in China
178
Yong-gao, Li Sheng-hong, Qiu Zong-hu, Leng Gang-song, Xiong
Bin, Feng Zhi-gang , Zhang Zheng-jie .
First Day
9 :30 - 13 :30 September 27, 2003
@@@
Put numbers 1, 2, 3, 4, 5, 6, 7 and 8 at the vertices of a cube,
such that the sum of any three numbers on any face is not less
than 10. Find the minimum sum of the four numbers on a face.
(posed by Qiu Zonghu)
Solution Suppose that the four numbers on a
face of the cube isal , a2 , a3 , a4 such that their
sum reaches the minimum and a1 < a2 < a3 <
J----------.
a4. Since the maximum sum of any three l
a
.’8
’
6, and
numbers less than 5 is 9, we have a4
5
thenal +a2 +a3 +a4
16.
As seen in the figure, we have 2 + 3 + 5 + 6 = 16, and that means
the minimum sum of the four numbers on a face is 16.
>
>
@BLet a1 , a2 , .-, a2, be real numbers with
+
c
2-1
(ai+l
-
ai)2
=
i= 1
1.
+
+ +
Find the maximum value of (a,+l
an+2
a2,) - (a1
a2
a,). (posed by Leng Gangsong)
Solution First, for n = 1, we have (a2 - a1 > 2 = 1, a2 - a1
1.
Then the maximum value of a2 - al is 1.
Secondly, forn>2,1etx1 = a l , xi+l= a i f l - a i , i = l , 2,
+ +
=+
..a,
2n
2n-1.
c d = 1, andab = X I +.-+xb,
Then
K
=
1, 2,
i=2
Using Cauchy’s Inequality, we have
b,+l
=
+a,+2
n(x1+
+-.+a2n)
...+x,) + nx,+1+
-
(a1 +a2
+-.+a,)
+ ...+
( n - 1)xn+2
-[nx1 + (n - 1 ) x 2 + + . - + x n 1
x2,
..a,
2n.
The equality holds when
K
=
1, 2,
..a,
n.
So the maximum value of (a,+l
+
an+2 + m - + a 2 , )
-
Remark The first, second and third problems in this competition
are all concerning with finding the maximum or minimum values. A
solution of such a kind of problems usually involves two steps: First,
get an upper bound or a lower bound of the problem using given
conditions and well-known inequalities. Secondly, show that such an
upper bound or lower bound is attainable, and then is the maximum or
minimum value we are looking for.
@& Let rz be a given positive integer. Find the least positive integer
u, , such that for any positive integer d ,the number of integers
divisible by d in every u, consecutive positive odd numbers is not
less than the number of integers divisible by d in 1, 3, 5,
2n - 1. (posed by Chen Yonggao)
Solution The correct answer is u, = 2n - 1. The proof is given in
the following.
( 1) u,
2n- 1. As u1 = 1, we only need to consider n 2. Since
the number of integers divisible by 2n- 1in 1, 3, 5, , 2n- 1is 1 and
2(n+2n-2)-1isO,
then
thatin2(n+l)-l, 2(n+2)-1,
..a,
>
>
..a,
Mathematical Olympiad in China
180
>2n- 1.
(2) u, 2n- 1. We only need to consider the case when 2 1 d and
1 d
2n - 1. For any 2n - 1 consecutive positive odd numbers:
u,
< <
<
.-, 2 ( a + 2 n - 1 ) - 1 1 ,
2(a+l)-1,2(a+2)-11,
let s and t be
positive integers such that
(2s - l)d
< 2n
(2t - l)d <2(a
< (2s + l)d,
+ 1) 1< (2t + 1)d.
-
1
-
, 2n
Then the number of integers divisible by d in 1, 3 , 5,
is s, and
(2(t+s)
-
-
1
1))d = (2t- l ) d + (2s- l ) d + d
+ 1) 1+2n 1+2n
=2(a + 2n 1) 1.
<2(a
-
-
-
-
1
-
<
That means u,
2n - 1.
Remark The key to the solution lies in finding out the fact that the
number of integers divisible by 2n - 1 in 2n - 2 consecutive positive
odd numbers2(n+l)-l, 2(n+2)-1, .-, 2(n+2n-2)-1
is0, then
u,
2n - 1. The remaining is to prove that u,
2n - 1.
>
<
Suppose the sum of distances from any point P in a convex
quadrilateral A B C D to lines A B , B C , CD and D A is constant.
Prove that A B C D is a parallelogram. (posed by Xiong Bin)
Solution Let d ( P , 0 denote the distance from point P to line 1 .
We first prove the following lemma.
Lemma Let LSAT = abe a given angle
and P a moving point in LSAT. If the sum of
distances from P to lines AS and A T is a
constant number m , then the trace of P is a
,/'Q'i,,
segment BC with points B and C on AS and A T
1 ,
I 1
1 1
respectively , and A B = AC = m
sin a'
~
,
\
I
'
Further, if
a point Q lies inside AABC, then the sum of
S
T
China Western Mathematical Olympiad
2003
181
distances from Q to lines AS and A T is less than m , and i t is greater
than m i f Q lies outside AABC.
Proof of Lemma
For AB
=
AC
m
sin a
= -and
+
1
SAPAB SAPAC
= SAABCi. e. -AB
2
d(P,
P on B C , We have
1
AB) + TAC
d(P,
+
AC) =
AB AC sin a. Then d ( P , AB) d ( P , AC) = m. If
2
point Q lies inside A A B C , SAQAB SAQAC
< SUBC , then d(Q,
AB) d(Q, AC) <m. If Q lies outside A A B C , SAQAB SAQAC
>
SAABC,
then d(Q, AB) d(Q, AC) > m. The proof of Lemma is
complete.
We now consider the following two cases:
(1) Neither pair of the opposite
E
sides of the quadrilateral A B C D is
,,\%
parallel. We may assume that sides BC
-.-...
and AD meet at point F and sides B A
--.
-.
and C D meet at point E. Through
--..
’ F
C
point P draw segments ZI and 12 such
+
+
+
+
ljl.
I
\
________________>
that the sum of distances from any point on ZI to A B and C D is
constant and the sum of distances from any point on 12 to BC and AD
is also constant. As seen in the second figure, for any point Q in area
S, using the given condition and the lemma, we have
d ( P , AB) + d ( P , BC) + d ( P ,
CD)+ d ( P , DA)
It leads to a contradiction.
(2) The quadrilateral A B C D is a trapezoid. In the same way, we
can also show that it will lead to a contradiction.
This completes the proof.
Mathematical Olympiad in China
182
Second Day
9 :30 - 13 :30 September 28, 2003
@@& Let K be a given positive integer and {a, } a number sequence with
a. = 0 and
U,+l
=
Ka,
+&2
-
1>a:
+1 ,n
=
0, 1, 2,
..a.
Prove that every term of the sequence {a, } is an integer and
2K I a2, for all n . (posed by Zhang Zhenjie)
Solution We have
- 2Ka,~,+~
a: - 1 = 0 and
2Kan+lan+2 +:+I
- 1 = 0. Subtracting the first expression from the
second onet we obtain
+
a:+2
= (a,+2
-
2
a,
-a,>
-
2Ka,+1 an+2
(a,+2
+a,
+2Ka,a,+1
- 2KU,+l)
=
0.
Since {a,} is strictly monotonic increasing, then
an+2 = 2KU,+l
0
-a,.
From a0 = 0, a1 = 1and 0,
we get that every term of {a,} is an
integer.
Furthermore, from 0we have
2K I an+2 - a,.
0
From2K I a0 and 0,
we get2K I a,, n = 1, 2,
Remark Apparently, the recursive relation in {an} is nonlinear.
Actually it is not. When we encounter a problem involving a nonlinear
sequence, the first thing to do is to linearize and to simplify the problem.
Specifically, if it has an expression with radical terms, try to eliminate
these terms to get a linear recursive expression of order 2.
..a.
s?-?=!=.
%SF
Suppose the convex quadrilateral A B C D has an inscribed circle.
The circle touches A B , BC, C D , D A at Al , B1, Cl , D1
China Western Mathematical Olympiad 2003
183
A
respectively. Let points E , F , G ,
H be the midpoints of A1 B1 , B1 CI ,
CIDI , DlAl respectively. Prove
that quadrilateral EFGH is a
rectangle if and only if ABCD is a
cyclic quadrilateral. (posed by Feng
Zhigang)
Solution As seen in the figure, let
point I be the center of the inscribed circle of ABCD. Since H is the
midpoint of DIAl and lines AA1, AD1 are the two tangent lines
through A to the circle, then point H lies on the line segment AZ and
A l l A1 D1 . From ID1 1AD1and using the proportional theorem for
similar triangles we get that IH LA = ID: = r 2 ,where I is the radius
of the inscribed circle. In the same way, we get that IE IB = r 2 .
Then IE IB = IH LA, and that means points A, H, E, B lie on one
circle, so LEHI = L A B E. Similarly, we obtain that L I H G =
LADG, LIFE = L C B E , L I F G = LCDG. Adding these four
equations, we get t h a t L E H G + L E F G = LABC+LADC, and that
means A, B, C, D lie on one circle if and only if E, F, G, H lie on
one circle. Notice that quadrilateral EFGH is a parallelogram as E,
F, G, H are the midpoints of the sides of quadrilateral A1 B1 CI D1
respectively. So E, F, G, H lie on one circle if and only if EFGH is a
rectangle. This completes the proof.
, x5 be nonnegative real numbers with
@&@ Let XI, x2 ,
c
;=I
4+2
1
1+xi
= 1.
< 1. (posed by Li Shenghong)
z-l+xi
i=l
~
-
1
Solution Let y . - , i = l , 2,
We have
5
;=I
5
Prove that
c
..a,
5, thenxi=-,
1-yi
Yi
.
z=1,
Mathematical Olympiad in China
184
5
5
-Y? +Yi
5yi2 - 2yi 1 < 1
+
;=I
5
;=I
+
1-5~; 5 ~ i
<5
5yi - 2yi 1
w5(-l+5
+
3yi
+ 1 )<5
+1
Y? - 2Yi
i=l
3yi
+1
< 10.
Furthermore ,
5
4
=-
x (3+5)
=
10.
This completes the proof.
Arrange 1 650 students in 22 rows by 75 columns. It is known
that for any two columns, the number of occasions that two
students in the same row are of the same sex does not exceed 11.
Prove that the number of boy students does not exceed 928.
(posed by Feng Zhigang)
Solution Let ai be the number of boy students in the ith row, then
the number of girl students in this row is 75 - ai. By the given
c
22
condition, we have
CC;
1
r=l
0 5 .
<- 30 525, implying c(2ai
-
,=l
Inequality, we have
c
22
[
22
< 2 2 c (2ai
(2ai - 75>12
i= 1
-
1
22
75ai)
< 11 X That is, c
(a?
,=
7 5 ) 2 < 1 650. Using Cauchy’s
22
+
i= 1
-
75)2
< 36 300.
China Western Mathematical Olympiad 2004
c
c
i= 1
i= 1
22
Then
(2ai - 75) < 191, and
22
ai
< 19' +
185
650 < 921. That
means the number of boy students does not exceed 928.
Remark This problem was derived from a more general one: Given
a rectangular matrix with rz rows and m colums and consisting of
elements + 1 and - 1, if for any two colums, the sum of products of
two elements in the same row is less than or equal to 0, find an upper
bound of the number of elements + 1 in the matrix. In our problem,
n =22 as there are 22 teams attending the competition, and there is an
upper bound less than 928 when m = 75 as the competition date is
September 28.
2004
(Yinchuan, Ningxia)
The 4th (2004) China Western Mathematical Olympiad was held on
September 25 - 30, 2003 in Yinchuan, Ningxia, China, and was
hosted by Ningxia Mathematical Society and Ningxia Changqing
Yinchuan senior high school.
The Competition Committee consisted of the following: Xiong
Bin, Wang Haiming , Xu Wanyi, Liu Shixiong , Wang Jianwei, Zhang
Zhengjie, Wu Weichao, Feng Zhigang and Feng Yuefeng.
First Day
8 :00 - 12 :00 September 27, 2004
@@Find all integersn, such that n4 +6n3 + l l n 2 +3n+31 is a perfect
square. (posed by Xu Wanyi)
Solution SupposeA = n4 +6n3 + l l n 2 +3n+31 is a perfect square,
it means that A = (n2 3n 1>2- 3(n - 10) is a perfect square.
If n > 10, then A < (n2 3n 1>2,thus A
(n2 3 d 2 .
+ +
+ +
<
+
Mathematical Olympiad in China
186
Therefore
(n2+ 3 ? ~ + 1 )~ (n2 +3n)2
< 3n-30,
or
2n2 +3 n +3 1 <
0,
which is impossible.
If n = 10, thenA = (lo2 +3 X 10+1)2 = 1312is a perfect square.
If n< 10, thenA> (n2 + 3 ~ ~ + 1 ) ~ .
3 or 0 n < 10. Then n2 3n 0. Thus
Case I n
<-
<
+ >
A > (n2+ 3 ~ ~ + 2 ) ~ .
That is,
2n2 +9 n -2 7 <0 ,
or
Therefore, n=-6, -5, -4, -3, 0, 1, 2. For these values of n,
the corresponding values of A are 409, 166, 67, 40, 31, 52, 145. All
of them are not perfect squares.
Case II n =- 2, - 1. Then A = 37, 34 respectively, none of
them is a perfect square.
Hence, only when n = 10, A is a perfect square.
@@ Let ABCD be a convex quadrilateral, 11and I2 be the incenters of
A A B C and ADBC respectively. The line III2 intersects the lines
AB and DC at the points E and F respectively. Suppose lines AB
and DC intersect at P , and PE = PF. Prove that the points A, B ,
C, D are concyclic. (posed by Liu Shixiong)
Solution Draw line segments I l B , IIC, I2B and IzC, as shown in
the figure.
Since PE
=
P F , we have L P E F
=LPFE.
China Western Mathematical Olympiad
2004
187
But
LPEF =LIzIIB-/EBIl
=LI~IIB-LIIBC,
LPFE
=/I1
and
=/I1
IzC-LFCIz
Therefore, L I 2 I 1 B - L I 1 B C
LI2IIB+LI2CB
=L
I2C-LI2CB.
I I I ~ C - L I ~ C Bor,
= L I 1 I2C+LIIBC.
+
On the other hand, L I ~ I I B
L I z C B + / I 1 IzC+LIlBC = 2x. Thus
L 1 2 1 1 B+ L 1 2 C B = x , and points I1 ,
12, C , B are concyclic.
Since L B I l C = L B I 2 C , we have
I1
A
,’\
E B
+1 1 2 CB ,
LI1 BC +LI1 CB
=
LABC+LACB
= LDBC+LDCB.
LI2 BC
P
and
Hence, L B A C
concyclic.
=LBDC,
@$!$ Find all real numbers K
a3 +b3
and the points A , B , C, D are
, such that the inequality
+ 2 +d3
+ 1> K(a+b+c+d)
E [- 1, +m>. (posed by Xu Wanyi)
3
Solution I f a = b = c = d = - 1 ,
then-3>K*(-4).
HenceK>-. 4
holds for anya, b ,
c, d
Ifa=b=c=d=-,
1
1
then4*-+l>K*
2
8
and so K
=
-.34
Now we want to prove that the inequality
3
1
( 4 . ~ ) .ThuskG-,4
Mathematical Olympiad in China
188
holds for anya, b , c , d E [-1, +m>.
At first, w e p r o v e t h a t 4 2 + 1 > 3 x , x E [-1, +m>.
In fact, from (x 1)(2x- 1>2 0, we have 4 2 1 3x, x E
[- 1, 00). Therefore
+
+
+>
>
4a3 +1 >.a,
4b3
+ 1> 3b,
4 2 +1>3c,
4d3
+ 1 > 3d.
By adding the above 4 inequalities together, we get the inequality
(1).
Thus, the real number we want to find is K
=
-.34
Remark To prove an inequality of the above type, it is natural that
we prove an in equality for each variable separately and then add
them together.
@$$$ Let n E N (the set of positive integers) , and d(n) be the number
of positive divisors of n. Next, p(n> denotes the number of
integers in the closed interval [l , n] which are co-prime with n.
Find all non-negative integers c , such that there exists n E N
satisfying
d(n)
+ p(n>
=
n+ c.
For such c , find all rz satisfying the above equation. (posed
by Feng Zhigang)
Solution We denote the set of positive divisors of rz by A, and the
set of integers in the closed interval [l , n] which are co-prime with rz
by B. Since there is only one number 1 E A B among 1, 2 , , n , we
get d(n) p(n> n 1. Thus c = 0 or 1.
(1) If c = 0, then d(n) p(n> = n implies that there exists only
n which is not contained inA U B. If rz
one number among 1, 2,
is even, and n >8, then all of n- 2 and n -4 are not contained in A U
+
n
< +
+
..a,
China Western Mathematical Olympiad 2004
189
B. In this case, rz does not satisfy the equation. If rz is odd, then
when rz is a prime or 1, d ( n ) p(n> = n+ 1 (which will be discussed
in case ( 2 ) below). When rz is a composite number, we can write n =
p q , where p and q are odd and 1<p q. If q 5 , then 2 p and 4 p are
not contained i n A U B, so rz does not satisfy the equation.
From the above discussion, we find that
+
<
d(n)
+ p(n>
<
=
n
<
when either n 8 and rz is even, or n 9 and rz is an odd composite
number.
By direct verification of all the solutions of the above equation, rz
can only be 6, 8 and 9.
( 2 ) I f c = 1, thend(n)+dn) = n+limplies that amongl, 2 ,
n every number belongs to A U B. It is easy to find that this time n = 1and
primes can satisfy the required condition. For the case when rz is even (not
prime) , by the same argument as above we have n< 4 (consider n-2 ) . If
rz is an odd composite number, write n = p q , where p and q are odd and
3<p
q , then 2 p @ A U B, a contradiction. By direct calculation we find
that only n = 4 satisfies the condition.
Therefore, the solutions of the equation d ( n ) p(n> = n 1 are
n = 1, 4 or primes.
Remark A B = (1)is clear, thusc< 1. When rz is a prime, it is
easy to see that d ( n ) + p ( d = n + l . When rz is a composite number,
for any composite number rz which is large enough it is easy to find
sufficiently many composite numbers less than rz such that they are
neither divisors of rz nor coprime with n. In this case d ( n ) p(n> #
n +c ( c = 0 or 1). The general case of this problem is to remove the
restriction of c being non-negative.
..a,
<
+
+
n
+
Second Day
8 :00 - 12 :00 September 28, 2005
@&@ The sequence {a, } satisfies a1
= a2 =
1 and
Mathematical Olympiad in China
190
an+2
=
1
%+I
~
+a,,
n = 1, 2,
..a.
Find a2004. (posed by Wu Weichao)
Solution According to the assumption we have
an+2an+1 -an+1a,
=
1.
Thus, {U,+~U,} is an arithmetic progression with first term 1 and
common difference 1. Hence
an+lan= n , n = 1, 2,
so an+2 = n + l
~
-
%+I
n+l
n
-
-
~
..a.
n+l
a , , n = 1, 2,
n
~
2 003
a2 004 = m a 2 002 =
2003
2002
-
3 5
2 4
~
...
Consequently,
2001
m a 2 000
=... = 2003
.2001
2 002 2 000 ...
~
..a.
3
--a
2 2
2 003
2 002'
%$@@ All the grids of an m X n chessboard ( m
>3 , n >3 ) are colored
either red or blue. Two adjacent grids (with a common side) are
called a good couple if they are of different colors. Suppose that
there are S good couples, explain how to determine whether S is
odd or even. Does it depend on certain specific color grids?
(Reasoning is required. ) (posed by Feng Yuefeng)
Solution I Classify all grids into three parts: the grids at the four
corners, the grids along the borderlines ( not including four
corners) , and the other grids. Fill all red grids with label number 1,
all blue grids with label number - 1. Denote the label numbers filled
in the grids in the first part by a , b , c and d, in the second part by
XI 9 x 2 9
, XZmf2-8 , and in the third part by y1 , y2 , ,
~(~-2)(-2).
For any two adjacent grids we write a label number
...
China Western Mathematical Olympiad 2004
191
which is the product of two label numbers of the two grids on their
common edge. Let H be the product of all label numbers on
common edges.
There are 2 adjacent girds for every grid in the first part, thus its
label number appears twice in H. There are 3 adjacent grids for every
grid in the second part, thus its label number appears three times in
H. There are 4 adjacent grids for every grid in the third part, thus its
label number appears four times in H. Therefore,
If X Ix2 "'X2&2-8
= 1, then H = 1, and in this case there are
even good couples. If X Ix2 .-x2&24 =- 1, then H =- 1, and in this
case there are odd good couples. It shows that whether S is even or
odd is determined by colors of the grids in the second part. Moreover,
when there are odd blue grids among the grids in the second part , S is
odd. Otherwise S is even.
Solution I[ Classify all grids into three parts: the grids at the four
corners, the grids along the borderlines (not containing four
corners) , and the other grids.
If all grids are red, then S = 0, which is even. If there are blue
grids we pick any one of them, say A, and change A into a red one.
We call this changing a transformation.
(1) A is a grid in the first part. Suppose that there are K red grids
and 2 - K blue grids among A's two adjacent grids. After changing A
into a red one, the number of good couples increases by 2 - K - K =
2 - 2K. It follows that the parity even or odd of S is unchanged.
( 2 ) A is a grid in the second part. Suppose that there are p red
grids and 3 - p blue grids among A's three adjacent grids. After
changing A into a red one , the number of good couples increases by
3 -p - p = 3 - 2p. It follows that the parity of S is changed.
(3) A is a grid in the third part. Suppose that there are q red
192
Mathematical Olympiad in China
grids and 4 - q blue grids among A’s four adjacent grids. After
changing A into a red one , the number of good couples increases by
4 -q - q = 4 - 2q. It follows that the parity of S is unchanged.
If there are still blue grids on the chessboard after above
transformation, we continue doing the transformation over and over again
until no blue grid left on the chessboard. Now S is changed into 0.
Clearly, S changes its parity odd times if there are odd blue grids
among the second part of grids. Similarly, S changes its parity even
times if there are even blue grids among the second part of grids. It
implies that the parity of S is determined by the coloring of the second
part of grids. When there exist odd blue grids among the second part
of grids, S is odd. When there exist even blue grids among the second
part of grids, S is even.
Remark In Solution I the method of evaluation is used, and in
Solution II the method of transformation is used. These two methods
are used quite often in solving this kind of problems.
Let 1 be the perimeter of an acute triangle AAB C which is not
equilateral, P a variable point inside AABC , and D , E and F be
projections of P on B C , C A and A B respectively.
Prove that
2(AF+BD+cE) = I ,
if and only if P is collinear with the incenter and circumcenter of
AABC. (posed by Xiong Bin)
Solution Denote the lengths of three
sides of AABC by BC = a, CA = band
Y
h
*
AB = c respectively. No loss of
generality, we can suppose b # c. We
,’ (+,Y)
choose a rectangular coordinate system
‘,
(see the figure) , then we haveA(m, n ) ,
C(a, 0 )
B(0, O ) , C(a, 0) a n d P( x , y ) .
SinceAF2 - BF2 = A P 2 - BP2, it follows that
193
China Western Mathematical Olympiad 2004
AF2 - ( c - A F ) ~ = AP2 - BP2.
Therefore
2c
AF - 2 = ( x - m ) 2
+( y -
n)2
-2
-
y2,
and
AF=
m2 +n2 -2mx-2ny
2c
+ y.c
On the other hand, from
CE2-Ap=pc2--Ap2,
we have
CE2 - ( b - C E ) 2
=
PC2-AP2.
Thus
.
2b CE - b2
CE=
Since AF
2
=
( x- a )
+y2
-
( x- m)
-
2m+2ny-m2-n2-2m+a2
2b
+BD + CE
m2 +n2 -2mx-2ny
2c
+-b
=
(y
-
n)
,
+y.b
1
, we get
2
=-
2mx +2ny -m2
+$+x+
-
n2 - 2 m +a2
2b
l
-,
2
that is,
Since b # c and n # 0, point P is on a fixed straight line. Since the
CE) = 1 is satisfied for both incenter and
condition 2 ( A F BD
circumcenter, we complete the proof.
+
+
Il.Pathematica1Olympiad in China
194
Remark
The method we used above is
analytic. Another solution using a purely
geometric method is as follows: construct
projections of the incenter, circumcenter of
A A B C and point P on the three sides of
A A B C respectively, we can get the result by
means of proportional segments (see the
figure).
aSuppose that a, b,
1<d
c are
A
2
C
positive real numbers, prove that
ma + d $ bq ? + d Cm
343
< 2.
(posed by Wang Jianwei)
Solution S e t x = - 4
, y = - ,2
b2
a2
a2
z=-,
2
thenx, y , z E R + a n d x y z =
1. It suffices to prove that
Without loss of generality, we assume that x
xy, we havez
=
u=
-,
A<
A
~
l+la.
< y < z. Set A
=
1. Thus
1
Let u =
and only if
1
(O,
Hence
&
a
]
, andx
=a
i f
China Western Mathematical Olympiad 2005
195
= I + ( I -A)u2 +2u.
Set f(u> = (1 -A)u2
+2u + 1, we see that f(u) is an increasing
] , which implies that
function on u E
Now set
a=v, we get
2005
(Chengdu, Sichuan)
The 5 th (2005) China Western Mathematical Olympiad was held on
Mathematical Olympiad in China
196
November 3 - 11, 2005 in Chengdu, Sichuan, China, and was
hosted by Sichuan Mathematical Society and Chengdu No. 7 middle
school.
The Competition Committee consisted of the following: Li
Shenghong, Tang Xianjiang, Leng Gangsong, Li Weigu, Zhu
Huawei , Weng Kaiqing , Tang Lihua and Bian hongping.
First Day
8 :00 - 12:00 November 5, 2005
aAssume that C?
Oo5
+p2 Oo5 can be expressed as a polynomial in a+
Band ap. Find the sum of the coefficients of the polynomial.
(posed by Zhu Huawei)
Solution I In the expansion of a’+@, let a+p= 1and ap= 1. We
get the sum of coefficients s k = a k + p k . Since
<a+p><ak-l+Pk-l)
=(ak+pk)
we get
Thus s k
+ap(~r~-~+p~-~),
sk =
= S,
s&l-s&2.
and { s k } is a periodic sequence with period 6 and
1.
Solution I[ Set a+p= 1and ap = 1in the expansion of a k + @ . The
sum of the coefficients is s k = a k + p k . Since a, pare solutions of the
s2005 = s
1=
equation2 - X + I
= 0,
x
a = cos-+
3
i sin-,x p= cos--x
3
i sin-.x
3
3
Therefore
ak+pk=
(COST+
=(COSTk
+x
kx
=2cos -.
3
i sin-; ) k + ( cos--x3
i
kx
.
I sin3
)k
China Western Mathematical Olympiad 2005
Let K
=
2 005, we have S k
=
19
1.
As shown in the diagram, P A , P B are two tangent lines of a
circle from a point P outside the circle, and A , B are the contact
points. PD is a secant line, and it intersects the circle at C and
D. B F parallels P A and meets the lines A C , AD at E , F
respectively. Prove that BE = BF.
(posed by Leng Gangrong)
Proof
Join B C , B A and BD, then
L A B C = / P A C = LE. Thus A A B C c/)
BC A C
D
a A E B and - = -, that is,
BE A B
BE
=
A B BC
AC '
Since L A B F = L P A B = L A D B , we have A A B F
BF A B
Therefore - = -, that is,
BD AD
BF
=
c/)
A B BD
AD '
AADB.
0
On the other hand, since A P B C c/)A P D B and A P C A c/)A P A D ,
we get
BC - -PC
AC
_
andBD-PB
AD
Since P A
=
=
-.PC
PA
P B , we have
0
BC
Thus-=-.
AC
BD
B y O , O a n d 0 , wehaveBE=BF.
AD
@& Let S = { 1, 2,
2 005). If there is at least one prime number
in any subset of S consisting of rz pairwise coprime numbers, find
..a,
Mathematical Olympiad in China
198
the minimum value of n . (posed by Tang Lihua)
Solution First we prove n 16. In fact, let
>
&=
(1, 22, 32, 52,
.a*,
412, 432},
where the members in A0 , other than 1, are the squares of prime
numbers not greater than 43. Then & C S, I & I = 15 and the
numbers in & are pairwise coprime but & contains no prime number.
Thus n 16.
Next we show that for arbitrary A C S with n = I A I = 16, if the
numbers in A are pairwise coprime, then A must contain a prime
number.
In fact, if A contains no prime number, denoteA = { al , a2 , ,
< a16 }. Then there are two possibilities.
a16 ; a1 < a2 <
(1) If 1 @ A, then a1 , a2 ,
a16 are composite number. Since
(ai , a j ) = 1 (1
i <j
16) , the prime factors of ai and aj are
pairwise distinct. Let pi be the smallest prime factor of ai. We may
assume that p l < p2 < < P I 6 , then
>
<
UI
>p f >22
a2
<
..a,
>p$ > 32
a15
> > 472 > 2 005
pf5
it leads to a contradiction.
(2) If 1 E A , let a16 = 1, a1 , a2 , , a15 are composite numbers.
By the same assumption and argument of (1) , we have al p f 22 ,
> >
>
> >
>
p$ 32, .-, a15 p f 5 472 > 2 005. Again, it leads to a
contradiction.
From (1) and (2) , A contains at least one prime number, i. e.
when n = IAl = 16, the conclusion is true. Thus, the minimum
number of n is 16.
a2
@&@It is given that real numbers x1, x2,
I pxil>l,lxil<l
( i = l , 2,
i=l
a positive integer K such that
xn ( n
> 2)
satisfy
n). Provethatthereexists
I 5xi 5xi I < 1. (posed by
-
i=l
Leng Gangsong)
..a,
..a,
i=Wl
China Western Mathematical Olympiad 2005
k
n
199
n
n
g(n>= C X i .
i=l
I g(l)
Then
Ig(K+I)-g(K)I
-
=
I <2 ,
g(0) I = 2 I XI
21x~+lI<2,
K
=
I, 2,
.-, n - 2 ,
Ig(n>-g(n--)I=21X~2=,<2.
So for each 0
< K < n-
1,
I g(K+l) -g(K) I < 2.
0
If the conclusion is not true, by the condition for each K , O < K <
n , we have
I g(K) I > 1.
0
If thereisani, O<i<n-1, such thatg(i)g(i+l)<O, wemay
assume that g(i) >0 and g(i 1) <0. By 0,
g(i) >1 and g(i 1) <
- 1. Thus I g(i
1) - g(i>I > 2. This contradicts 0.
Thus g(O),
g(n> = 0. The
g(l), .-, g(n> have the same sign. But g(0)
contradiction implies that the conclusion is true.
+
+
+
+
Second Day
8 :00 - 12 :00 November 6, 2005
@&f@The circles O1 and O2 meet at points A and B. The line D C
passes through 0, , intersects the circle O1 at D and is a tangent
to the circle O2 at C . Also, C A is a tangent to the circle O1 at
A . The secant AE of the circle O1 is perpendicular to D C . AF is
perpendicular to and meets DE at F.
Prove that BD bisects the line
segment AF. ( posed by Bian
Hongping)
Proof Let AE intersect D C at point H ,
Mathematical Olympiad in China
200
and AF intersect BD at point G. Join AB, BC, BH, BE, CE and
GH. By symmetry, CE is also a tangent line of the circle 01 and H is
the midpoint of AE.
Since L H C B = L B A C andLBAC = L B E H , we haveLHCB =
L H E B . Thus H, B, C, E lie on the same circle, and L B H C =
LBEC.
From
we get
Since AF
LBEC
=LBDE,
LBHC
= LBDE.
0
1DE , we have
LAGB
=
2
0
-LBDE.
By 0,
0and 0,
LAGB = L A H B . Therefore A , G , H , B lie
on the same circle, andLAHG = LABG = L A E D . Thus G H I D E .
Since H is the midpoint of AE, G is the midpoint of AF.
@@ In an isoseles right angled triangle A A B C , C A = CB = 1, and P
is an arbitrary point on the perimeter of A A B C. Find the
maximum value of P A P B PC. (posed by Li Weigu)
Solution
(1) In the first diagram, if P E A C , we
1
Thus P A
have P A PC 7 and PB
<
PB
PC
< ./z
-.4
<&.
A
A
B
The equality is not valid,
since the two equality signs cannot be valid at the same time.
Therefore P A P B
PC
<&
T.
(2) In the second diagram, if P E A B , writeAP
=x
E [o,
&] ,
201
China Western Mathematical Olympiad 2005
then
f(x)
=
PA2 P B 2 PC2
-fix>.
=&fi-x)2(1+2
Note that g ' ( t )
[0, $1
=
2t - 3t2
<
and f<x> g(
=
A
P
B
t ( 2 - 3t). Thus g ( t ) is increasing on
3) i.
=
Therefore PA P B
PC
< 2 1f i
~
=
1
fi
The equality is valid if and only if t = - andx = -. So P is the
2
2
midpoint of AB.
2/z
4'
%g
.=7&
Given real numbers a , b , c , satisfying a
10(a3 +b3
=
1, prove that
+ 2 >-9(a5 +b5 + 2 >> 1. (posed by Li Shenghong)
Solution Since
b)[Ca2
+b +c
Xu3 =
1 - 311(a
+ b), X u 5
=
1 - 511(a
+
+C 4
therefore, the original inequality holds
Cub)]>
wlo[1-3II~n+~~l-9[1-5II~a+~~~~a~+1
w45II(a
+b) ( C u2 + C a b ) > 3 0 m a +b)
w3(Ca2+ C a b ) 2 2
=2
(Ca)
-
2(Ca2+2Cab)
w Z a 2> C a b .
+@> 2ab, @ + 2 > 2bc and 2 + u2 > 2 a c , we have
2 C a 2 > 2 C a b , i. e. , X u 2 > Cab. Therefore the original
From u2
inequality holds.
There are rz new students. Suppose that there are two students
Mathematical Olympiad in China
202
who know each other in every three students and there are two
students who do not know each other in every four students. Find
the maximum value of n . (posed by Tang Lihua)
Solution The maximum value of n is 8.
A
A
When n = 8, the example shown in the
diagram satisfies the requirements, where Al ,
A2 ,
As represent 8 students. The line
A
segment between Ai and Aj means Ai and Aj
know each other.
Next, if n students satisfy the conditions, we want to show that
n< 8. To do this, we first prove that the following two cases are
impossible.
(1) If someone A knows at least 6 persons, denoted by B1,
B2 , , B6. By Ramsey’s theorem, there exist 3 persons among them
who do not know each other. This contradicts that there are two who
know each other in every three students, or that there exist 3 people
they know each other. A and the three persons form a group of four
persons such that every two of them know each other. A
contradiction.
(2) If some one A knows at most n - 5 persons, then in the
remaining there are at least 4 persons, none of whom knows A . Thus
every two of the four persons know each other, a contradiction.
When n 10, one of (1) and (2) must occur. So such n does not
satisfy the requirements.
If n = 9, in order to avoid (1) and ( 2 ) , each person knows
exactly 5 other persons. Thus the number of pairs knowing each other
..a,
@ N. This contradiction implies n
2
value of n is 8.
is
~
< 8.
Thus the maximum
International Mathematical
Olympiad
T h e international Mathematical Olympiad, founded in 1959, is one
of the most competitiw and highly intellectual activities in the
world. Till now, there are more than 90 countries and areas that
take part in it. IMO, hosted by each participating country in turn, is
held in mid-July ewry year. The competition lasts for 2 days, and
there are 3 problems to be completed within 4.5 hours each day.
Each question is 7 marks which total up to 42 marks. The full
score for a team is 252 marks. About half of the participants will be
awarded a medal, where 1/12 will be awarded a gold medal. The
numbers of gold, silver, and bronze medals awarded are in the
ratio of 1 : 2 : 3 approximately. In the case when a participant
provides a better solution than the official answer, a special award
is given.
All participating countries are required to send a delegation
consisting of a leader, a deputy leader and 6 contestants. The
204
Mathematical Olympiad in China
problems are contributed by the participating countries and are
later selected carefully by the host country for submission to the
international jury set up by the host country. The host country does
not provide any question. Then the problems are translated in
working languages and the team leaders will translate them into
there own languages.
The answer scripts of each participating team will be marked
by the team leader and the deputy leader. The team leader will
later present them to the coordinators for assessment. If there is
any dispute, the matter will be settled by the jury. The jury is
formed by the various team leaders and an appointed chairman by
the host country. The jury is responsible for deciding the final 6
problems, finalizing the marking standard, ensuring the accuracy
of the translation of the problems, standardizing replies to written
queries raised by participants during the competition,
synchronizing differences in marking between the leaders and the
coordinators and deciding on the cut-off points for the medals
depending on the contestants’ results as the difficulties of
problems each year are different.
2003
(Tokyo, Japan)
The 44th IMO (International Mathematical Olympiad) was hosted by
Japan in Tokyo during July 7 - 19 in 2003
The leader of Chinese IMO 2003 team was Prof. Li Shenghong
who was from Zhejiang University and the deputy leader was Feng
Zhigang who was from Shanghai High School. In IMO 2003, China
came second among the nations, with two golds, one silver. Here are
the results:
International Mathematical Olympiad
Fu Yunhao
the High School Attached to
Tsinghua University
Wang Wei
the High School Attached to
Hunan Normal University
Xiang Zhen
the First High School of
Changsha, Hunan
2003
205
36 points
gold medal
I
Fang Jiacong
the Affiliated High School of
South China Normal University
Wan Xin
35 points
gold medal
Sichuan Pengzhou Middle School 33 points
gold medal
No. 3 High School of WISCO ( a
second-year student)
Zhou You
28 points silver medal
First Day
9:OO - 13:30 July 13, 2003
, 1000 000) containing
exactly 101 elements. Prove that there exist numbers t l , t 2 ,
t100 in S such that the sets
~2 Let A be a subset of the set S = { 1 , 2,
-.=-s
i
..a,
Aj
=
{x+tj I x E A} f o r j = 1, 2,
..a,
100
are pairwise disjoint.
Solution Consider the set D = {x-y I x, y E A}.There are at most
101 X 100 1 = 10 101 elements in D. Two sets Ai and Aj have
nonempty intersection if and only if ti - t j is in D. So we need to
choose the 100 elements in such a way that the difference for any two
elements is not in D.
Now select these elements by induction. Choose one element
arbitrarily from S. Assume that K elements, K
99, are already
chosen. An element x in S that is already chosen prevents us from
selecting any element from the set x D, where x D = { x y I y E
D } . Thus after K elements are chosen, at most 10 l O l k
999 999
+
<
+
+
+
<
Mathematical Olympiad in China
206
elements in S are forbidden. Hence we can select one more element.
Remark The size I S I = lo6 is unnecessarily large. The following
statement is true:
If A is a K -element subset of S = { 1, 2 , , n } and m is a positive
integer such that n > (m- 1) ( ( t ) + l ) ,thenthereexisttl,
SsuchthatthesetsAj= { x + t j I x E A ) f o r j = l ,
disjoint.
..a,
..a,
t,E
marepairwise
@
BDetermine all pairs of positive integers (a, b) such that
U2
2ab2 - b3
+1
is a positive integer.
Solution I Let (a, b) be a pair of positive integers satisfying the
+
> 0, we have 2ab2 -b3 1 > 0 or
2ab2 - b3 1
b
1
a >-, and hence a -.2b Using this, we infer from K 1, or
2 2b2
condition. Since K
U2
=
>
~
u2 >b2
+
(2a - b)
>
+ 1, that u2 > @ (2a
-
a>bor2a=
>0. Hence
b)
0
b.
Now consider the two solutions al , a2 of the equation
a2 - 2Kb2a
+ K(b3
-
0
1) = 0
for any fixed positive integers K and b , and assume that one of them is
an integer. Then the other is also an integer because a1 +a2 = 2Kb2.
We may assume that al
a 2 , and we have al
Kb2 > 0.
>
Furthermore, since ala2
Together with
= K(b3 - 1)
>
, we get
0,
we conclude that a2
=
0 or a2
=
b ( in the latter
2
-
International Mathematical Olympiad
2003
207
case b must be even).
If a2 = 0, t h e n d
Ifa2
-
1 = 0 , and henceq
b2
b
=-,
2
b4
=---.
2
thenK=-andal
4
= 2K
andb = 1.
b
2
Therefore the only possibilities are
( a , b)
=
(21, 1 ) , ( I , 21) or (8z4- I , 21)
for some positive integer 1. All of these pairs satisfy the given
condition.
Solution I[
be even.
Let
If b = 1 , it follows from the given condition that a must
U2
2 d 2 - b3
+1
=
K. If b > 1 , then there are two solutions to
the equation 0 and one of them is a positive integer. Thus the
discriminant a of the equation 0 is a perfect square, that is a =
4k2b4 -4Md - 1 ) is a perfect square.
Note that, if b 2 , we have
>
(2Kb2-b-02
<a<
0
(2Kb2-b+1)2.
The proof is given as follows,
a - ( 2 / ~ b ~ - b - l ) ~ = 4Kb2-b'-2b+4K-1
=(4K-l)(b2+1)-2b
>2(4K
-
1)b - 2b
>0
( 2 l ~ b ~ - - b + l ) ~ - A = 4Kb2-44K-(b-1)2
=4K(b2
-
1 ) - ( b - 1>2
>(4K-l)(b2-1)
>O,
this completes the proof of 0.
Since a is a perfect square, it follows from 0that
a = 4K2b4 - 4K(d
-
1)
=
(2Kb2 - b y .
Mathematical Olympiad in China
208
Then 4k = b2 , and hence b must be even. Let b = 21. We have
k = Z 2 . Together with 0,
we have a = 1 or 8Z4 - 1.
Therefore the only possibilities are
( a , b)
=
(21, 1) , ( I , 21) or (8z4 - I , 21)
for some positive integer 1. All of these pairs satisfy the given
condition.
A convex hexagon is given in which any two opposite sides have
the following property: the distance between their midpoints is
fi
times the sum of their lengths. Prove that all the angles of the
2
hexagon are equal.
(A convex hexagon ABCDEF has three pairs of opposite
sides: A B and DE, BC and EF, CD and F A . )
Proof I We first prove the following lemma.
Lemma Consider a triangle PQR with LQPR>60". Let L be the
2QR . The equality holds if and only if
midpoint of QR . Then PL< 43the triangle PQR is equilateral.
Let S be the point such that the
triangle QRS is equilateral, where
the points P and S lie in the same
,' ;'p
,,/'
I//
;
,
1
I1
I
I
Q
\,
L
5
,,/R
I
'.___-,'
half-plane bounded by the line Q R .
Then the point P lies inside the circumcircle ( including the
circumference of the circle) of the triangle QRS , which lies inside the
circle with center L and radius J3
-QR.
2
,1
lemma.
The main diagonals of a convex
hexagon form a triangle though the
,'
%',
P
,' I
,,' (I
,
D
N
\\\
\
E
International Mathematical Olympiad
2003
209
triangle can be degenerated. Thus we may choose two of these three
diagonals that form an angle greater than or equal to 60". Without loss
of generality, we may assume that the diagonals AD and B E of the
given hexagon ABCDEF satisfy LAPB
60", where P is the
intersection of these two diagonals. Then, using the lemma, we
obtain
>
where M and N are the midpoints of A B and DE respectively. Thus
it follows from the lemma that the triangles ABP and DEP are
equilateral.
Therefore the diagonal CF forms an angle greater than or equal
to 60" with one of the diagonals AD and BE. Without loss of
generality, we may assume that LAQF
60", where Q is the
intersection of AD and CF. Arguing in the same way as above, we
infer that the triangles AQF and CQD are equilateral. This implies
that L B R C = 60", where R is the intersection of B E and CF. Using
the same argument as above for the third time, we obtain that the
triangles BCR and EFR are equilateral.
This completes the proof.
Proof II Let ABCDEF be the given c
>
-
+
hexagon and let a =AB , b = BC,
+
.\
,f =
FA.
D
d
N
/
E
Let M and N be the midpoints of the
sides A B and DE respectively. We have
1
1
IWV- = -a+b+c+-dandIWV
2
2
Thus we obtain
-
1
IWV = -(b+c-e2
From the given property, we have
=--a-
2
f).
f-e--d. 1
2
0
Mathematical Olympiad in China
210
0
Set x = a-d,
y
=
c-f,
z
=
IY-zl
From
e-b.
0and 0,
we obtain
>43lxl.
0
Similarly, we see that
I Z - X I >&I
IX-Yl
YI
0
9
>43lzl.
0
Note that
0wIy12-2y*z+
1z12 > 3 I x I 2 ,
@wIz12-2z.x+
1xp>31y12,
0 w I x 1 2 - 2 x * y + Iy12 >31z12.
By adding up the last three inequalities, we obtain
-
I XI 2
-
I y 12
-
I z 12
-
2y z - 22 x- 2x y
> 0,
or- Ix+y+z12 2 0 . T h u s x + y + z = Oand the equality holds in
every inequality above. Hence we conclude that
x+y+z
= 0,
Iy-zl=&Ixl,a//d//x,
Iz-xI = & l Y l ,
Ix-yl=&lzl
+
RP
c//
f / / Y,
,e//b//z.
-
Suppose that P Q R is the triangle such that PQ
= z.
QR
=
=
We may assume LQPR >60", without loss of generality. Let
L be the midpoint of Q R . Then PL
=
- 211
z-XI
=
43
-1
2
yI
=
43
-QR.
2
It
follows from the lemma in Proof I that the triangle PQR is
equilateral. Thus we have L A B C = L B C D = = L F A B = 120".
International Mathematical Olympiad
2003
211
Second Day
9:OO - 13:30 July 14, 2003
.zg.g~
$-z
Let
A B C D be an inscribed quadrilateral. Let P, Q and R be the
feet of the perpendiculars from D to the lines B C , C A and A B
respectively. Show that P Q = Q R if and only if the bisectors of
L A B C andLADC meet on AC.
Proof By Simson’s Theorem, we know that
P , Q , R are collinear. Moreover, since L D P C
Q ,
a n d L D Q C are right angles, the points D, P,
Q , C are concyclic and so L D C A = L D P Q =
L D P R . Similarly, since D , Q, R , A are
C P
concyclic, we have L D A C = DRP. Therefore
A D C A c/)A D P R .
Likewise, A D A B c/)A D Q P and A D B C c/)ADRQ. Then
@
Thus P Q
=
QR if and only if
DA
DC
~
=
-.B A
BC
Now the bisectors of the angles ABC and ADC divide A C in the
BA
DA
ratios of - and -respectively. This completes the proof.
BC
DC
aLet rz be a positive integer and
< < <x n .
XI
, x2 ,
..a,
xn be real numbers
with x 1 x2
(a) Prove that
(b) Show that the equality holds if and only if x 1 , x2 ,
, xn is
Mathematical Olympiad in China
212
an arithmetic sequence.
Proof (a) Since both sides of the inequality are invariant when we
subtract the same number from all kis , we may assume without loss of
generality that
Cxi = 0.
i=l
We have
By the Cauchy-Schwarz inequality, we have
On the other hand, we have
Therefore
;=I
j-1
(b) If the equality holds, then there exists a real number k , such
that xi = K(2i - n - 1>,which means that x l , x 2 , .-, xn is an
arithmetic sequence.
On the other hand, suppose that XI , x 2 ,
xn is an arithmetic
sequence with common difference d. Then we have
..a,
xi
Subtract
n
2
= -((22-n-l)+d .
2
XI + x n
from everyxi, we obtainxi
C xi = 0, from which the equality follows.
i=l
2
.
d
2
.
= -(2z-n-l)
and
International Mathematical Olympiad
2004
213
@&jBLet p be a prime number. Prove that there exists a prime number
q such that for every integer n , the number np - p is not divisible
by 4.
Proof Since p p - 1
~
P--1
=
l + p + p 2 +.-+pP-l
can get a prime divisor q of
p-1
P--1
=p+l
such that q
(modp2 ), we
+ 1 (mod p 2 ) . This q
is what we wanted. The proof is given as follows.
Assume that there exists an integer rz such that np
=p
(mod q>.
2
Then we have np = pp = 1 (mod q > by the definition of q. On the
other hand, from Fermat's little Theorem, n e l =1 (mod q>, because
q is a prime. Since p 2 I q- 1 , we have gcd(p2 , q- 1 ) I p , which leads
to np = 1 (mod q >. Hence we have p = 1 (mod q >. However, this
implies 1 p
+pP-l = p (mod q>. From the definition of q , we
+ +
have0 =
p-1
P--1
=
1 +p+.-+pp-'
=p
(mod q ) , but this leads to
a contradiction.
2004
(Athens, Greece)
The 45th IMO (International Mathematical Olympiad) was hosted by
Greece in Athens during July 4 - 18 in 2004. 84 countries and 486
contestants participated. 45 gold medals, 78 silver medals, and 120
bronze medals were awarded.
The leader of Chinese IM02004 team was Chen Yonggao, deputy
leader was Xiong Bin, observers were Wu Jianping, Wang Shuguo. In
IM02004, China came first among the nations with six golds. Here
are the results:
214
Mathematical Olympiad in China
Huang Zhiyi
the High School Attached to
South China Normal University
41 points
gold medal
Zhu Qingsan
the High School Attached to
South China Normal University
38 points
gold medal
Li Xianyin
the High School Attached to
Hunan Normal University
37 points
gold medal
35 points
gold medal
35 points
gold medal
34 points
gold medal
Lin Yuncheng
Shanghai High School
Peng Minyu
No. 1 High
Yingtan, Jiangxi
Yang Shiwu
Hubei
School
School
Huanggang
of
High
First Day
9:OO - 13:30 July 12, 2004
Let A B C be an acute-angled triangle with A B # A C . The circle
with diameter B C intersects the sides A B and A C at M and N
respectively. Denote by 0 the midpoint of the side BC. The
bisectors of the angles B A C andMON intersect at R. Prove that
the circumcircles of the triangles BMR and CNR have a common
point lying on the side BC.
Solution We first show that the points A , M, R , N are concyclic.
Since ABC is an acute-angled triangle, M and N are on the line
segments A B and A C respectively. Let R1 be the point such that the
points A , M, R1, N are concyclic, where R1 is on the ray AR. Since
AR1 bisects L B A C , we have RIM = R1N. Since M and N lie on the
circle with centre 0, we have OM = ON. It follows from OM = ON
andRIM = R I N that R1 is on the bisector of L M O N . Since A B #
A C , the bisectors of the angles BAC and MON intersect at the unique
point R , and so R1 = R , or A , M, R , N are concyclic.
Let the bisector of L B A C meet BC at K. Since the points B , C ,
International Mathematical Olympiad 2004
215
N, Mare concyclic,LMB C = LANM. Moreover, since A, M, R , N
are concyclic, LANM = LMRA. This implies LMBK = LMRA.
Therefore, the points B, M, R , K are concyclic. Using the same
argument as above, we obtain that C, N, R, K are concyclic. This
completes the solution.
@BFind
all polynomials P ( x ) with real coefficients, which satisfy
the equation
+P(c-a)
P(a-b) +P(b-c)
= 2P(a+b+c)
+ + wz
for all real numbers a , b, c such that ab k
Solution Let P ( x ) satisfy the given equation.
thenP(0) = O .
Ifa=b=c=O,
If b = c
= 0,
= 0.
then P(- a )
=
P ( a ) for all real a.
Hence P ( x ) is even. Without loss of generality, we may assume
that
P ( x ) = a,x2n
If b = 2a,
($I22(
-
c
2
=--a,
3
$)']a2
=
+ ...+a1 2 , a, # 0.
we have that
0 for all a E R. Then all coefficients of the
polynomial with variable a are 0.
If n 3 , it follows from 86 = 262 144 > 235 298
(
+)2n
>
>(
+)6
> 2.
1
This implies
+ (+ ) + ($)
2n
2n
-
2 (?)
7 2n
> 0.
=
2 X 76 that
Mathematical Olympiad in China
216
<
Hence n 2. Let P ( x ) = ax4 +px2 , with a, p E R.
We now show that P ( x ) = ax4 +px2 satisfies the given equation.
Let a , b , c be real numbers satisfying ab k wz = 0. Then
+ +
( ~ - b )+ ~( b - ~ ) +~ ( c - u ) ~ - 2 ( a + b + ~ ) ~
=C(a4 -4u3b+6a2b2
=C(a4 -4u3b+6a2b2
-4ub3 + b 4 ) -2(a2 +b2 +C2)2
-4ub3 + b 4 )
=C(-4u3b+2a2b2 -4ab3)
=
4 2 (ab + wz) 4b2 (k+ ab)
-
-
-
-
4 2 (wz
+k)+
2(a2b2 +b22 + 2 a 9
+4@cu +4 2 a b +2a2b2 +2@2 +2 2 a 2
0,
=2(ab + bc +
=4a2 bc
C U > ~=
(a-b)2 +(b-c)2
=C(a2 -2ab+b2)
+(c-u)~ - 2 ( ~ + b + ~ ) ~
-2Ca2 -44ub
=0.
Hence P ( x ) = ax4 +b2
satisfies the given equation.
@& Define a hook to be a figure made up of six unit squares as shown
in the diagram or any of the figures obtained by
applying rotations and reflections to this figure.
Determine all m X n rectangles that can be
covered with hooks so that
the rectangle is covered without gaps and
without overlaps;
.no part of a hook covers area outside the rectangle.
Solution I m and rz should be the positive integers and should satisfy
International Mathematical Olympiad
2004
217
one of the following conditions: (1) 3 I m and4 I n (or vice versa) ; (2) one
of m and rz is divisible by 12 and one is not less than 7.
A figure is obtained by applying rotations and reflections to
another figure. We regard the two figures as equivalent.
Label the six unit squares of the hook as shown below. The
shaded square must belong to another hook, and it is adjacent to only
one square of this other hook.. Then the only
possibility of the shaded square is 1 or 6.
(9 If it is 6 , two hooks form a 3 X 4 rectangle. We
call it 0.
(ii) If it is 1, there are two cases.
It is easy to see that the shaded square cannot
be covered in the first diagram as shown below.
Hence the latter is true. We call it 0.
Thus, in a tessellation, all hooks are matched
0
into pairs. Each pair forms 0or 0.
Hence 12 I mn.
There are 12 squares in 0and 0.
p
J
0
Now we consider three cases, separately.
(1) 3 I m and 4 I n (or vice versa)
Without loss of generality, we may assume m = 3 q and n = 4%.
Thenmonorectangles of the type 0form a n q X rectangle. Since
two hooks cover a 3 X 4 rectangle, an m X n rectangle can be covered
with hooks.
(2) 12 I m or 12 I n. Without loss of generality, we may assume
12 I m.
If 3 I n or 4 I n, the question reduces to (1).
218
Mathematical Olympiad in China
Assume that rz is not divisible by 3 nor by 4. If a tessellation
exists, then there is at least one of 0 and 0 in it, so n 3. Hence
n 2 5 because 3 X n and 4 X n. Since the square at the corners can
belong to either 0or 0,
it follows from n 5 that the squares at the
Hence n
adjacent corners cannot belong to the same type 0or 0.
6. Since rz is not divisible by 3 and 4, n 7.
We now show that if n
7 and rz is not divisible by 3 and 4, a
tessellation exists.
If n= 1 (mod 3 ) , then n = 4+3t ( t E N" ). Together with (1) ,
we have that if 12 I m, an m X 3t, rectangle and an m X 4 rectangle can
be covered with hooks. So the problem can be solved.
If n = 2 (mod 3 ) , n = 8 3t ( t E N" >. Together with (1) , we
have that if 12 I m, each of m X 8 and m X 3t rectangles can be covered
with hooks. The problem is solved.
(3) 12 I m ,but neithermnor rz is divisible by 4. Now 2 I m, 2 I n.
We may assume without loss of generality that m = 6 q , n = 2% ,
neither q nor
is divisible by 2. We will prove that if these
conditions are satisfied, an m X n rectangle cannot be covered with
hooks.
Consider coloring the columns of an m X n matrix with black and
white colors alternately. Then the number of the black squares equals
that of the white ones. One 0 always covers 6 black squares. A
horizontal 0 always covers 6 black squares. A vertical 0 covers
either 8 black squares and 4 white ones, or 4 black squares and 8 white
ones. Since the number of the black squares equals that of the white
ones, the number of 0is the same in the preceding two cases. Hence
the total number of a vertical 0is even. Using the same argument as
above (coloring the rows alternately) , we obtain that the total
number of a horizontal 0is even.
Consider classifying the squares of the m X n rectangle into 4 types
marked 1, 2, 3, and 4 as shown below. The number of squares of
>
>
>
+
each type is equal to mn.
4
>
>
International Mathematical Olympiad
2004
219
From the two diagrams,
a
a
C
C
a
C
we obtain that the number of a and c covered by 0is the same, so is
for b and d. Hence the number of squares of type 1 covered by 0
equals that of type 3.
(9
(ii)
(iii)
(i.)
The number of squares of type 1 covered by (i) or (ii) equals that
of type 3. The difference between the number of squares of type 1 and
type 3 covered by (iii) or (iv) is 2. There are two cases: the number
of squares of type 1 is 2 more than that of type 3, or vice versa. Since
the number of squares of type 1 equals that of type 3 in the rectangle,
the frey nancy of the two cases is the same. Hence the total number of
(iii) and (iv) is even.
220
Mathematical Olympiad in China
Consider classifying the squares of the m X n rectangle as shown
below.
Similarly, the total number of (i) and (ii) is even. Then the number
of 0is even. So there is an ever number of 0and 0.
Hence 24 I m X n ,
contrary to the assumption that neither m nor rz is divisible by 4.
Solution I[ m and rz should be the positive integers and should
satisfy one of the following conditions: (1) 3 I m and 4 I n (or vice
versa); (2) 12 I m , n # 1, 2, 5 (or vice versa).
Consider a covering of an m X n rectangle satisfying the
conditions. For any hook A, there is a unique hook B covering the
“inner” square of A with one of its “tailend” squares. In turn, the
“inner” square of B must be covered by an “tailend” square of A.
Thus, in a tessellation, all hooks are matched into pairs. There are
only two possibilities to place B so that it does not overlap with A and
no gap occurs. In one case, A and B form a 3 X 4 rectangle; in the
other , their union is an octagonal shape, with sides of length 3 , 2 , 1,
2, 3, 2, 1, 2 respectively.
So an m X n rectangle can be covered with hooks if and only if it
can be covered with the 12-square tiles described above. Suppose that
such a tessellation exists; then mn is divisible by 12. We now show that
one of m and rz is divisible by 4.
Assume on the contrary that this is not the case. Then m and rz
are both even, because mn is divisible by 4. Imagine that the rectangle
is divided into unit squares, with the rows and columns labeled 1,
m and 1, , n. Write 1 in the square (i, j ) if exactly one of i a n d j is
divisible by 4, and 2, if i and j are both divisible by 4. Since the
..a,
International Mathematical Olympiad
2004
221
number of squares in each row and column is even, the sum of all
numbers written is also even. Now, it is easy to check that a 3 X 4
rectangle always covers numbers with sum 3 or 7; and the other
12-square shape always covers numbers with sum 5 or 7.
Consequently, the total number of 12-square shapes is even. But then
mn is divisible by 24 , and hence by 8 , contrary to the assumption that
m and rz are not divisible by 4.
Notice also that neither m nor rz can be 1, 2 or 5 (any attempt to
place tiles along a side of length 1, 2 or 5 fails). We infer that if a
tessellation is possible, then one of m and rz is divisible by 3, one is
divisible by4, andm, n @ (1, 2, 5).
Conversely, we shall prove that if these conditions are satisfied,
then a tessellation is possible (using only 3 X 4 rectangles). The result
is immediate if 3 divides m and 4 divides rz (or vice versa). Let m be
divisible by 12 and n @ (1, 2, 5) (or vice versa). Without loss of
generality, we may assume that neither 3 nor 4 divides n. Then n 7.
In addition, between n-4 and n-8 , at least one can be divisible by 3.
Hence the rectangle can be partitioned into m X 3 and m X 4 rectangles,
which are easy to cover, in fact with only 3 X 4 tiles again.
>
Second Day
9:OO - 13:OO July 13, 2004
>
Let n
3 be an integer. Let tl ,
numbers such that
t2,
..a,
tn be positive real
Show that t i , t j , t k are the lengths of the sides of a triangle for
alli, j , K w i t h l < i < j < K < n .
Solution Assume on the contrary that there exist three numbers
tn that do not form the sides of a triangle. Without
among tl , t2,
loss of generality, we may assume that these three numbers are t l , t 2 ,
..a,
Mathematical Olympiad in China
222
t3,
andtl+tz<tg.
=4-+-
t3
tl+t2
Ifx=-
t3
tl+t2
One has
tl+t2
t3
,t h e n x > l ,
+n2
-4.
(1)
1
and4~+--5
X
=
( ~ - 1 ) ( 4 ~ - 1 ) 2..
X
Together with (1) , we obtain that
(tl+
+t n >(G1 + +;)> 5 +
n2
-
4 = n2
+I ,
a contradiction. This completes the proof.
Remark By the AM-GM inequality,
1
which is the same as tl
+1>
4
-
-
t2
tl+t2'
@&!&
In a convex quadrilateral AB C D , the diagnal BD bisects neither
the angle A B C nor the angle C D A. The point P lies inside
AB C D and satisfies
International Mathematical Olympiad
LPBC
= LDBA
2004
223
a n d L P D C = LBDA.
Prove that ABCD is a cyclic quadrilateral if and only if AP = CP.
Solution I
(i) Necessity.
Assume that ABCD is a cyclic quadrilateral. Let the circle r be
the circumcircle of quadrilateral ABCD. Extend BP and DP beyond P
to meet the circle r at X and Y respectively.
Since DB does not bisect L A B C and P lies inside ABCD, it
follows from L P B C = LDBA that @ =
D # X, and the points
D and X lie in the same half-plane bounded by the line AC. Hence
DX I A C . Similarly, B # Y and BY // AC.
The points D, X, A, C, B, Y lie on the circle r, as mentioned
above. So the points D and X, the points A and C, the points B and Y
are symmetrical with respect to the perpendicular bisector 1 of A C
respectively.
Since P = DY BX , then P is on the line I , or AP = CP. This
completes the proof of the necessity.
(ii) Sufficiency.
Lemma. Let 1 be a fixed line and A , B , C be fixed points such
that A and B , C lie in the different half-planes bounded by the line 1.
Assume that the point X lies on the line 1 . Let a ( X > be the smallest
angle of rotation from the line XA to the line 1 anticlockwise. Let
P(x> be the smallest angle of rotation from the line xc to the line X B
m,
n
anticlockwise. If a ( X >= P ( X >, then X is called a good point. Prove
that there are at most two good points.
Proof of the lemma
We set up a coordinate system with the line 1 as the zaxis and
the perpendicular of 1 as the y-axis.
LetA(a, b ) , B ( c , d ) , C(e, f), X(x, 0). ThenA = a + b i , B =
c + d i , C = e + f i , a n d X = x. Hence
A--=a-x+bi,
+bi) (cos a(X>+ isin a(X>)
x) cos a(X> bsin a(X>+ [(a
(a - x
=
(a -
-
-
x) sin a(X>
+bcos a(X>]i.
Mathematical Olympiad in China
224
Rotate the line XA by a(X> anticlockwise, coinciding with 1. So
( a - x)cos a(X)
+bcos a(X>
+
-
=0
(1)
Moreover, since XC = e-x +fi, XB = c-x+di,
being rotated by p(X> anticlockwise becomes
+fi) (cos p( X>+ isin p( X>)
x)cos p(X) fsin p(X> + [(e
+
thenXCafter
(e - x
= (e
-
-
-
x)sin p(X>
+fcos p(X>]i
which is parallel to X B . Thus,
(c-x)[(e-x)sinB(X)
fsinp(X>] = 0,
+fcosp(X>]
-
d[(e-x)cosp(X)
-
and
[<c-x>(e-x)
+dflsinP(X>
+[(c-x)f-d(e-x)]cosp(X)
(2)
= 0.
Since sin 0 and cos 0 are not 0 at the same time, so
(1)(2)
X is a good point H a(X> = p(X> H the simultaneous equations
inuand v .
(a-x)u+bv
= 0,
[( c - x)(e- x)
+ df]u+
[( c - x)f- d(e- x)]v
=O
have the non-zevo solutions
+df] +(a-x)[(c-x)f-d(e-x)]
Hb[(c-x)(e-x)
H(b+d-
f>2
+ ( a f + cf- k -be
+ke+bdf
+ade-acf
+
-ad
-
=O
fd)x
= 0.
+
+
+
Let g ( x ) = ( b d - f>2 ( a f cf - k - be - ad - d x
k e +M f ade - acf . If b+d- f = 0, it follows from b # 0 that d #
f. Hence BC and 1 are not parallel. Let BC intersect 1 at T ( t , 0). We
have that p(T) = 0 and a ( T ) > 0. Hence T is not a good point. This
implies g(t># 0 , so the equation g ( x ) = 0 has at most two solutions.
+
International Mathematical Olympiad
2004
225
Hence there are at most two good points.
This completes the proof of the lemma.
Let the points A , B, C , D be arranged clockwise andAP = CP.
Let D * be the point such that A , D * , C , B are concyclic, where D *
lies on the ray BD. Since AP = CP and BD bisects neither L A B C nor
L A D C , BP intersects the perpendicular bisector of A C at the unique
point P.
Let D* R be the ray satisfying L C D * R = LAD * B and P * =
D* R BP.
Together with ( i ) , we have that P* A = P* C , or P* is the
intersection point of 1 and the perpendicular bisector of AC , so P * =
P. It follows from L C D * R = LAD * B that LAD * B = L C D * P.
Replace I , A , B, C by BD, A , C , P. Then B , D, D* are good
points. SinceB # D and B # D* , D = D* . Hence A , B, C , D is
concyclic.
This completes the proof of the sufficiency.
Combining (i) and (ii) , we obtain that the conclusion is true.
Solution I[ We may assume without loss of
A
generality that P lies in the rectangles A B C and
BCD.
Assume that the quadrilateral A B C D is
cyclic. Let the lines BP and DP meet A C at K B
and L respectively. It follows from the given
n
equalities and L A C B = L A D B , L A B D = L A C D that the triangles
D A B , DLC and CKB are similar. This implies L D L C = L C K B , so
L P L K = L P K L . Hence PK = PL.
It follows from L B D A = L P D C that L A D L = LBDC. Since
L D A L = L D B C , the triangles ADL and BDC are similar. Hence
AL-AD-KC
- BD - BE’
BC
yielding AL = KC. It follows from L D L C
L C K P . Moreover, since PK = PL and AL
L C K B that /ALP =
= K C , the triangles ALP
=
Mathematical Olympiad in China
226
and GYP are congruent. Hence AP = CP.
Conversely, assume that AP = CP. Let the
circumcircle of the triangle BCP meet the lines
CD and DP again at X and Y respectively.
B
It follows from LADB = L P D X and
L A B D = L P B C = L P X C that the triangles Y
ADB and PDX are similar. Hence
A
A~-~
D
BD
PD - X D
Since L A D P = LADB + L B D P = L P D X + L B D P
triangles ADP and BDX are similar. Theref ore,
BX
BD
=L B D X ,
XD
the
(1)
AP=AD=PD'
Since the points P, C , X , Y are concyclic, L D P C = L D X Y and
L D C P = L D Y X . Hence the triangles DP C and D X Y are similar.
Thus ,
Y
X D
_X-CP PD'
(2)
Since AP = C P , it follows from (1) and ( 2 ) that BX = YX. Hence
L D C B = LXYB = LXB Y = L X P Y = L P D X L P X D =
LADB L A B D = 180" - L B A D . The above equality means that
A B C D is a cyclic quadrilateral.
*
-*
2:
- We call a positive integer alternating if every two
+
+
--
consecutive digits in its decimal representation are of different parity.
Find all positive integers n such that n has a multiple which is
alternating.
Solution 1 Lemma 1 If k is a positive integer , then there exist 0
< a1 , a2 , .-, a2&9
such that a1 , a3 ,
, a2k are even integers, and
..a,
a2k-1 are odd integers,
a2 , a4 ,
22k+1I a1 a2-aa2k ( The decimal representation
Proof of Lemma 1 by mathemations induction.
)
International Mathematical Olympiad 2004
227
If K = 1, it follows from8 I 16 that the proposition is true.
Assume that if K = n - 1, the proposition is true.
When K = n , let a1 a2 .-a2-2 = 22n-1 t by the inductive hypothesis.
The problem reduces to proving that there exist 1 a , b 9 with a
odd and b even such that 22"t1 I a b X
+22"-1 t , or 8 I a b X 52n-2
2t, or 8 I ab+ 2t in view of 52n-2 = 1 (mod 8).
It follows from 81 1 2 + 4 , 81 1 4 + 2 , 81 1 6 + 0 and 8150+6 that
the Lemma 1 is true.
Lemma 2
If k is a positive integer, then there exists an
alternative number a1 a2.-a2k with an even number 2k of digits such
<
<
+
that a2k is odd and 52k I a l a2.-a2k, where a l can be 0 , but a2#0.
Proof of Lemma 2 by mathematical induction.
If K = 1, it follows from25 I 25 that the proposition is true.
Assume that if K = n-1 , the proposition is true, or there exists an
alternative multiple of a1 a2 .-a2-2 satisfying 52n-2 I a1 a2 .-a2-2.
When K = n , let a1 a2 .-a2-2 = t 52n-2. The problem reduces to
proving that there exist 0 a , b 9 with a even and b odd such that
52" I a b X
t 52n-2, or 25 I a b X 22n-2 t.
Since 22n-2 is coprime to 25 , there exist 0 < ab
25 such that
25 I ab X 22n-2 t. If b is odd, between ab and ab 50, at least one
satisfies that the highest-valued digit is even. If b is even, between
ab +25 and ab 75 , at least one satisfies that the highest-valued digit
is even.
This completes the proof of Lemma 2.
Let n = 2a5Pt, where t is coprime to 10 and a, BE N. Assume that
a
2 and p> 1. Let 1 be an arbitrary multiple of n. The last decimal
digit is 0, and the digit in tens is even. Hence these n do not satisfy
the required condition.
0 When a = p = 0, consider 21, 2 121, 212 121,
2 121.-21 ,.-. There must exist two of them congruent modulo n.
+
+
<
<
+
+
<
+
>
..a,
Y
nrm6er k
of
21
Without loss of generality, we may assume that t l >
t2
, and
Mathematical Olympiad in China
228
= 2 121.-21
2 121.-21
Y
tamzber
tl
21
of
nmzber
Then
of
t1-t2
t2
-
2 121.-21
of
OO.-O
Y
nrm6er
(mod n).
Y
21 tamzber 22,
of
21
=O
(mod n)
0
Hence
=0
2 121.-21
Y
nrm6er
t1-t2
of
(mod n ) ,
21
because rz is coprime to 10.
Now these positive integers rz satisfy the required condition.
@ When G, ' = 0 and a 1, it follows from Lemma 1 that there
exists an alternative number a1 a2 .-a2k satisfying that 2" I a1 a2 . - a 2 k .
Consider
>
a1 a2 .**a2k , ala2 .**a2kala2 .**a2k ,
,
There must exist two of them congruent modulo t. Without loss of
generality, we may assume that t 1 > t 2 , and
Since t is coprime to 10, a1a2...a2k...a1a2...a2k
Moreover, since t is coprime to 2,
=0
(mod t ) .
Y
mrm6er
t1*2
of
saiarF
which is alternating.
0When a = 0 , ,G' 1, it follows from lemma 2 that there exists
an alternative multiple a1 a2 .-a2k satisfying 5p I a1 a2 .-a2k with a2k odd.
Using the same argument as in 0, we obtain that there exist t 1 > t 2
satisfying t I a1a2...a2k...a1a2...a2k.
Since t is coprime to 5,
>
International Mathematical Olympiad
5't
I U l a 2 "'U2k"'Ul
a 2 "'U2k.
In addition,
2004
U l a 2 "'U2k "'Ul a 2 "'U2k
Y
mmzber
t1-t2
229
is
Y
of
seniarr
mmzber
t1-t2
of
LwAOns
alternating, and the last decimal digit a 2 k is odd.
@
When
Ia = 1 and ,8> 1, it follows from 0 that there exists an
alternative number a1 a 2 "'a2k ...a1 a 2 "'a2k satisfying that a 2 k is odd and
5't I
Hence 2
5't I a 1 a 2 " ' a 2 k " ' a 1 a 2 " ' a 2 k o
which is alternating.
In conclusion, if n is not divisible by 20, then these positive
integers n satisfy the required condition.
Solution I[
n should be the positive integers and should satisfy
that n is not divisible by 20.
(1) Assume that 20 I n. Select a multiple of n arbitrarily. Denote
Hence
( a k a & l ...al
, where a k # 0. We have 20 I ( a k a & l .-al
a1 =0, 2 I ( U k U & l " ' u 2 ) 1 0 .
This implies that a 2 is even. Therefore,
( a k a & l ...a1 > l o is not alternating.
(2) We will prove that if n is a positive integer and is not
divisible by 20, then n must have a multiple which is alternating.
We will show three lemmas as follows. Then we divide the proof
into four cases.
Lemma 1 If the positive integer n is coprime to 10, then for any
1 E N , there exists K E N such that
Y
nrm6er m of
1
There must exist two of them congruent modulo n. Without loss of
generality, we may assume that
Mathematical Olympiad in China
230
x,
= xt (mod n>, s > t
1.
Then n I x, - xt. Moreover, since x, - xt = xFt 10t(zfl) and n is
0 and xSptis a positive integer. This
coprime to 10 , n I xSpt, s - t
completes the proof of Lemma 1.
Lemma 2 For any m E N, there always exists an alternative
number ,s with m digits such that its f i r s t digit can be 0 , and its last
>
decimal digit i s 5 , and 5" .I,s
Proof of Lemma 2 by inductive construction. Start with s1 = 5.
Suppose that ,s = (%awl .-a1 > l o is alternating, where a1 = 5 and a,
can be 0. In addition, 5"
A={
I.,s
Lets,
=
5". Denote
( 0 , 2 , 4, 6 , 8 } , when a, is odd,
(1, 3, 5 , 7, 9 } , whenamiseven.
Any two of them in A are not congruent modulo 5 , and 2" is
coprime to 5. So any two of the numbers in { 2"x I x E A } are not
congruent modulo 5. Select x E A such that 2"x =- 1 (mod 5 ) . Then
5 I 2"x
1. Let s+l = (mma,l
...a1 > l o . Then s+l is an alternative
number w i t h m + l digits, whereal = 5 , and its first digit can be 0. In
+
+
+
1) is a multiple of 5*'.
This
addition, s+l = x10" ,s = 5"(2"x
completes the proof of Lemma 2.
Lemma 3
For any integer m E N, there a l m y s exists an
alternative number t , with m digits such that i t s last decimal digit i s
2 , and
2 2 k f 1 II
t2kfl9
22 k f 3
II t 2 k f 2 .
Proof of Lemma 3 by inductive construction. Start with tl
= 2.
Suppose
(a2&1a2k.-al>lo is alternating, where a1 = 2 , and
22kt1 11 tzk+l. It follows from the property of the alternative numbers
that a 2 e l = a1 = 0 (mod 2 ) .
Denote A = { 1, 3 , 5 , 7 } . Any two of them in A are not congruent
modulo 8, and 52kf1 is coprime to 8. So any two numbers of
{ 52e1x I x E A } are not congruent modulo 8. Now diride the four
numbers in { 52kf1x I x E A } by 8 respectively. The original sentence
that
t2&1
=
International Mathematical Olympiad
2004
231
means the 4 numbers are divisible by 8. Since 52&1x with x EA is odd,
the remainders are 1, 3, 5, 7.
Let t 2&1= 22&1 I with I odd. Select x E A such that 52&1x =
+4 (mod
+
Then t2&2
8). Then 22 11 52&1x I . Let t 2&2 = ( m 2 & l .-a1 ) l o .
is alternating with2k+2 digits. In addition, a1 = 2 and t 2 ~
= X1O2&'
+
-
I
Since 22 11 52&1 x
+ I , we have 22&3 11
Moreover, suppose that t 2&2
22kt3 11
Let t2&3
t2&2.
number with 2k
= (4a2&2
+3 digits, and
Since 22kf3 11
of Lemma 3.
+I ) .
22&1 (52&1 X
t2k+l=
t2&2
t2kf2.
= ( a z e 2 ...a1 ) 10
...a1
, where a1 = 2, and
Then t 2&3 is an alternating
)lo.
t2k+3 = 52&2
22w
, we have 22kt3 11 t2&3.
+
t2kf2.
This completes the proof
-
Next, we discuss the four cases.
(1) If n is coprime to 10, it follows from Lemma 1 that there exists
k E N" such that 10 101..*101 is a multiple of n. The conclusion is
nmzberkofl
equivalent to selecting I = 1 in Lemma 1.
(2) If n is not divisible by 5, and n is divisible by 2, let n
such that no is not divisible by 2 , then
=
2"no
is coprime to 10. Select q
>
even. It follows from Lemma 3 that there exists an
.-a1 > l o with q digits, where a1 = 2. In
alternative number t , = (ho
m with
~TQ
addition, 2"of1
11 tmo. Hence 2" I
t,
. It follows from Lemma 1 that
there exists k E N" such that
1
-
oo...o
tamzber mo-1
of
1
0
-
oo...o
tamzber mo-1
of
1...1
0
Y
tamzberkofl
is a multiple of no. Let
Then P is alternating, and
-
oo...o
nrm6er mo-1
of
1
0
2
232
Mathematical Olympiad in China
nrm6er k of
1
can be divisible by 2"no, or n I P.
(3) If n is divisible by 5, and n is not divisible by 2, let n = 5"no
such that no is not divisible by 5, then is coprime to 10. Select q
m w i t h q even. It follows from Lemma 2 that there exist an alternative
number ,s = ( a , ...a1)10 w i t h q digits, whereal = 5 and% can be
>
0. In addition, 5"o
I ,s . Hence 5" I ,s . Using the same argument as
an alternative number P such that 5"no I P and the
(2), there exists
last decimal digit a1 = 5.
(4) If n is divisible by 10, and n is not divisible by 20, let n = 10%
with
odd, then satisfies the assumption in case(1) or in case (3).
It follows from (1) and (3) that there exists ( a k . * * a l ) l O an alternative
multiple of , where a1 = 1 or 5. Now Let P = (ah ...a1 0)Io. Then
P is an alternative multiple of n.
In conclusion, n should be a positive integer and should not be
divisible by 20.
2005
(Merida, Mexico)
The 46th IMO (International Mathematical Olympiad) was hosted by
MQida in Mexico during July 8 - 18 in 2005. 92 countries and 513
contestants participated.
The leader of Chinese IM02005 team was Xiong Bin who was from
East China Normal University, deputy leader was Wang Jianwei who
was from University of Science and Technology of China, observer was
Chen Jinhui who was from the High School Attached to Fudan
University. In IM02005, China came first among the nations with total
International Mathematical Olympiad
2005
233
points of 235. Here are the results:
Ren Qingchun
Yaohua Middle School,
Tianjin
42 points
gold medal
Diao Hansheng
the Second Middle School
attached to East China Normal
University
42 points
gold medal
Luo Ye
the High School Attached to
Jiangxi Normal University
42 points
gold medal
Shao Xuancheng
High School Affiliated to
Fudan University
42 points
gold medal
Shenzhen Middle School
35 points
gold medal
No. 2 Middle School of
Shijiazhuang, Hebei
32 points
silver medal
Kang Jiayin
Zhao Tongyuan
First Day
9:OO - 13:30 July 13, 2005
ints are chosen on the sides of an equilat ral triangle ABC:
A1 , A2 on BC , B1 , B2 on C A , and Cl , C, on AB. These points
are the vertices of a convex hexagon A1A2 B1 B2 CI C, with sides of
equal length. Prove that the lines AlB2, BIG and CIA2 are
concurrent. (proposed by Romania, average score 2.61. )
Solution (posed by Diao Hansheng)
Assume AlA2 = d, and AB = a. Construct an equilateral triangle
&Bo COwith side of length a -d. Points A’, B’ , C’ are chosen on the
sides of triangle A,BoCo such that A ’Co = A2C, B ’A0 = &A, and
C’BO = C,B.
Therefore ,
A’Bo
= a-d--A’&
Similarly,
= BC-A1A2-A2C=
BA1.
Mathematical Olympiad in China
234
B’Co
= BIG,
C’AO
=
CIA.
Since
LB1 CA2
= LB’CoA’,LB2ACI
=LB’A~C’,LC,BA~
= LC’B~A’,
thus
ACBlA2 Z ACOB’A’, a A B 2 C 1 Z aAoB’C’,
AC,BAl Z ABOC’A’,
which implies that B’C’
equilateral.
=
C’A’
= A’B
’ = d and triangle A’B ’C’ is
so
LAB2C1 = LAoB’C’ = 180°-LC’B’A’-LA’B’Co
=12Oo-LA’B’Co,
LciBzBi
= LBlAzAi.
In view of B2 Cl = B1 B2 = A2 B1 = AlA2 = d , triangles Cl B2B1
and B1 B2Al are congruent, implying that B1Cl = Al B1. Together
with CI C, = A1 C, , we show that C,B1 is the perpendicular bisector of
AlC1 and C,B1 is the height of triangle AIBICl on side AICl.
Similarly, CIA2 and A1 B2 are the altitudes of triangle Al B1Cl to the
sides Al B1 and B1Cl respectively.
Therefore the lines A1 B2 , B1 C2 and CIA2 are concurrent.
&
*
a
Let a1 , a2 ,
be a sequence of ..itegers with infin,,ely many
International Mathematical Olympiad
2005
235
positive terms and infinitely many negative terms. Suppose that
for each positive integer n , the numbers al, a2 , , a, leave n
different remainders on division by n. Prove that each integer
occurs exactly once in the sequence. ( proposed by Holland,
average score 3.05. )
Solution (posed by Ren Qingchun)
First we will show that every positive integer will occur at most
once in the sequence. Note that if ai = a j = K (i < j ) , then two
numbers among a1 , a2 , , aj , say ai, a j , are congruent modulo j ,
which is impossible.
Let X k , yk be the greatest and the smallest number among a1 ,
a2 9 "', ak respectively. Then X k - yk
k - 1. For if X k - yk
k,
without lose of generality , ai = X k , a j = yk , ai- a j = 1 K , then i,
j
K 1. Therefore two numbers among al, a2 , , a j , say ai, a j ,
are congruent modulo 1 , which is impossible.
Now we will show that for every integer t ( Y k
t
X k ) , there
exists an integer s(l
s
K) such that a, = t. For, if al, a 2 ,
ak E { u E Z I yk
u
X k , u # t }, then the sequence has X k - yk
<
>
< <
>
< <
< <
< <
..a,
different values at most. Note that X k - yk
<K 1<K. Therefore two
-
numbers among a1 , a2 , . *, U k have the same value, which is a
contradiction to the above argument.
Now for any integer m , since there are infinity many negative and
positive numbers in the sequence, it is trivial to see that there exists a
p such that up >I m 1. By a similar argument, there
exists a positive integer q such that a4 <- I m I . Denote r = max{p , q } ,
positive integer
then x ,
> I m I , y , <-I
m
I , i. e.
y,
<m < X k .
The above arguments
lead us to conclude that there exists a positive integer s such that a, =
m. We conclude that every number must appear exactly once in the
sequence.
@@$! Let x , y and z be positive real numbers such that xyz
that
> 1. Prove
Mathematical Olympiad in China
236
'-'>
2-2
+
by Korea(R. 0.
0. (proposed
>>
Solution (posed by Kang Jiayin)
We shall prove
0
So we only need to prove 0in the case when xyz
Since
=
2
x5+4+2
c2
=c
=
2
+xyz($
=
+ 22)
-
x4
x4+y3z+yz3
1,
1.
International Mathematical Olympiad 2005
the left hand side of
While
237
0holds.
n
by the AM-GM inequality,
x4
+x 4 +y3z + yz3 2 4 2 y z ,
x4 + y 3 z + y 3 z + y 2 2 2 4 x y 2 z ,
x4
+ yz3 +yz3 +y 2 2 24xyz2 ,
y3z+yz3 3 y 2 2 ,
the summation of the above four inequalities leads to
Therefore
c2 + $ + 2
X2
-
2
-
=s4+y3:+yz3
This is just the right hand side of inequality 0.
Comments Boreico Iurie from Moldova won a special prize for his
outstanding solution. Observe that
2 -2
2-2
2+$+2-2(2++2+2)
Therefore
Mathematical Olympiad in China
238
>O.
Second Day
9:30 - 13:30 July 14, 2005
@$@ Consider the sequence al , a2 ,
defined by
~,=2"+3"+6"-l(n=l,
2, ..*).
Determine all positive integers that are relatively prime to every
term of the sequence. (proposed by Poland)
Solution (posed by Luo Ye)
First, we will prove the following result: for a fixed prime p ( p 5) ,
>
2PP2
+3PP2 +6PP2
-
1 = 0 (mod
p).
0
Since p isaprimeno less than 5, (2, p ) = 1, (3, p ) = 1, and(6,
p ) = 1. By Fermat's little theorem, we have
2P-I
= 1 (mod p ) ,3P-' = 1 (mod p ) , 6P-I = 1 (mod p ) .
Therefore
3*2P-1+2*3P-1+6P-1
=3+2+1
= 6 (modp),
i. e.
6 2PP2 + 6
3PP2 + 6
6PP2 = 6 (mod p ) .
Simplifying gives
+3PP2 +6PP2 1 = 0 (mod p ) .
So 0holds, and ap2
2PP2 + 3PP2 + 6PP2 1 = 0 (mod p ) .
2PP2
-
=
-
It is trivial that al = 10 and a2 = 48.
For any integer rz greater than 1, it has a prime factor p. If
International Mathematical Olympiad
2005
239
p E ( 2 , 3 ) , then(n, a z ) > l . I f p 2 . 5 , then(n, a,z)>l.
Therefore
we can claim that every integer greater than 1 does not match the
condition.
Since 1 is co-prime to every other positive integer, 1 is the only
number satisfying the condition.
Let A B C D be a given convex quadrilateral with sides B C and
AD equal in length and not parallel. Let E and F be interior
points of the sides BC and AD respectively such that B E = DF.
The lines A C and BD meet at P , the lines BD and EF meet at
Q,the lines EF and A C meet at R . Consider all triangles PQR
as E and F vary. Show that the circumcircles of these triangles
have a common point other than P. (proposed by Poland)
Solution (posed by Zhao Tongyuan)
Since BC and AD are not parallel, the circumcircles of triangle
APD and BPC are not tangent to each other. Otherwise, construct a
common tangent line 9 3 ’ through tangent point P, then
LDPS
= LDAP,LBPS’ = LBCP.
It follows from the equality L D P S = L B P S ’ that L D A P =
L B C P . ThenAD // BC, which is in contradiction with the condition.
Let the second common point of the circumcirles of triangles BCP
and DAP be 0, which is fixed. With loss of generality, let 0 be an
interior point of triangle DPC. We will prove that the circumcirle of
triangle PQR passes through 0 as E and F vary.
Connect the lines O A , OB, O C , OD, OE, OF, OP, OQ, OR ,
as show in the figure. Since B, C , 0, P and 0,P , A , D are
concyclic, then
LOBC
= LOPC,
LOPC
= LADO*LOBC
= LADO.
Similarly, L O C B = L D P O = L D A O .
Together with AD = B C , we get A O B C Z AODA. So
OB
= OD,LOBE = LODF.
Mathematical Olympiad in China
240
Note that BE = DF, so the triangles OBE and ODF are
congruent, giving OE = OF and OB = OD.
The equalities LFOE = / F O B
L B O E = L B O F LFOD =
/ B O D imply that the triangles BOD and FOE are similar. This means
L E F O = L B D O , i.e. LQFO = L Q D O , so the points Q , F , D , 0
are concyclic. Therefore
+
+
LRQo = L F D O .
Since 0, P , A , D are concyclic, we have L F D O = LAD0 =
L R P O , so LRQo = LRPO. We conclude that the points 0 , R , P ,
and Q are concyclic, i. e. the circumcircle of PQR passes through 0 .
In a mathematical competition 6 problems were posed to the
2
contestants. Each pair of problems was solved by more than - of
5
the contestants. Nobody solved all 6 problems. Show that there
are at least 2 contestants who each solved exactly 5 problems.
(proposed by Romania)
Solution (posed by Shao Xuancheng)
Assume there were rz contestants C1, C, , , C, , and the six
problems were PI , P 2 , P 3 , P4, Ps , Pg.
Cbsolved both
Let S = {(CbiPi, P j ) I l < K < n , l < i < j < 6 ,
Piand P j } .
Now we will count I S I .
Let xij be the number of contestants having solved both Piand P j
2
2n + Theref ore
(1 i <j
6). By hypothesis, xG > TnHxG
5
<
<
~
'.
International Mathematical Olympiad
2005
241
If the number of contestants who solved exactly five problems
was at most one, by hypothesis, no contestants has solved all
problems, so the other contestants each solved four problems at most.
a, be the number of problems solved by
Let al , a2 ,
C, respectively. Without loss of generality,
contestants C1 , C, ,
we can assume that 5 a1 a2
a,
0.
If 4 al , then 4 a k ( l < K 4),
..a,
..a,
>
> > > > >
>
<
this is in contradiction with 0.
Therefore al
It is trivial that n 2. If a,
3 , then
>
<
= 5,
4
>a2 > >a,.
>
So a,
4, i. e. a2 = a3 = =
This is also in contradiction with 0.
a, = 4.
Assume, without loss of generality, that the five problems Cl
solved were PI , P2 , P3 , P4, Ps , and the number of contestants who
solved the problem Pj (1 j 6) was bj (1 j 6). Then
<<
bl+b2+"'+bfj
<<
=a1
=5
+U2+"'+U,
+4(n
-
1)
= 4n
+ 1.
0
0
Consider
j=2
j=2
Since the problem PI has been solved by bl contestants, and one of
them solved five problems and the others solved four problems,
Combining 0and
0we obtain
Mathematical Olympiad in China
242
which can be rewritten as bl
Similarly, bk
2n
> -,
3
> -.2n3
for K
=
1, 2, 3 , 4,5.
0
2n+1
@
Consider
6
- 2n+l,
cxj6>c7j=l
j=l
By a similar argument, since exactly b6 contestants solved P6, and
each of these contestants solved four problems, we have
c
6
Xj6 =
0
3b6.
j=l
Together with @ and
0we obtain
3b6 >22n+1,
which we can rewrite as b6
If n
> 2n+3 1.
@
~
+ 0 (mod 3 ) , then 2.3 is not an integer. By 0we obtain
which implies
8
Together with @ and 8 we obtain
bl +bz
+..a
+b6
>6
2n+ 1
3
-= 4 n + 2 ,
0.
2n
Therefore n = 0 (mod 3 ) , and @ implies b6 >-.
3
which is in contradiction with
are both integers,
2n
Since b6 and 3
International Mathematical Olympiad
Together with
2006
243
0and @ we obtain
bl+b2+"'+b6
2n 2n
>5*-+-+1
3
3
=4n+1.
0
Compare with 0,
the 0should be an equality, so are 0and @.
Hence
Consider
c
l<Kj<S
xij
>
c
l<KSS
5
=
2(2n+1)
=
4n+2.
@
Since one contestant solved P I , P 2 , P 3 , P4, P5, whereas b6
contestants solved exactly three problems of PI , P 2 , P3, P4, P 5 , and
n -1-b6 contestants solved exactly four problems of PI , P2 , P3 , P4,
P5 , we have
=6n+4-3b6
= 4n+1,
which is in contradiction with @. This completes the proof.
2006
(Ljubljana, Slovenia)
The 47th IMO (International Mathematical Olympiad) was hosted by
Slovenia in Ljubljana during July 6 - 18 in 2006.
The leader of Chinese IMD 2006 team was Li Shenghong who was
from Zhejiang University and deputy leader was Leng Gangsong who
was from Shanghai University. The Chinese IM02006 team came first
Mathematical Olympiad in China
244
among the nations with 6 golds. Here are the results of the six
contestants :
Liu Zhiyu
No. 1 Middle School Attached to
Central China Normal University
Shen Caili
Zhejiang Zhenhai High School
37 points gold medal
Deng Yu
Shenzhen Senior High School
35 points gold medal
Jin Long
the Affiliated High School of
Northeast Normal University
Gan Wenyin
I
I
Yaohua Middle School , Tianjin 34 points gold medal
Ren Qingchun
I
No. 3 High School of WISCO
I
I
31 points gold medal
First Day
9:OO - 13:30July 12, 2006
@&@Let ABC be a triangle with incentre I . A point P in the interior
of the triangle satisfies
L P B A+LPCA
=LPBC+LPCB.
Show that AP AI , and that the equality holds if and only if P = I.
Solution Since L P B A L P C A = L P B C L P C B , we get
+
2(LPBC+LPCB)
+
=LPBC+LPCB
+LPBA+LPCA
=LABC+LACB,
i.e. L P B C + L P C B
=
i(LABC+LACB)
=
9 0 " - y1L B A C ,
LBPC = 90"+2 LBAC.
On the other hand L B I C
=
90"
+ 21 L B A C .
-
Hence L B P C
=
International Mathematical Olympiad
2006
245
L B I C , B, C, I, P are concyclic.
It is easy to show that the middle point M of arc BC is the center
of circumcircle of triangle BIC. This is because M is also the point
where A I intersects the circumcircle of triangle ABC. Furthermore,
MI=MB.
so
From triangle APM,
AP
+ PM > AM
c"
=
AI +IMP.
Therefore AP > AI.
AI +IM
=
The equality holds if
and only if P lies on the line segment AI, that is
I=P.
A
Let P be a regular 2006-gon. A diagonal of P is said to be good if
its endpoints divide the perimeter of P into two parts, each
consists of an odd number of the sides of P. The sides of P are by
definition good.
Suppose P is partitioned into triangles by 2003 diagonals, no
two of which have a common point in the interior of P. Find the
maximum number of isosceles triangles having two good sides
that could appear in such a configuration.
Solution Let ABC be a triangle in the partition. Here are AB denotes
the part of the perimeter of P outside the triangle and between points A
and B, and similarly for arcBC and arcCA. Let a , b and c be the number
of sides on arcAB, arcBC, arc CA respectively. Note that a +b c =
2006. By parity check, if an isosceles triangle having two good sides, these
sides must be two e q d sides.
We call such isosceles triangles good.
Let one of the good triangles be A B C with A B = A C , Also,
inscribe our polygon into a circle.
+
Mathematical Olympiad in China
246
If there is another good triangle in arcAB, the two equal good
sides cut off an even number of sides in arcAB. Since there is an odd
number of sides in arcAB , there must be one side not belonging to any
good triangle. The same holds for arcAC.
So every good triangle corresponds to at least two sides of P.
Hence there are no more than 2 006/2 = 1 003 good triangles.
And this bound can be achieved. Let P = AlA2A3 .*.A2006. Draw
diagonals between AIA2&1(1< K
1 002) and &&IA2&3 (1 K
1 001). This gives us the required 1 003 good triangles.
<
< <
@@& Determine the least real number M such that the inequality
I &(a2 b2) +k(b2 2 )+ wz(2 -a2> I <M a 2 +b2
-
-
+2 > 2
holds for all real numbers a , b and c.
Solution Factorizes the left side of the above inequality and the
problem is reduced to finding the smallest number M that satisfies the
inequality
I (a-b>(b-cC)(c-ua)(a+b+c)
Letx= (a-b), y = (b-c),
the inequality becomes
I<M(a2
z = (c-a),
+ +
+b2
+2>2.
a n d s = (a+b+c).
Then
with the property that x y z = 0.
Since x + y + z = 0, without loss of generality, we can suppose x ,
y to have the same sign and both positive (otherwise we can replace
a , b , c by - a , - b , - c > .
Now, for any fixed x + y = 2 m , let x = y = m, so z =- 2m, the
left side gets greater and the right gets smaller and the inequality still
holds.
International Mathematical Olympiad
2006
(2+ y 2 +2 +.?>2 2 ( ( x + d2+ 2 +
247
.q2
=(6m2 + s ~ ) ~ ,
By the AM-GM inequality,
(6m2 +.?>2
=
(2m2 +2m2 +2m2
3 ( 4 J W > 2
1 6 f i M 1 m3Sl
9
so
=
+ s ~ > ~
161/21 m3sl
2 I 2m3S~
i.e. M>- 9 4 3
16 '
The conditions for the equality can now be stated as x = y ,
2m2 =.?,or restated as2b=a+c, ( c - d 2 = lSb2. Settingb= 1yields
a=1--1/2,
3
c = 1 + T 1 3/ 2 .
2
We can conclude that M = @ is indeed the smallest constant
16
satisfying the inequality, with the equality for any triple (a, b, c>
3
3
proportional to (I - y 1/2, 1, 1 y 1/2) up to permutation.
+
Second Day
9:OO - 13:30 July 13, 2006
Determine all pairs ( x , y > of integers such that
+2" +2-1
1
= y2.
Solution It is easy to show that x 2 0. Since (- y>2= y2 , we only
need to find all solutions with y > 0.
Ifx=O, theny=&2;
Now y is odd, let y = 2 n + l . Then2"(2"+' +1> = 4 n ( n + l > . It
is clear that there is no solution f o r x = 1, 2.
Mathematical Olympiad in China
248
~ s s u m e x > 2 . ~ i n c e ( 2 " ,2"+' +I>
=
1, ( n , n + l >
=
I, -4no r
2"
4(n + l) are integers.
2"
Casel: L e t 4 n = a * 2 " , thena2 * 2 " + 4 a = 8 * 2 " + 4 .
Ityields2"=
>
4(1 - a )
8, so a = 1, 2. In both cases we get a contradiction.
a2 -8
Case 2: Let 4(n+ 1) = a 2", then a2 2" -4a = 8 2" +4. It yields
2"
= 4(1+a)
a2 -8
2 8 , sou
= 1,
2, or 3. It is easy to check that onlya =
3 is good. So x = 4 , and y2 = 529.
Thus we have the complete list of solutions ( x , y ) : (0, 2),
(0, -2), (4, 23), (4, -23).
%&@ Let P ( x ) be a polynomial of degree n > 1 with integer
coefficients and let k be a positive integer. Consider the
polynomial Q ( x ) = P(P(.- P ( P ( x ) > .->>,
where P occurs k
times. Prove that there are at most rz integers t such that
Q(t> = t.
Solution The claim is obvious if every integer fixed point of Q is a
fixed point of P itself. In the sequel, assume that this is not the case.
Take any integer xo such that Q(xo> = xo and P ( x 0 ) # X O . Define
inductively xi+l = P ( x i > for i = 0, 1, 2, ..., then x k = X O .
It is evident that
u -v
I
P(u>- P(v>, for distinct integers u , v .
(Indeed, if P ( x ) = c a i x i then each u - v
I ai(ui -vi>.
)
(1)
Therefore
each term in the chain of (nonzero) differences
XO-Xl,
xl-x29
'"9
x&l-xk,
xk-Xk+l,
(2)
is a divisor of the next one; and since X k - Xk+I = x0 - x 1 , all these
differences have equal absolute values. Take xm = min(xl , , X k ) ,
this means that x-1 - xm =- ( x , - ~ 4 1 ) .Thus X-1
= X+I
(# x m > .
International Mathematical Olympiad
2006
249
It follows that consecutive differences in the sequence ( 2 ) have
opposite signs. Consequently, xo, x1, x 2 ,
is an alternating
sequence of two distinct values. In other words, every integer fixed
point of Q is a fixed point of the polynomial P ( P ( x > > .Our task is to
prove that there are at most n such points.
Let a be one of them so that b = P ( a ) # a (we have assumed that
such an a exists). Then a = P(b). Take any other integer fixed point
a of P ( P ( x > >and let P(a> = G
,,’ so that P(p>= a. The numbers a and
b’ need not be distinct ( a can be a fixed point of P>, but each of a , b’
is different from each of a , b. Applying property (1) to the four
pairsof integers ( a , a ) , (b’, b ) , ( a , b ) , (b’, a ) , we get that the
numbers a -a and ,h- b divide each other , and also a- b and ,h- a divide
each other. Consequently
a-b
=+ ( p - a ) ,
a-a
=+ (P-b).
(3)
Suppose we have a plus sign in both instances: a - b = ,G- a and a a =p- b. Subtracting yields a - b = b - a , a contradiction, as a # b.
Therefore at least one equality in (3) holds with a minus sign. This
means that a p = a b; equivalently a b - a - P(a> = 0.
Denote a +b by C. We have shown that every integer fixed point
of Q other that a and b is a root of the polynomial F(x) = C - x P ( x ) . This is of course true for a and b as well. Since P has degree
n > 1, the polynomial F has the same degree. So it cannot have more
than n roots. Hence the result.
+
+
+
GB Assign to each side b of a convex polygon P the maximum area
of a triangle that has b as a side and is contained in P. Show that
the sum of the areas assigned to the sides of P is at least twice the
area of P.
Solution Every convex (212)-gon , of area S , has a side and a vertex
that jointly span a triangle of area not less than S / n .
Proof of the lemma
By main diagonals of the (2n)-gon we shall mean those which
Mathematical Olympiad in China
250
partition the (2n)-gon into two polygons with equally many sides.
For any side b of the (2n)-gon, denote by &, the triangle AB P where
A , B are the endpoints of b and P is the intersection point of the
main diagonals AA ’, BB ’. We claim that the union of triangles &, ,
taken over all sides, covers the whole polygon.
To show this, choose any side AB and consider the main diagonal
AA ’ as a directed line segment. Let X be any point in the polygon,
not on any main diagonal. For definiteness, let X lie on the left side
of the ray AA ’. Consider the sequence of main diagonals AA ’, BB ’,
GC’, , where A , B, C, are consecutive vertices, situated to the
right of AA ’.
The n-th item in this sequence is the diagonal A ’A (i. e. AA ’
reversed), having X on its right side. So there are two successive
vertices K , L in the sequence A , B , C , before A’ such that X still
lies to the left of KK ’ but to the right of LL ’. This means that X is in
the triangle & , where I ’ = K ’L ’. We can apply the analogous
reasoning to points X on the right of AA ’ (points lying on the main
diagonals can be safely ignored). Thus indeed the triangles &, for all
b jointly cover the whole polygon.
The sum of their areas is no less than S. So we can find two
opposite sides, say b = A B and b’ = A’B ’ (with AA ’, BB ’ being main
diagonals) such that [ &, ] + [ &,/ ] > S / n , where [.-] stands for the
area of a region. Let AA ’ and BB ’ intersect at P. Assume without loss
P B ’.
of generality that P B
Then
>
[ABA’]
=
+ [ P B A ’ ] > [ A B P ] + [ P A ’B ’1
+Cab’] > S / n ,
[ABP]
=[&,I
proving the lemma.
Now, let P be any convex polygon, of area S, with m sides
al ,
a,. Let Sibe the area of the greatest triangle in P with side
ai. Suppose, contrary to the assertion, that
..a,
International Mathematical Olympiad
Then there exist rational numbers q1 ,
qi
2006
, qm such that
251
c
qi
=
2 and
>Si/S for each i.
Let rz be a common denominator of the m fractions q1 ,
Write qi
=
ki/n; so
cki
=
, qm.
2n. Partition each side ai of P into ki
equal segments, creating a convex (2n)-gon of area S (with some
angles of size 180"), to which we apply the lemma. Accordingly, this
induced polygon has a side b and a vertex H spanning a triangle T of
area [TI S/n. If b is a portion of a side ai of P , then the triangle
W with base ai and summit H has area
>
[W]=ki*[T]>ki*S/n=qi*S>Si,
in contradiction with the definition of Si.This completes the proof.